303 Cost and Management Accounting
303 Cost and Management Accounting
PROGRAMME COORDINATOR
Dr. Sabiha Khatoon, CDOE, Jamia Milli Islamia
COURSE WRITERS
Dr. Bibhu P. Sahoo, Department of Commerce, SGTB Khalsa College, University of Delhi, New Delhi
Block: 1: to 4 (Block: 1 Basic Concepts, Block 2 Material and Labour, Block 3: Overheads , Block 4: Method of Costing)
Unit : 1 to 12 (Unit 1: Nature and Scope of Cost Accounting, Unit 2: Cost Classification and Cost Sheet, Unit 3: Account
for material (Nature and Scope), Unit 4: Accounting for labour, Unit 5: Accounting for Overheads,
Unit 6: Absorption of Overheads, Unit 7: Single or Output Costing, Unit 8: Job Batch and Contract Costing,
Unit 9: Process Costing, Unit 10: Operating or Service Costing, Unit 11: Reconciliation of Cost and
Financial Accounts, Unit 12: Budgetary Control)
Prof. Abdul Aziz Ansari, Department of Commerce & Business Studies, Jamia Millia Islamia, New Delhi.
Block: 5 (Block 5: Management Accounting)
Unit: 13 & 14 (Unit 13: Financial Statement Analysis, Unit 14: Accounting Ratio)
C.A. Md. Feroz, Department of Humanities, National Institute of Technology, Kruchhetra, Haryana.
Block: 5 (Block 5: Management Accounting)
Unit: 15 & 16 (Unit 15: Budgeting-I, Unit 16: Budgeting-II)
All rights reserved. Printed and published on behalf of the CDOL by Hi-Tech Graphics, Jamia Nagar, New Delhi-110025
March, 2023
ISBN: 978-93-82997-84-9
All rights reserved. No part of this book may be reproduced in any form or by any means, electronic or mechanical, including
photocopying, recording or by any information storage or retrieval system, without permission in writing from the CDOE,
Jamia Millia Islamia, New Delhi.
Cover Credits: Anupma Kumari, Faculty of Fine Arts, Jamia Millia Islamia
CONTENTS
Block 3: Overheads
Unit 7: Single or Output Costing 131
Unit 8: Job Batch and Contract Costing 150
Unit 9: Process Costing 175
Basic Concepts
The present block deals with the introductory part of Cost accounting. The
learners will learn about the meaning, nature, scope of Cost Accounting. Further,
they will have the opportunity to learn about the cost classification, cost sheets
and accounting for materials. How the cost account is maintained and what is it
relevance in the business. The present block refers the following unit;
Unit 1: Nature and Scope of Cost Accounting
4
UNIT-1 NATURE AND SCOPE OF COST ACCOUNTING
LEARNING OBJECTIVES
STRUCTURE
1.1 Introduction
1.2 Limitation of Accounting
1.3 Distinction between Financial Accounting and Cost Accounting
1.4 Steps of the Installation of a Costing System
1.5 Advantages of Cost Accounting
1.6 Disadvantages of Cost Accounting
1.7 Essential of Good Cost Accounting System
1.8 Cost Centre and Cost Unit
1.9 Method of Costing
1.10 Introduction of Management Accounting
1.11 Characteristics of Management Accounting
1.12 Technique of Management Accounting
1.13 Difference between Management and Cost Accounting
1.14 Limitation of Management Accounting
1.15 Cost Control, Cost Reduction & Cost Management
1.1 INTRODUCTION
In brief, cost accounting is a branch of accounting, which has been develops to meet the
managerial needs of business. This concept is relatively a recent development due to
5
Cost and Management
Accounting
The following limitations of financial accounting have led to the development of cost
accounting.
Limitations of Accounting
According to the institute of cost and management accountant England (ICMA) has
defined cost accounting as “the process of accounting for the cost from the point at which
expenditure increase, to establishment of its ultimate relationship with cost centers and
cost units. In its widest sense, it embraces the preparation of statistical data, the
6
Nature and Scope of
Cost Accounting
From the above definition, it is clear that, cost accounting is a method of accounting in
which all costs incurred in carrying of an activity or accomplishing a purpose are
collected, classified and recorded. This data is then summarised and analyzed to serve at
a selling price or to determine where saving are possible. Cost accounting includes the
principles, convention techniques and system which are employed in a business to plan
and to control the unilisation of its resources. Cost accounting and expected future cost of
a product. Cost accounting is a system of determining cost of production.
Costing is a system which determines cost of a product by using any method like
arithmetic process, however, cost accounting denotes the formal accounting mechanism
by means of which costs are ascertained by recording them in the books of accounts.
Cost accounting is a science as well as an art. It is a science in the sense that it is a body
of systematic knowledge, which a cost accountant shared possess. It is an art as it
requires the ability and skills on the part of a cost accountant in applying the principle of
cost accountancy to various management problems. Cost accountancy includes costing,
cost accounting, cost control and cost audit.
7
Cost and Management
Accounting
8
Nature and Scope of
Cost Accounting
(9) SELLING PRICES:- Cost accounting guides the management in the fixation of
selling price.
(10) COST CONTROL:- Cost accounting helps in controlling costs with the help of
standard costing and budgetary control.
(11) ADVANTAGES TO SOCIETY: An efficient system will lower down the cost of
production and by it the whole society will be benifitted.
(12) ADVANTAGES TO WORKERS:- Undercost system incentive plans are
introduced which results in higher in productivity and higher earning to the
workers.
(13) ADVANTAGES TO GOVERNMENT:- Cost system produces suitable and
important data for the use by Government, trade unions etc. which are useful in
solving many problems like price fixing, wages fixation, settlement of disputes etc.
(1) EXPENSIVE: costing system is quite expensive and small concerns can not afford
for it. It will involve additional expenditure which will reduced profits of the
organizations. Costing system should be treated as an investment and it should
earned more profits. It should not a give a burden on a management. Unnecessary
recording should be avoided.
(2) FAILURE: if this system does not produced desired results it is set that there is a
fault in the system. Generally employee’s oppose this system because they look
this system with the suspected eyes. In order the make the system, a success one,
the utility of the system should be explained to the management and employee’s
and they should be convinced that the system is quite beneficial for them.
(3) UNNESSACERY:- maintenance of cost records is not necessary and it involves
duplication of work. The management must know about the detailed cost
information for taking suitable decision.
(4) NOT APPLICABLE:- costing system is not applicable in many industries. Every
costing system must be specially designed to meet the needs of the business. In
fact, applications of costing are very wide. All types of activities should consider
the use of cost accounting.
(1) DESIGNED SYSTEM:- the cost accounting system should be designed as per the
requirements of business. A common type system will not be useful to each organization.
10
Nature and Scope of
Cost Accounting
A cost centre is a location, person or item of equipment for which cost may be
ascertained and used for the purpose of control. Cost centre refers to a section of the
business, to which costs can be charged. The main purpose of ascertaining the cost is
control over the cost. The whole organization is divided into small parts and each section
is treated as a cost centre to which cost is ascertained.
Cost unit is a step which breaks up the cost into smaller sub division. Cost unit is divided
as unit of product, service or time in relation to which cost may be ascertained. A tone or
sugar and metre of clothes are cost units. Hence cost unit is an unit of measurement of
cost.
(1) UNIT OF PRODUCTION: a tone of steel, a metre of cable, a ream of paper etc. is
known as unit of production.
(2) UNITS OF SERVICE: Cinemas seats, Hotel room, consulting hours etc. are
include in units of service.
CONTRACT COSTING : Contract costing is big one and the unit cost is a contract,
which is a long duration and it may continue for more than a year.it is most suited to
construction of dams, roads, bridge, ship, building etc.
UNIT COSTING : This methods is used to when production is uniform and consist of a
single variety of the same product. If the product is produced in different grades, cost are
as curtained grade wise. per unit cost is calculated by dividing the total cost by the total
number of unit produced. this methods is applicable in mines, bricks, floormills, steel
production etc.
12
Nature and Scope of
Cost Accounting
SERVICE COSTING: This methods is used where services are produced for example
transport undertakings, hotels, electricity companies, cinemas, hospitals etc. this method
is a variation of process costing.
OPERATION COSTING : Any process may consist of a number of operations and this
costing involves cost ascertainment for each operation. it provides minutes analysis of
costs and ensure greater accuracy and better control.
BATCH COSTING: In this method the cost of a batch of identical products is a cost
unit. This method is used in readymade garments, shoes, tyres and tubes etc.
TECHNIQUES OF COSTING:
(2)MARGINAL COSTING : In this technique cost is separated into fixed and variable
costs. Marginal cost is related with the variable cost and fixed cost is treated as period
cost as attempts are not made to allocate this cost to individual cost units.it is transferred
to profit and loss account.this technique is used to study the effect on profit of the
changes in output.
maximizing profits and minimizing losses. In managerial accounting managers use the
provisions of accounting information in order to decide on the matters with in their
organization. which aids their management and performance of control functions.
Definitions
(i) Management accounting increases the efficiency of managers, and it helps them
to take suitable decision.
14
Nature and Scope of
Cost Accounting
(ii) In management accounting managers get reports from various corners, which
help them to take proper decision.
(iii) The strengths and weakness of the organization can be ascertained easily through
various techniques of management accounting.
(iv) Management accounting provides accounting information to the managers, which
facilitates them in taking suitable decisions.
(v) Management accounting is a managerial function which helps the managers in
preparing plans for business and in other activities.
(vi) Management accounting provides feedback to managers and it facilitates the
manager to control the concern in a better way.
(vii) Decision making: managements accounting helps the manager to take best
decision, so that best financial results may be obtained.
(viii) Management accounting helps the manager to take or make buy decisions in the
interest of the organization.
(ix) Management accounting is an integrated system, under which targets are
achieved through various activities. In its various parts, cost accounts, budgetary
control and financial accounts, statistics effort for the concern.
(x) Management accounting ensures a systematic process in which various methods
and techniques are used. Various activities are coordinated with each other and it
is a systematic effort for the concern.
(xi) Management accounting is related to future activities and managerial decisions
are taken accordingly. Thus managements accounting, various informations, facts
and figures are made available.
(xii) Management accounting is related to information systems and various facts and
systements are necessary oart of the system. It helps in smooth running &
operation of the concern.
(xiii) Management accounting is an analytical system, which explains the reasons for
profitability etc. of the concern. All these facts helps in getting to a correct
conclusion.
(i) Management accounting divides managerial activities into various units and it
helps in the preparation of budgets, cost centers and division of resources. A
close relation is maintained in various departments.
(ii) The objective of management accounting is to maintain cooperation among
various activities. For every department a separate budget is prepared. The
departments help each other and valuable figures and facts are provided.
(iii) On the basis of information received for management accounting valuable
decisions are taken by the managers and best alternatives are selected.
(iv) With the help of management accounting responsibilities are assigned to
different employees and they are expected to perform their duties in a decent
manner.
(v) Management accounting helps in the preparation of managerial policies. Various
plans are formulated through which efforts are to be made to achieve the desired
goals. For casts are made on the basis of this plans. Expected rate of return are
for casted and proper decisions are taken for investing the funds.
15
Cost and Management
Accounting
(vi) For the growth of the concern, it is essential that every member should take
active part in the working of the organization. For this purpose, persons working
in the organization are motivated. Policies are framed in the interest of
employees and efforts are made to keep their morale high.
(vii) Various parties may obtain important information regarding business with the
help of management accounting. Managers get information through reporting,
creditors get full information about loans, returns etc. investors get information
about rate of returns, suppliers get information about their payments through
various sources of accounting. In the organization, information system is
developed, so that all relevant information may be made available.
(viii) Management accounting helps in providing reports to managers which facilitates
them for taking decisions. Formulation of policies and control becomes easy on
the basis of these reports.
The difference between management accounting and cost accounting may be given as
under:
levels.
Records Management accounting records Cost accounting does not record
various operations. such operations.
Decision Various information assists Cost accounting does not help in
Making managers in formulating policies making decision.
and decision making.
Techniques Managers use various techniques Cost accounting does not use
of cost accounting such as ratio management technique.
analysis, budgetary control etc.
Nature Where cost accounts ends Cost accounting is related to
management account accounts determination of cost per unit etc.
begins.
Adviser Management accountant has a Cost accounting does not have a
higher position and acts as a main higher position and does not act as
advisor. a main advisor.
Assist Management accounting assists Cost accounting does not help
managers in day to day operations. managers in daily operations.
Scope The scope of management The scope of cost accounts is very
accounting is very wide, as it limited as it does not include
includes financial accounting and financial accounting and
cost accounting. management accounting.
(i) More expensive: management accounting is more useful for large scale
organizations on account of its expensive nature. The small concern are not
in a position to bear its burden. The technique of management accounting is
comparatively more expensive.
(ii) Delays in decision: many times the managers are not in a position to take
proper decision. On account of delayed process, relevance of decisions taken
become useless.
(iii) Manipulation: the figures abtained from management accounting may be
misused and there are possibilities of manipulations of information.
(iv) Lack of knowledge: in case of persons not taking decisions at proper time,
due to lack of managerial efficiency, the goal may be effected adversely.
(v) Dependency of data: management accountant has to depend on financial
accounting and cost accounting for accurate information of facts and figures.
If the figures are inaccurate, it will adversely effect the managerial decisions.
(vi) Resistance from existing staff: the existing accounting and management
staff may oppose the introduction of management accounting system in the
organization.
17
Cost and Management
Accounting
Business firms, now a day aims at producing the product at the minimum cost. It is
necessary in order to achieve the goal of profit maximation and wealth maximasation.
The success of financial management is judge by the action of the business executives in
controlling and reducing the cost of product.
Cost control
It means a search for better and more economical ways of completing each operation.
Cost control helps to eliminate inefficiencies and wastages with in the existing
environment to increase the profits and to improve the performance by minimizing the
costs. Cost control is defined as the regulation by executive action of the cost of operating
an undertaking. It aims at achieving the target sales. Cost control is the procedure where
by actual results are compared against the predetermined standards. The firm is expected
to adhere to the standards. Deviations of actual performance form the standards are
analysed and reported and corrective action are taken.
(i) Planning: Initially a plan or set of targets is established in the form of budget
and standards.
(ii) Communication: The next step is to communicate the plan to those whose
responsibility is to implement the plan.
(iii) Motivation: After the plan is put into action, evaluation of the performance
starts. Costs are ascertained and information about achievement is called and
reported to higher officials.
(iv) Appraisals: Comparison has to be made with the predetermined targets and
actual performance. Deficiencies are noted and discussion is started to
overcome deficiencies.
(v) Decision making: Finally, the reported variance are received. Corrective
actions and remedial measures are taken or set or target is revised depending
upon the administration understanding of the problem.
18
Nature and Scope of
Cost Accounting
Definition
cost control process involves setting targets and standers, ascertaining the variance
and taking corrective action. Some of the important features of cost are:
Advantages of Cost Control: some of the merits of cost control are discussed below:
Disadvantages of cost control: some of the disadvantages of cost control are discussed
as below.
Techniques of cost control: some of the techniques which are generally aploed in cost
controlling techniques are:
Budgetary control
Standard costing
Ratio analysis
Variance analysis
Inventory control
Cost Reduction
The process of identifying and eliminating unnecessary costs to improve the profitability
of a business is known as cost reduction. It refers to bringing down the cost of
production. Cost reduction is possible only when the business firms makes the optimum
19
Cost and Management
Accounting
Features of cost reduction : Some of the important features of cost reduction are:
Techniques of Cost Reduction: The various techniques used for cost reduction are as
follows:
reduction is identified
It predetermines the cost and which affirms a
5. Function then compares it with actual competitive advantage
6. Uses of Accounting performance for achieving for a longer time.
techniques. the targets.
It is a preventive function It does not lay down the
It heavily depend/upon cost standards in
accounting techniques. advance.
It is a corrective
function.
It may not involve the
use of accounting
techniques.
Cost Management
Cost management is the process of planning and controlling the budget of a business.
Cost Management is a form of management accounting that allows a business to predict
impending expenditures to help in reducing cost. Cost management is a term of recent
origin. It may be defined as follows, "Cost management identifies, collects, measures
classifies and reports information that is useful to management in determining the cost of
products or services, planning, controlling and decision making. Cost Management
includes both cost accounting and management accounting. This techniques helps in
revenue and profit planning. The main two areas for cost managements are (i) to focus on
value-added activities and (ii) to reducing consumption of cost drivers in value added
activities. In brief cost management is activities which is achieved by collecting,
analysing, evaluating and reporting cost information used for budgeting, estimating,
forecasting and monitoring cost. Cost management is process of planning and controlling
the budget of the project. It predicts the expenditure and reduce the project from going
over budget.
1.16 QUESTIONS
1. List out the different methods of costing and explain their practical application.
2. "A costing system that simply records costs for the purpose of fixing sale prices has
accomplished only a small parts of its mission". Discuss, What other functions does
costing perform?
3. "A govt. system of costing series as a means of control over expenditure and helps to
secure economy in manufacture" Discuss.
4. What are the essential principles of a good costing system? What are the objections
to the introduction of a costing system.
5. State and explain main difference between cost accounting and financial accounting.
21
Cost and Management
Accounting
Mittal & Maheswari, Elements of Cost Accounting, Shree Mahavir Book Depot
(Publishers) 2015.
Varshney J.C., Principles and Practice of Cost Accounting, Wisdom Publication
House (2009).
Mittal & Maheswari, Management Accounting, Mahavir Publication, 2015.
Arora M.N. Cost Accounting, Vikas Publishing House Pvt. Ltd. 2013.
Jain & Narang, Cost Accounting, Kalyani Publishers 1998.
Arora M.N., Management Accounting, Himalaya Publishing House, 2006.
22
UNIT-2 COST CLASSIFICATION AND COST SHEET
LEARNING OBJECTIVES
STRUCTURE
2.1 Introduction
2.2 Classification of Cost
2.3 Meaning of Cost Sheet
2.4 Specimen of Cost Sheet
2.5 Element of Cost Sheet
2.6 Questions
2.1 INTRODUCTION
A cost sheet is a report on which is accumulated all of the costs associated with a product
or production job. A cost sheet is used to compile the margin earned on a product or job,
and can form the basis for the selling of process on similar products in the future. It can
also be used as the basis for a variety of cost control measure. According to CIMA
London cost sheet, is „a statement which provides for the assembly of the detailed cost of
a centre or a cost unit.‟ It is also a periodic statement. The expenditure which has been
incurred up on product for a period is extracted from the financial books and the store
records and set out in a memorandum statement. If this statement is confined to the
disclosure of the costs of units produced dividing the period, it is known as cost sheet, but
where the statement records both total cost, profit and loss. It is known as statement of
cost or production account. If information derived from the books is set on usually in the
form of a statement, it is cost sheet but where it is set out in the form of an account
recordings the cost incurred, there being separate accounts to show also sell and profits, it
would be known as production or manufacturing account. It is desirable that beside total
expenditure incurred, cost per unit of input, cost per unit of output in case of each
element of cost should be calculated. Therefore “Cost sheet is a document, which
provides for the assembly of the detailed cost of a cost unit.” Cost sheet is a periodical
statement of cost showing in detail the various elements of cost of goods produced. It is
prepared at regular intervals. Comparative figures of the previous period may be shown
in the cost sheet.
When the details of cost sheet are shown in a T-shape account, it is known as production
account.
23
Cost and Management
Accounting
(i) It provides a comparative study of the cost of current period with that of the
previous period.
(ii) It represents the total cost as well as per unit cost.
(iii) It discloses the breakup of total cost into different elements of cost.
(iv) It acts as a guide in fixation of selling prices.
Cost may be classified into different categories depending upon the purpose of
classification, some of the important categories in which the costs are as follows.
1. Prime cost
(i) Manufacturing/Producing cost,
(ii) Administrative Cost,
(iii) Selling and distribution cost, and
(iv) Research & development cost
24
Cost Classification and
Cost Sheet
(i) Fixed cost: Fixed cost is a cost which does not change in total for a given time
period. For example, a retailer must pay rent and utility bills irrespective of sales.
This cost is a basic operating expenses of a business that cannot be avoided.
(ii) Variable cost: The cost which varies directly in proportion with every increase or
decrease in the volume of output or production is known as variable cost. Some
of the examples are: wages of labourers, cost of direct material etc.
25
Cost and Management
Accounting
(iii) Semi variable cost: A semi variable cost is a cost the contains both fixed and
variable cost elements. The fixed element of the cost will be incurred repeatedly
over time, while the variable element will only be incurred, irrespective of
volume. This concept is used to project financial performance at different level of
activity.
For example: commercial leases: which often have a fixed rent per month plus an
additional rent based on the amount of production or sales.
(iv) Product cost: The cost which are a part of the cost of a product rather than
expenses of the period in which they are incurred are called as product cost. For
example: cost of raw material and direct wages, depreciation or plan etc.
(v) Period cost: The cost which are not associated with production are called period
costs. They are treated as an expenses of the period in which they are incurred. It
may be fixed as well as variable cost. Such cost include general administration,
salesman salaries, commission etc.
26
Cost Classification and
Cost Sheet
(vi) Direct and indirect cost: The expenses incurred on material and labour which are
economically and easily traceable for a product, service or job considered as
direct-costs. For example: in the process of manufacturing of production of
articles, materials are purchased, labourers are employed and wages are paid to
them.
The expenses incurred on those items which are not directly chargeable to
production are known as direct cost. For example, salary of storekeeper, watch
man etc.
(i) Relevant cost Irrelevant costs: Relevant costs are those cost which change by
managerial decision. This cost have to satisfy at least two conditions (i) this cost
are those which are to used in future (ii) this cost changes with alternative.
Irrelevant cost are those which do not get affected by the decision. It does not
change by a decision of the management. All future cost are relevant cost and all
past costs are irrelevant cost
27
Cost and Management
Accounting
(ii) Marginal cost: It means the increase or decrease in the total cost of a production
run for making one additional unit of an item. It is computed in a situation where
the break even point has been reached. Marginal costs are variable costs
consisting of labour and material costs plus an estimated portion of fixed costs.
(iv) Uniform costing: Uniform costing is the application of the same accounting and
costing principles methods or procedures uniformly by various under takings in
the same industry. It is a particular technique which applies the usual accounting
methods. Like standard costing, marginal costing and budgetary control.
(v) Replacement cost: This is the cost or replacing an asset which is being used, for
example, the replacement cost of material is the present market price of material
on the date of issue to the production department. It is the actual cost to replace
an asset or an item or structure at its pre-loss condition. It means replace the
assets of a company or property of the same or equal value.
(vi) Shut down and sunk cost: A manufacturer or an organization may have to
suspend its operations for a period on account of some temporary problems. For
example, shortage of raw material, non-availability of skilled laboures etc. during
this period, no work is done yet, certain fixed costs, such as rent and insurance of
building, depreciation etc. for the entire plant will have to be incurred. Such cost
is known as shut down cost.
Sunk cost are historical or past cost. These are the costs which have been created
by a decision that will be made in the near future. For example: investment in
plant and machine, building etc.
(vii) Opportunity cost: This is the cost of an alternative that must be foregone on order
to pursue a certain action. This concept is used in problems of alternative choice.
For example, when a fixed deposit in a bank is converted in to debenture of a
company, interest on fixed deposit on the bank will not be received but interest
on debenture will be received. It means interest on bank fixed deposit has been
sacrificed to earn interest on debenture, therefore, the interest on bank deposit is
an opportunity cost for the investor who has purchased debenture.
(viii) Conversion cost: It is the total of direct wages and factory over heads. Hence,
conversion cost = production cost – cost of raw materials.
28
Cost Classification and
Cost Sheet
(ix) Imputed cost: A cost that is incurred by virtue of using an asset instead of
investing it or undertaking an alternative course of action. An imputed cost is an
invisible cost that is not incurred directly.
For example: When a building is owned then no rent is paid. The national rent or
the rental value of such a building is an imputed cost which is used for decision
making purposes.
(x) Out of pocket cost: Out of pocket costs in management accounting are expenses
that could be incurred or avoided depending on management‟s decisions. In other
words, out of pocket expenses is a potential future outlay of cash that-
management needs to decide whether or not make. It is an expense that requires a
future disbursement of cash. For example; depreciation of plant is not payable in
cash but is a part of the cost. It is not an out of pocket cost.
Cost sheet is a document, which provides for the assembly of the detailed cost of a cost
unit.” Cost sheet is a periodical statement of cost showing in detail the various elements
of cost of goods produced. It is prepared at regular intervals. Comparative figures of the
previous period may be shown in the cost sheet.
When the details of cost sheet are shown in a T-shape account, it is known as production
account.
(i) DIRECT MATERIAL: Since there will be only one product and process of
manufacture is also simple, the raw material if any is directly charged to the
production of the period in told. The openings stocks purchases and the closing
30
Cost Classification and
Cost Sheet
stock of raw material should not be shown separately but suitably adjusted to
give one figure of raw material consumed or use.
(ii) DIRECT LABOUR: The labor cost are directly charged to production is known
as direct labour.
(iii) DIRECT EXPENSES: Expense other then direct labour and direct material are
known as direct expense: for example excise duty, expense on decisions.
(iv) PRIME COST: The total of direct material consume, direct labour and other
direct expense is known as prime cost.
(v) OVERHEAD: It is to be noted that the term overheads has a wider meaning than
the term indirect expenses. Overheads includes the cost of indirect material,
indirect labor besides indirect expenses. Indirect expenses may be classified
under the following three categories –
b. Office and administrative expenses are not related to factory but they
pertain to the management and administration of business. Such expenses are
incurred on the direction and control of an undertaking. Examples are: Office
rent, lighting and heating, postage and telegrams, telephones and other
charges; depreciation of office building, furniture and equipment, bank
charges, legal charges, audit fee.
(vi) Work in progress: It rarely happens that unit initiated during a period is finished
in that very period. These incomplete units are called „work-in-progress‟. It may
be valued either on the basis of prime cost or works cost. However, its valuation
on works cost basis i.e. prime cost plus proportionate factory overheads, is more
31
Cost and Management
Accounting
(vii) Finished goods: Of the opening and closing stocks of finished goods are also
given, these must be adjusted before calculating cost of goods sold as follows:
Cost of production + opening stock of finished goods – closing stock of finished
goods = cost of goods sold
Illustration 1. compute cost of raw material purchased from the data given below
RS
Opening stock of raw materials 10,000
Closing stock of raw materials 15,000
Expenses of purchases 5,000
Direct wages 50,000
Prime cost 1,00,000
[B.com. (pass) Delhi]
Solution: COMPUTATION OF RAW MATERIALS PURCHASED
Particular RS RS
Prime cost 1,00,000
Less: direct Wages 50,000
Direct material 50,000
15,000
Add: Closing Stock 65,000
10,000
Less: Opening stock 5,000 15,000
Expenses on purchase 50,000
Materials purchased
Illustration 2. From the following information prepare cost sheet and find out the
amount of profit:
RS
Raw material purchased 24,000
Works overhead 20,000
Stocks:
Raw material as on 1st January, 2015 4,000
Finished goods (800 quintals as on 1st January, 2015 3,200
Work-in-progress:
1st January, 2015 960
st
31 January, 2015 3,200
Office and administrative overheads 1,600
Sales (finished goods) 60,000
32
Cost Classification and
Cost Sheet
Advertising discount allowed and selling cost is Rs. 0.40 per quintal. During the month
12,800 quintals of the commodity were produced.
[B.Com. Delhi]
Solution:
COST SHEET
Output: 12,800 quintals
Particulars Amount
RS
Opening stock of raw materials 4,000
Add: Raw materials purchased 24,000
Value of Raw Material 28,000
Add: Works Overhead 20,000
48,000
Add: Opening work-in-progress 960
48,960
Less: Closing work-in-progress 3,200
Works Cost 45,760
Add: office & Administrative Overheads 1,600
Cost of Production 47,360
Add: opening stock of finished goods 3,200
Cost of production of goods sold 50,560
Add: selling & distribution overheads”
Advertising, discount allowed & selling cost 5,440
(13,600*0.40) 56,000
Cost of sales 4,000
Add: profit (balancing figure) 60,000
sales
Illustration 3. Prepare cost sheet from the following data provided by Aruna Industries
Ltd. For the year ending 31st March, 2012:
Raw Materials Rs 15,000
Direct Labor RS 9,000
Machine Hours 900 hours
Machine hour rate Rs 5
Production 17,100 units
Sales 16,000 units
Selling Price per unit Rs 4
Selling overhead per unit 50 paise
Office overheads are 20% of works cost.
[B.com. (Pass) Delhi, 2007]
33
Cost and Management
Accounting
(i) Cost sheet, (ii)a statement showing profit for the period. [B.com Delhi,1991]
Solution:
Particular RS RS
Opening stock of raw materials 4,000
Add: raw materials purchased 15,000
Carriage inwards 1,000
20,000
Less: Closing stock of raw materials 5,000
Cost of raw materials consumed 15,000
Direct labor 9,000
34
Cost Classification and
Cost Sheet
STATEMENT OF PROFIT
PARTICULAR RS
Cost of production 34,200
Add: Opening Stock of finished Goods:
2,000 units @ Rs 1.50 per unit 3,000
Less: Closing Stock of Finished goods 37,200
3,1000 unit @ Rs 2 per unit 6,200
Cost of goods sold 31,000
Selling and distribution overheads
(16,000*Rs. 0.5) 8,000
Total cost 39,000
Profit 25,000
sales 64,000
35
Cost and Management
Accounting
36
Cost Classification and
Cost Sheet
Mittal & Maheswari, Elements of Cost Accounting, Shree Mahavir Book Depot
(Publishers) 2015.
37
Cost and Management
Accounting
38
UNIT-3 ACCOUNT FOR MATERIAL (NATURE & SCOPE)
LEARNING OBJECTIVES
STRUCTURE
3.1 Introduction
3.2 Material Control procedure
3.3 Pricing Issue of Material
3.4 Methods of Pricing of Materials
3.5 First on First out (FIFO)
3.6 Last ion First Out (LIFO)
3.7 Simple average method
3.8 Weighted average method
3.9 Techniques of Material Control
3.10 Material Turnover Ratio
3.11 Stock Levels
3.12 Economic Ordering Quantity.
3.1 INTRODUCTION
The term materials refers to raw materials which are used in production and assembling .
Here, the term raw materials means inventory. It also includes work-in, process goods
and completely finished goods. Material control aims at efficient purchasing of materials,
their efficient storing and efficient use or consumption. Further, it can be defined as a
comprehensive framework for the accounting and control of material cost designed with
the object of maintaining material supplies at a level so as to ensure uninterrupted
production but at the same time minimising investment of funds.
Material Control
It includes procuring, storing and supplying of materials required for production, at the
lowest cost per unit, consistent with the required quality and with the least investment in
inventory. In other words, material control involves systematic control and regulation of
procurement, storage and usage of materials so as to maintain smooth flow of production
with an optimum investment in inventory. Material control is to be done in three stages:
39
Cost and Management
Accounting
Important requirement of material control system: The following are the important
requirements of a successful material control system:
40
Account for Material
(Nature & Scope)
stock. Usually, a bin card is used to record the quantity of materials in stock for each
item.
When items of material have reached their recorder point, the storekeeper will make
a purchase requisition requesting the purchasing department to contact with
appropriate suppliers or vendors.
When the purchasing department receives the purchase requisition, the purchasing
officer will examine the different sources of supply for the purposes of securing the
highest quality materials at the lowest-reasonable price.
On the receipt of the goods, the stores department will inspect and compare the
supply with the purchase order.
When the departmental foreman receives a production order, he will give a material
requisition to the storekeeper. On the receipt of requisition, the storekeeper checks
for correctness and authorisation. If satisfactory the issue will be made and entered
the details in bincards. He then forwards the store requisition to account department.
When the account department receives the store requisitions, it will price each of the
items listed on it by appropriate pricing methods. Then the amount of materials issued
is charged to appropriate job or overhead account and the stock values are reduced.
The important point regarding pricing of the issue of material are discussed below:
The important methods of pricing of issue of materials to the production departments are:
First in First out Price (FIFO).
Last in First out price (LIFO).
Simple average price.
Weighted average price.
In this method issues of material are priced at the price of the latest purchase of
materials.
In this method, closing stock gets priced at the price of the earliest purchases of
material lying unissued as per record, That is the closing stock is valued at the oldest
price.
In this method, both issue and stocks are priced at the actual cost.
This method suggest cost of material should be used on the most recent price.
43
Cost and Management
Accounting
FIFO Method
Date Receipts Issue Balance
201 Quantit Rat Amoun Quantit Rat Amoun Quantit Rat Amou
5 y Units e t Rs. y Units e t Rs. y Units e nt Rs.
Rs. Rs. Rs.
Jan 1000 1.00 1000 - - - 1000 1.00 1000
1
Jan 260 1.05 273 - - - 1000 1.00 1000
10 260 1.05 273
Jan - - - 700 1.00 700 300 260 1.00 300
20 1.05 273
Jan 400 1.15 460 - - - 300 1.00 300
21 260 1.05 273
400 1.15 460
Jan 300 1.25 375 - - - 300 1.00 300
22 260 1.05 273
400 1.15 460
300 1.25 375
Jan - - - 300 1.00 300 340 1.15 391
23 260 1.05 273 300 1.25 375
44
Account for Material
(Nature & Scope)
60 1.15 69
Jan - - - 240 1.15 276 100 1.15 115
24 300 1.25 375
Jan 500 1.10 550 - - - 100 1.15 115
25 300 1.75 375
500 1.10 550
Jan - - - 100 1.15 115 20 1.25 25
26 280 1.75 350 500 1.10 550
Solution:
LIFO
Receipts Issues Balance
Date- Quantity Rate Amount Quantity Rate Amount Quantity Rate Amount
2015 Pieces Rs. Rs, Pieces Rs. Rs, Pieces Rs. Rs,
Jan. 1 200 2.00 400
Jan. 5 100 2.20 220 200 2.00 400
100 2.20 220
Simple Average Method: This method of pricing materials is based on the principle that
materials are to be issued to production or job on an average price. A simple average of
prices is calculated by dividing all the prices of different lots of a material in stock at the
time of issue by total number of prices. For example, before making an issue of250 units
the following three lots of materials are in store :
400 units @ Rs. 35 per unit
900 units @ Rs. 36 per unit
200 units @Rs.37 per unit
35 36 37
Simple Average Price = Rs.36
3
Thus, according to this method, 350 units will be issued at Rs. 36 per unit.
Though this method is very easy to operate, but it is crude and usually produces
unsatisfactory results. The value' of closing stock may be quite absurd. Moreover,
materials are not charged out of actual cost and, therefore, a profit or loss will usually
arise out of pricing.
This method assumes that the cost of materials used and closing stock are valued at the
weighted average cost. According to CIMA, Weighted Average price is, "a price which is
calculated by dividing the total cost of materials in the stock from which the materials to
be priced could be divided by the total quantity of materials in that stock". Thus, the
weighted average price takes into account both the price and quantity of the materials in
store.
LIFO Method
Date Receipts Issue Balance
201 Quantit Rat Amoun Quantit Rat Amoun Quantit Rat Amou
5 y Units e t Rs. y Units e t Rs. y Units e nt Rs.
Rs. Rs. Rs.
Dec. - - - - - - 500 2 1000
1
Dec 1000 3 3000 - - - 500 2 1000
5 1000 3 3000
Dec. 1500 4 6000 - - - 500 2 1000
8 1000 3 3000
1500 4 6000
Dec. 1500 4 6000 500 2 1000
10 100 3 300 900 3 2700
1600 6300 1400 3700
47
Cost and Management
Accounting
48
Account for Material
(Nature & Scope)
The ABC analysis is a business term used to define an inventory categorization technique
often used in inventory management. It is also known as selective inventory control.
ABC method of inventory control means 'Always Best Control' which aims at giving
more attention to those items which are costliest and less attention to those materials
which are used in a large quantity~but are of comparatively low value. All items of
materials are classified into three types—A, B and C. 'A' items are costliest but used in
the smallest quantity, 'B' items are of middle value and middle quantity and 'C' type of
items are those which are cheap but used in largest quantities.
On the basis of Physical quantities and value of material used a table is given for the
better understanding of the students
49
Cost and Management
Accounting
Periodic Inventory System : Under this system stock-taking for each item of material is
done only at the end of a specified period, generally at the end of the accounting year.
Periodically, all inventory items physically verified are valued according to the method of
valuation, i.e., FIFO (First in First Out). LIFO (Last In First Out), etc. The value of the
closing inventory, arrived at, is used to compute Cost of Goods sold in the following
manner:
Periodic inventory system is particularly suited for small value items and materials
consumed.
Disadvantages
Important disadvantages of just in time invenotry systme are given below.
- This system is very difficult to implement and maintain
- It may not give accurate forecasting.
51
Cost and Management
Accounting
This ratio is popularly known as Inventory turnover Ratio. This ratio shows how many
times a company's inventory is sold and replaced over a period. The days on the period
can then be divided by the inventory turnover formula to calculate the days it takes to sell
the inventory on hand or inventory turnover days. The inventory turnover formula
measures the rate at which inventory is used over a measurement period. One can use the
formula to see IF a business has an excessive Inventory Investment in comparison to its
sales level, which can indicate either unexpectedly low sales or poor inventory planning.
This ratio establishes a relationship between cost of raw materials consumed during the
period and average stock of materials during the period. Thus, the formula is
Cost of materials consumed during the period
Inventory turnover Ratio =
Average stock of material during the period
Inventory turnover ratio may also be calculated in terms of days. This is done by dividing
365 days by the Inventory turnover ratio. Thus, the formula is
Days of the period
Stock conversion period =
Stock turnover rate
Note|: If the length of the stock turnover period as short, it is considered good for
organisation. It will indicates that material is considered to be fast moving.
Illustration 5
The following information relates to the year 2014-15 given below:
Particulars Material X (Rs.) Material Y (Rs.)
Opening Stock 30,000 90,000
Closing Stock 15,000 60,000
Purchases 200,000 125,000
Solution:
Opening Stock + Closing Stock
Average Stock =
2
30,000+15,000
Average stock (Material X) = Rs.22500
2
90,000+60,000
Average stock (Material Y) = Rs.75, 000
2
Calculation of cost of Material Consumed:
Cost of Material consumed = Opening Stock + Purchase Closing Stock.
Material X = 30,000 + 200,000 15,000 = Rs. 215,000
Material Y = 90,000 + 125,000 60,000 = Rs. 155,000
Cost of Material Consumed
Stock Turnover Ratio =
Average Stock
52
Account for Material
(Nature & Scope)
Rs.215,000
Material X = 9.55 Time.
Rs.22,500
Rs.155,000
Material Y = 2.07 time.
Rs.75,000
Stock turnover ratio can be expressed in number of days.
365 (No. of days in a year)
Formula =
Stock turnover Ratio
365
Material X = 38 days (approx.)
9.55
365
Material Y = 176 days (approx.).
2.07
Comment:
Thus, material X is fast moving than Material Y. Because it takes only 38 days to
consume the average stock. However, in the case of material Y, it takes 176 days to
consume the average stock.
Illustration 8
Calculate Danger stock level from the following particulars.
Daily consumption = 100 to 200 units.
Maximum re-order period for emergency purchase = 5 days
Solution:
Danger level = Normal Consumption Maximum re-order period for emergency
purchase
= 150 5 = 750 units.
56
Account for Material
(Nature & Scope)
EOQ is the order quantity which minimizes the total inventory holding cost and ordering
cost. EOQ represents the most favourable (optimum) quantity to be ordered each time
fresh supplies are required In purchasing materials one of the important problem to be
faced is how much to buy at a time. If large quantities are purchased, the cost of carrying
the inventory would be high. On the other hand, if purchases are made frequently in small
quantities, ordering cost will be high. In deciding the optimum quantity to order, the
problem is to balance the above two costs, carrying costs against the ordering costs. The
sum of the two represents the total costs of ordering and holding inventories.
Illustration 11.
Monthly usage : 250 units
Carrying cost: 10% of inventory value
Cost of placing and receiving one order : Rs 30
Cost of materials per unit: Rs. 5
Find out the EOQ.
57
Cost and Management
Accounting
Solution.
2UP
EOQ
S
Here, U = 250 12 = 3000 units
P = Rs. 30
10
S = Rs. 5 = Re. 0.50
100
2 3000 30
EOQ
50
3, 60, 000 = 600 units.
The EOQ is given in figure-A. In this figure, we plot carryings costs and ordering costs.
Carrying costs vary directly with the size of the' order (units) whereas ordering costs vary
inversely with the size of the order. If these two cost curves are added, the sum represents
the total costs of ordering and holding inventories. The total inventory-associated costs
curves has a minimum point and this point is known as EOQ or optimum order quantity.
The total costs line declines first when costs of ordering are spread over more units.
Subsequently, it begins to rise when the decrease in average ordering cost is more than
offset by the additional carrying costs. Thus, point E represents the optimum order which
minimise the total costs of inventory management.
EOQ can also be determined using the tabular method to determine EOQ, a comparison
of total costs at different order size is made. The optimum order quantity is corresponding
to the minimum total costs- at this point both carry-it and ordering costs & would
balance.
Illustration 12: From the following data calculate EOQ by applying tabular method.
Annual Demand = 3000 nits
Cost per unit = Rs. 5.00
Carrying Cost 10% of Inventory
Ordering cost = Rs. 30 per order.
58
Account for Material
(Nature & Scope)
Illustration 13: From the following data calculate EOQ and RC-Order level.
Maximum consumption per week = 6600 kg.
Recorders period = 6 to 8 weeks
Annual Usage = 2000 units.
Cost of placing an order (B) = Rs. 100
Purchase price of raw material (C) = Rs. 20 per kg. Carrying cost of inventory (S) =
20% per annum
2UP 2 20, 000 100 40, 00, 000
EOQ 10, 00, 000 1, 000 kg.
S 20 20% 4
Re-order Quantity (EOQ) = 1,000 kg.
i) Re-order level = Maximum Consumption Maximum Re-order period
Maximum consumption per week = 600 kg.
Maximum Re-order period = 8 weeks
Re-order level = 600 8 = 4,800 kg.
Illustration 14:
A manufacturer requires 800 units of a certain component monthly. This is currently
purchased from a regular supplier at a Rs.50 per unit. The cost of placing an order is
Rs.60 per order and the annual carries cost is rs.5 per piece. What is the economic order
quantity (EOQ) for placing order?
2UP
Solution: EOQ =
S
U = Annual usage = 800 12 -= 96000 units
P = Cost of placing an order = Rs.60
S = Storage cost per unit = Rs. 5
2 9600 600 1152000
EOQ 230400 480 units.
5 5
59
Cost and Management
Accounting
Illustration 15:
A company manufactures 5,000 units of a product per month. The purchase price of raw
material is
Rs. 20 per kg. The re-order period is 4 to 8 weeks. The consumption of raw materials
varies from 200 kg to 600 kg per week. The cost of placing an order is Rs.100. The
carrying cost of inventory is 20% per annum. You are required to calculate:
(i) Re-order quantity;
(ii) Re-order level. (50 weeks in a year)
Sol. (i) Minimum consumption per week = 200 kg.
Maximum consumption per week = 600 kg.
Average consumption per week =
Minimum Consumption + Maximum Consumption
2
200kg+600kg
=
2
800 kg
= 400 kg.
2
No. of weeks in a year = 50
Annual consumption (A) = 400 50
, = 20,000 kg
QUESTIONS
(i) Re-order quantity (ii) Reorder level (iii) Minimum level (iv) Maximum level
SUGGESTED REFERENCE
Mittal & Maheswari, Elements of Cost Accounting, Shree Mahavir Book Depot
(Publishers) 2015.
Varshney J.C., Principles and Practice of Cost Accounting, Wisdom Publication
House (2009).
Mittal & Maheswari, Management Accounting, Mahavir Publication, 2015.
Arora M.N. Cost Accounting, Vikas Publishing House Pvt. Ltd. 2013.
Jain & Narang, Cost Accounting, Kalyani Publishers 1998.
Arora M.N., Management Accounting, Himalaya Publishing House, 2006.
61
BLOCK-2
The present block produces the concepts of accounting for labour, overheads and
absorption for overheads. In this block the learners will learn about the concept,
relevance and types of labour and overheads. How is it treated in the Cost Sheet
of a business organisation? The present block refers the following unit;
Unit 4: Accounting for Labour
62
UNIT-4 ACCOUNTING FOR LABOUR
LEARNING OBJECTIVES
STRUCTURE
4.1 Introduction
4.2 Components of Labour cost
4.3 Control of Labour Cost
4.4 Elements of Labour cost
4.5 Accounting for Idle Time
4.6 Labour Turnover
4.7 Incentive Plans
4.8 Questions
4.1 INTRODUCTION
Labour cost is classified as direct and indirect. They form the labour cost which in turn
forms a significant percentage of the total cost of production in a manufacturing or
service organization and there is need to exercise maximum care to minimize these costs
minimizing cost does not mean reducing cost but means getting optional and efficient
productivity form the employees. Labour cost represents the human contribution to
production. It is an important cost factor requiring constant measurement, control and
analysis. Labour cost can be classified into (i) Direct Labour cost and (ii) Indirect labour
cost. The portion of wages and salaries cost unit is known as Direct labour cost and the
cost which cannot be directly related with the production of specific goods or services is
known as Indirect labour cost. Storekeeper, clerical staff etc. are examples of indirect
labour cost.
Labour costs represents the various components of expenditure, which includes the
following:
(a) Monetary Benefits For example: (i) Basic Wages (ii) Dearness Allowance; (iii)
Employer's Contribution to Provident Fund ; (iv) Employer's Contribution to Employees'
State Insurance (ESI) Scheme; (v) Production Bonus; (vi) Profit Bonus ; (vii) Old age
Pension ; (viii) Retirement Gratuity.
(b) Fringe Benefits or Labour Related Costs for example: (i) Subsidised Food ; (ii)
Subsidised Housing ; (iii) Subsidised Education to the children- of the workers ; (iv)
Medical facilities; (v) Holidays Pay; (vi) Recreational facilities.
Notes: Fringe benefits are indirect forms of employee compensation. The total of these
benefits given to the employee should be sufficient enough to attract and retain the labour
force.
Labour costs constitute a significant portion of the total cost of a product. Labour cost
may be excessive due to inefficiency of labour, more wastage of material by labour due
to lack of proper supervision, high labour turnover, idle time and unusual overtime work,
inclusion of bogus workers in the wages sheet and many other related factors. Therefore,
economic utilisation of labour is a need of the present day industry to reduce the cost of
production of the products manufactured or services rendered. Management is interested
in labour costs on account of the following two main reasons.
(a) to use direct labour cost as a basis for increasing the efficiency of workers; and
(b) to identify direct labour cost with products, orders, jobs or processes for
ascertaining the cost of every product, order, job or process ;
(i) Personnel Department, (ii) Engineering Department, (iii) Rate or Time and Motion
Study Department, (iv) Time-keeping Department, (v) Cost Accounting Department, (vi)
Pay-roll Department.
64
Accounting for
Labour
The functioning of some of these departments in relation to control of labour cost is given
below :
(i) Personnel department. This department is mainly concerned with the selection,
training, recruitment, placement, promotion etc. of workers in an organisation. It keeps
full details of each employee regarding date of employment, date of birth, particulars of
previous employment, wage rate, job specialisation medical history etc.
(ii) Engineering department. This department provides technical services and performs
the following functions:
(i) Preparation of job specifications. (ii) Conducting time study and motion study. (iii)
Looking after working conditions to. ensure safety of work force. (iv) Making job
analysis. (v) Supervising various production activities and exercising quality control.
4. Payroll department. The main task of the 'Payroll department is the preparation of the
payroll or the wage-sheets. In big enterprises this work is generally performed by a
separate 'Payroll department. Payroll accounting is that part of the main Accounting
department which is concerned with the computation and payment of wages and then
recording them in the books of accounts. Usually, following are the main functions of the
Payroll department:
(iv) Time keeping department. The main function of this department is to keep a record
of workers' time. Recording of workers' time is needed not only for the purpose of
attendance of workers and calculation of their wages but also for the purpose of cost
analysis of the job and apportionment of labour cost over various jobs. The time keeping
department is an important part of a firm's system of accounting and control of labour
costs. Its two main functions are:
Time keeping: A large factor, should have a separate time keeping office situated close
to the factory gate. The main function of this department is to maintain the exacted time
for which each worker has worked. It keep detailed records of any one of the following :
65
Cost and Management
Accounting
The various methods used for recording time may be classified into two groups:
(i) Manual methods : It includes both (a) Attendance Register method, and (b) Disc
method.
(a) Attendance Register Method : In this method an Attendance Register is kept at the
entrance gate and the workers' attendance in and out of the factory gate is being noted.
1. Disc Method : In this method each worker is assigned a number. A board with metal
discs bearing workers number booked. It is kept near factory main gate. At the time of
enteries into the factory, the worker remove their respective disc and place in a box on
empty tray.
(ii) Mechanical method: Big organisation use this method for recording workers time. it
includes:
(b) Dial time recorders: A dial time recording has a dial around the clock with several
holes each of which bears a number corresponding to identification number of the
concerned workers. When a worker will enter into the man gate of the factory, he is to
press the appropriate name. Then the time recorder will automatically record the time.
(c) Card time recorder: In this method each worker is given a card called time card. It
contains the employees history like name, department, overtime, total time for all week
days etc. When a worker enter into the company he picks up his card from out of rack
and puoxes the time of his entry with the help of machines
Time -Booking : In addition to recording of worker's attendance in and out of the factory
or department, it is also necessary to record the time spent on each work order, Job or
operation as well as the particulars of work done. This is known as time booking. The
66
Accounting for
Labour
purpose of time booking records is to ascertain the time taken by a worker, to complete a
job or an operation. Time booking records help in facilitating the ascertainment of labour
cost of every job and control over wastage of time. Large organisations keep time
recording clocks for maintaining records of time spent on each work, Job or operation. In
small organisation this may be done manually. The nature of documents maintained for
this purpose will vary depending upon the size of the organisation: The various methods
of time booking are as follows :
(i) Daily time sheet, (ii) Weekly time sheet, (iii) Job card, (iv) Combined time and Job
card, (v) Labour cost card and (vi) Piece work card.
1. Daily Time Sheet : Daily time sheet ie a daily record of the work done by a worker. It
shows the jobs on which he worked and the time spent on each job. The worker
completes the sheet everyday and hands it over to the foreman for signature. This method
is suitable in small organisations where the number of workers as well as number of jobs
handled by them are not many. This method is not very popular because there is too
much clerical work in this method.
2. Weekly Time Sheet : Weekly time sheet is similar to daily time sheet except that
weekly time sheets is prepared for a week in place of every day. The number of
documents to be prepared is considerably reduced. The weekly time sheet gives a ready
consolidation of the total hours shown in the clock cards. It is possible to have a better
supervision since worker's performance in a single work can be obtained readily.
3. Job Card : A Job card may be defined as a card containing the de-tails regarding the
time spent by a worker on a particular job. A job card is maintained for each job and is
allotted to the workers who work on that particular job. The purpose of "this card is, to
keep a close watch on time spent by a worker on each job and to calculate the labour cost
of each job. Job card travels along with the job from worker to worker and each worker
records time spent by him on it. It is useful when each job passes through different
workers of different grades.
4. Piece work card: Some time workers are paid wages on the piece work basis. In such
cases a piece work load is maintained for recording aspect and wages of each workers.
(i) Basic wages: The definition of Basic Wages' has been defined under section
2(b) of the EPF and MP Act, 1952 as below.
67
Cost and Management
Accounting
Section 2(b), "Basic wages" means all emoluments which are earned by an employee
while on duty or on leave or on holidays with wages in either case in accordance with the
terms of the contract of employment and which are paid or payable in cash to him, but
does not include
The amount of gross wages is calculated by adding up wages at ordinary rates plus
overtime premium.
Example. A worker's time card shows that he has worked for 20 hours in the factory out
of which he has worked 4 hours overtime. The ordinary rate is Re. 1 per hour while the
overtime rate is to be at 1½ times of the ordinary rate.
(iii) Overtime premium. Sometimes workers work for extra time over and above the
normal hours of work. According to the Indian Factories Act 1948, overtime is the time
worked for more than nine hours per day or 48 hours per week. Usually, workers are paid
at a higher rate for overtime than that for normal time. According to law, overtime has to
be paid at double the normal rate of payment. This coupled with the fact that overtime
comes at the end of the day when fatigue has set in, should make it clear that the jobs
done in overtime are rather costly. Overtime jobs are thus costly because of higher rate of
wage payment, low productivity of workers and additional expenses on lighting etc. The
Production Manager or some higher authority should authorise the overtirne because
there is a danger that workers may develop this as a habit.
68
Accounting for
Labour
The following treatment should be given to overtime wages in the following cases:
Overtime required because of some abnormal conditions like floods, earthquakes etc.
should be charged to Costing Profit & Loss A/c.
Overtime when required or seasonal pressure should be taken as an item of factory
overheads,
When overtime is direct i.e., it can be identified with individual jobs, it should be
charged entirely to that particular job or work order concerned.
When overtime is required to make up for any shortfall in production due to some
fault of management or some unexpected development, it should be charged to
Costing Profit & Loss A/c.
(iv) Set-up time. In factories, it is absolutely normal that some time is lost or
consumed on the frequent setting up of the machines. This loss of time may be
because of changes from one job to another or because of break-downs etc. This
time means the time when the machines just lie idle. This is also called 'Making
machines ready time'. The cost of all such hours lost in setting up of the
machines (including wages of workers as well as other overheads) may be spread
over the jobs actually completed. Sometimes even separate rates (Machine hour
rate) are computed for running and this setting-up time. Through this method,
one can ensure the full absorption of the manufacturing overheads. Alternatively,
the cost of this setting-up time may be treated like 'Idle time' dividing the same
into normal setting up time and abnormal setting-up time.
(v) Idle time : Normally the time booked to different job is less than the gate time.
The, difference represents time lost which is termed as idle time. It is the time for
which the employer pays to the worker but does not get any output in return. For
example, if an employer pays to the worker for 8 hour's work but the job cost
shows that he has worker for 7 hours, then 1 hour is the idle time. Idle time
increases the labour cost component of a job. Hence, there is a need to control
idle time. To control idle time, it is necessary to know the causes of idle-time.
II. Administrative Causes It arises out of administrative decisions, e.g., when there is a
surplus capacity of plant and machinery which the management decide not to work, there
may be some idle time. This is represented by idle facilities.
69
Cost and Management
Accounting
(i) controllable i.e., (idle time due to many of the productive causes is subject to control
internally.)
(ii) uncontrollable (idle time arising out of economic and administrative causes).
Idle time may be reduced by taking fallowing preventive measures :
(i) Proper planning of production in advance,
(ii) Careful supervision.
(iii) Proper maintenance of machines and power-systems,
(iv) Timely purchase of raw materials, tools and provisions.
Accounting for Idle time: Accounting for idle time in cost accounting to depends on
whether it is normal or abnormal idle time.
Normal idle time: This idle time arises out of normal process i.e. out of nature of human
labour. It is unavoidable, natural and anticipated. Therefore the cost of normal idle time
should be charged directly to production. It should be directly charged to costing profit
and loss account.
Abnormal Idle time: it is that kind of idle time which is not arises due to production or
work environment. Rather, it arises due to detective planning, inefficiency or accidents.
The cost of abnormal idle time is not the part of cost of production. It should be directly
charged to costing profit and loss account.
Refers to the rapidity with, which workforce in an organisation changes.. It is the rate of
change in the composition of the labour force in the organisation. Let us assume that in a
factory there were % employees on an average during the year 2015 and 100 persons left
100 100
the company during this period. So, the labour turnover will be 5%
2, 000
Generally, it is measured by the following formula :
No. of employees leaving during the period
100
Average number of employees employed during such period
1. Personal Causes: It includes: (i) Change for better job; (ii) death; (iii) retirement
because of old age; (iv) change for better working conditions, better environment; (v)
change for secured job; (vi) marriage especially of women workers; (vii) illness and
accident ' rendering the worker permanently incapable of doing and work; (viii) domestic
need and responsibility etc.
Effect of Labour turnover : Some labour turnover is inevitable, e.g. employees retire
every year, some may leave due to personal reasons, such as,, marriage, starting their
own business. However, excessive labour turnover is harmful and costly. It results in
increased cost of production due to the following reasons :
Labour turnover rate can be measured by the application of any one of the following
three methods :
In case it is desire to relate the labour turnover rate for a months or a fraction of a month
to annual rate of turnover,. This may be done by calculations "Equivalent annual rate'
with the help of the following formula:
Turnover Rate 365
Equivalent Annual Rate = 100
No. of Days in the Relevant Period
Calculates the labour turnover rate and equivalent annual rate under the different method.
Solutions:
(1) Separation method
Number of Separation
Labour Turnover Rate = 100
Average Number of Workers
10 30
100
1000 1100
2
40
100 3.81%
1050
3.81 365
Equivalent annual turnover Rate = 46.36%
30
Replacement Method Labour Turnover Rate
Number of Replacements
= 100
Average Number of Workers
20
100
1000 1100
2
72
Accounting for
Labour
20
100 1.90%
1050
Equivalent annual turnover rate
1.90 365
23.17%
30
(iii) Flux Rate Method
No. of Separation + No. of Replacament
Labour Turnover Rate = 100
Average Number of Workers
40 20
10 5.71%
1050
5.71 365
Equivalent annual turnover rate = 69.52%
30
Illustration 2: From the following data, find out the Labour Turnover Rate by applying:
[B. Com
(Delhi)]
(i) Flux method;
(ii) Replacement method;
(iii) Separation method.
No. of workers on the payroll:
At the beginning of the month 500
At the end of the month 500
During the month, 5 workers left, 20 persons were discharged and 75 workers were
recruited. Of these, 10 workers were recruited in the vacancies of, while the rest were
engaged for and expansion scheme.
Solution:
73
Cost and Management
Accounting
No. of Replacements
(ii) Replacement Method = 100
Average no. of workers
10
= 100 = 1.82%
550
No. of Separations
(iii) Separation Method : 100
Average No. of Workers
25
100 4.55%
550
Cost of labour turnover. Cost of labour turnover is classified into the following two
categories:
1. Preventive costs. These are the costs which a company incurs to prevent the labour
from leaving the job. These costs are incurred in the form of giving more and more
benefits and incentives to workers like free medical facilities, transport facilities,
rent-free housing, education for children of workers etc.
2. Replacement costs. These costs are those which are incurred after the workers have
left the company.
Examples are:
Cost of advertising the post.
Cost of recruitment and selection of workers.
Cost of accidents caused by new workers.
Loss of materials wasted by new workers.
Loss of production due to the delay in appointing new workers. ;
Loss of production due to the slow work by new and untrained workers.
The cost of labour turnover is distributed between normal cost and abnormal cost. The
"normal cost will inflate the labour cost while any abnormal cost is transferred to Profit
and Loss A/c.
Illustration 3: From the following particulars, calculate the labour turnover rate and
labour flux rate.
Number of workers at the beginnign of the year = 4000
Number of workers at the end of the year = 4200
During the year 40 workers leave while 160 workers are discharged. 600 workers are
required during the year, of these 150 workers are recruited because of leavers and the
rest are engaged in accordance with an expansion scheme.
74
Accounting for
Labour
Solution .
4000 4, 00
Average number of workers during the year = 4100
2
Number of workers replaced during the year
Labour Turnover Rate = 100
Average number of workers during the year
50
= 100 3.66%
4100
Numer of additions + separations during the year
Labour Flux Rate : = 100
Average number of workers during the year
600 200
100 19.51%
4100
Labour flux rate denotes total change in the composition of labour force due to additions
and separations of workers.
Illustration 4. From the following data provided to you find out the Labour Turnover
Rate by applying :
(a) Flux Method
(b) Replacement Method
(c) Separation Method
No. of workers on the payroll —
At the beginning of the month 500
At. the end of the month 600
During the month, 5 workers left, 20 persons were discharged and 75 workers were
recruited. Of these, 10 workers were recruited in the vacancies of those leaving, while the
rest were engaged for an expansion scheme.
(I.C.WA. Inter, June 1993)
Solution
(a) Labour Turnover Rate by Applying Flux Method
No. of additions + No. of separations
= 100
Average number of workers during a period
75 5 20 100
100 100 18.2%
1 550
(500 600)
2
(b) Labour Turnover Rate by Applying Replacement Method
Number of workers replaced
100
Average number of workers
10
100 1.8%
1
(500 600)
2
75
Cost and Management
Accounting
A suitable personnel policy should be framed for employing the right man for the
right job and giving a fair and equal treatment to all workers.
Good working conditions which may be conducive to health and efficiency should be
provided.
Fair rates of pay and allowances and other monetary benefits should be introduced.
Maximum non-monetary benefits (i.e., fringe benefits) should be introduced.
Distinction should be made between efficient and inefficient workers by introducing
incentive plans whereby efficient workers may be rewarded more as compared to
inefficient workers.
An employee suggestion box scheme should be introduced whereby workers who
suggest improvements in the method of production should be suitably rewarded.
Men-management relationships should be improved by encouraging labour
participation in management.
In addition to the above point, the personnel department should prepare periodical reports
on the labour turnover listing out the various reasons due to which workers have left the
organisation. The report should be sent to the management with the necessary
recommendations so that corrective measures may be taken to reduce labour turnover.
Wages to workers is one of the most complex problems in a democratic country like
India. There is no single unique method of wage payment which is acceptable to both
parties i.e. employers and the workers. However before deciding the wages for employee
the employer should take in to account the following points.
76
Accounting for
Labour
(i) Time Wage System: In this system, wages are paid to the worker on an hourly,
daily weekly and monthly basis. For example, a worker is paid at the rate Rs. 2
per hour and he has spent 250 hour during month of June. Therefore, his wages
for the month of June will be = 250 Hour Rs.2 = Rs.500.
(ii) Piece Rate system: Under this system, wages are paid to worker on the basis of
pieces completed by the worker. For example, a worker is paid at the rate of Rs.
3 per unit and produce 10 units during the day, he will get Rs. 103 = R.s 30 on
that day.
(iii) Premium and Bonus Plan: The main object of a premium plan is to increase the
production by giving an incentive to the workers in the form of higher wages for
less time worked. Some of the important premium a plans are discussed here
below:
77
Cost and Management
Accounting
It is a simple combination of time-speed basis of payment. The feature of the plans ara :
(a) Worker is paid at an hourly rate for the time for which he has worked.
(b) A standard time is determined and if a worker finishes a job before the time
fixed, he is paid a bonus for the time saved, besides the wages for the actual time
spent by him on the job.
(c) The amount of bonus is 50% of the time saved in case of Halsey Plan and 30% in
case of Halsey-Weir Plan and is allowed at the same hourly rate at which he is
paid for actual time worked.
Therefore, his total emoluments are the aggregate of guaranteed hourly wages for actual
time worked plus the amount of bonus. It can be expressed by way of a formula :
Total Earnings = T R + (S T) R 50% (or 30%)
Where T = Time Taken
R = Hourly Rate
S = standard Time
Thus, total earnings are = Time taken Hourly rate plus Time saved Rate 50% (or
30%)
Illustration 5: Calculate from the following data Halsey Plan Incentive System.
Standard time fixed = 30 hours
Time Taken = 6 hours
Hourly Rate = Rs. 2 per hour
Solution:
The total earnings of the worker under Halsey Plan shall be computed as follows :
Minimum wage = Time taken Hourly rate = 20 Rs.2 = Rs.40
Amount of Bonus = Time Saved x Rate x 50%
= (Standard time Actual time) Rate 50%
1 1
(30 20) 2 10 2 Rs.10
2 2
Total Earning of Rs. 40 + Rs. 10 = Rs. 50.
(i) Rowan Plan. Under this method, the worker is again guaranteed wages at the ordinary
rate for the time taken by him to complete the job or operation. The difference between
the Halsey Premium Plan and the Rowan Premium Plan is only in the calculation of the
bonus. Under the Halsey Plan, bonus is a fixed percentage of the wages of the time saved
whereas under the Rowan Plan bonus is that proportion of the wages of the time taken
which the time saved bears to the standard time allowed. Thus, bonus under this system
will be calculated as :
ST
TR
S
78
Accounting for
Labour
ST
and the total earnings will be calculated as : T R TR
S
where T = Time taken (Actual Time)
S = Standard Time (Time Allowed)
R = Rate per hour.
Solution
S (Standard Time) = 20 hours
T (Time taken) = 15 hours
R (Rate) = Rs. 1.50 per hour
Total Earnings = T R + 50% (S T)R
50
15 1.50 (20 15) 1.50
100
= 22.50 + 3.75 = Rs. 26-25
= Rs. 26-25
Total wages for 15 hours
Therefore, effective rate of earning per hour
Total Wages 26.25
= Rs.1.75
Actual Time Taken 15
Note Percentage of bonus is to be taken 50% when it is not given.
79
Cost and Management
Accounting
could act as sufficient incentive for them to produce more and he has accordingly given
this assurance to the workers. As a result of this assurance, the increase in productivity
has been observed as revealed by the following figures for the current month.
Hourly rate of wages (guaranteed) Rs. 2
Average time for producing 1 piece by one workers at the
previous performance 2 hours
(This may be taken as time allowed)
No. of working days in the month 25
No. of working hours per day for each worker 8
Actual production during the month 1,250 units
You are Required : to (i) calculate effective rate of earnings per hour under Halsey
Scheme and Rowan Scheme
(ii) Calculate the savings to Mr. A in terms of direct labour cost per piece under the above
schemes.
(iii) Advise Mr. A about the selection of the scheme to fulfil his assurance. (I.C.WA.
Inter, Dec. 1993)
Solution
Calculation of lime wages of 10 workers per month
= No. of working days in a month No. of working hours per day each worker
No. of
workers x Hourly rate of wages
= 25 8 10 Rs. 2 = Rs. 4,000
Calculation of time saved per month
Time allowed per piece 2 hours
Time allowed for actual output of 1,250 units 2.500 hours
Less : Actual time taken by 10 workers working for 25
days @ 8 hours per day per worker (10 x 25 x 8) 2,000 hours
Time saved 500 hours
(c) Calculation of Bonus under Halsey Scheme
Bonus = 50% of time saved x hourly rate
50
= 500 (hours x Rs-2 = Rs.500
100
Total wages under Halsey Scheme = Time Wages + Bonus
= Rs. 4,000 + Rs. 500 = Rs. 4,500
Calculation of Bonus under Rowan Scheme
Time saved
Bonus = Time Wages
Time allowed
500 hours
= Rs.4, 000 Rs.800
2,500 hours
Total wages to be paid to 10 workers under Rowan Scheme
= Time Wages + Bonus = Rs. 4,000 + Rs. 800 = Rs. 4,800
80
Accounting for
Labour
Solution
(a) Calculation of the Amount of the Award
Standard time 1hour 4
Actual time before use of the jig = hr. or 80 minutes
Efficiency 75% 3
Saving in time by use of the jig = 3 minutes in standard time.
Therefore, standard time after use of the jig = 1 hour - 3 minutes = 57 minutes
Revised Standard time
Actual time per piece after use of the jig =
Efficiency
57 minutes
76 minutes
75%
Saving in actual time per piece = 80 minutes - 76 minutes = 4 minutes
Time saved per year on 12,000 pieces @ 4 minutes = 48,000 minutes or 800 hours.
Saving in labour cost per annum = 800 hrs. Average rate per hour.
6
800 Rs.600
8
Rs.6
Average rate per day of 8 hours = Rs. 6, Average rate per hour 8
Amount of the award = 6 months' saving in labour cost
6
Rs.600 Rs.300
12
(b) Calculation of the Estimated Saving to the Company per Year
Rs.
Saving in labour cost per year 600
Saving in overhead : Saving in time x Overhead rate per hour
800 hrs. Rs.. 6 4,800
5,400
Less : Cost of the jig Rs. 600
Award to the workman Rs. 300 900
Net saving per year to the company , 4,500
(c) Calculation of the Revised Estimated Cost of the Component
Rs.
Material cost: 10 kgs. @ Re. 0.75 per kg. 7.50
Stock handling expenses @ 2% of material cost
2
Rs.7.50 0.15
100
per hour
6 76 6
Labour cost for 76 minutes @ Rs. Rs. 0.95
8 100 8
76
Overhead : hrs. Rs.6 7.60
60
Rs.300
Award for suggestion : 0.025
12, 000 pieces
600
Cost of the jig : 0.050
12, 000 pieces
Cost per component 16.275
82
Accounting for
Labour
(iv) Bcdeaux Point Premium Plan. Under this plan, every operation or job is expressed
in terms of so many standard minutes, which are called "Bedeaux points" or Each B
representing one minute through time and motion study. Upto 100% performance i.e.,
upto standard B's, a worker is paid time wages without any premium for efficiency. If the
actual performance exceeds the standard performance in terms of B's, then 75% of the
wages of the time saved is paid to the worker as bonus and 25% is earned by the foremen.
For example, standard time required for a job is 20 hours i.e., 1200 B's in terms of
minutes (20 x 60) whereas a worker has taken 16 hours i.e., 960 B's (16 x 60) instead of
1,200 B's. The worker has saved 240 B's or 4 hours (4 x 60). Suppose time wage rate is
Rs. 2 per hour ; the time saved will be equal to Rs. 8 (4 hours @ Rs. 2). The worker will
get 75% of Rs. 8 as bonus. So, his total earnings will be as follows :
Rs.
Time wages for 16 hours - actual time taken @ Rs. 2 per hour 32
Bonus—75% of 4 hours wages
75
4 R.2 6
100
Total earnings 38
This method ensures time wages to the workers and has the good feature of distributing
the wages of time saved among workers and foremen. It serves as a strong incentive for
workers for improving their performance above 100% of the standard. But workers
criticise this method as foremen are given a share of wages of the time saved. It is a
complicated method as the determination of standard time in terms of B's is not easily
understood by the workers.
Illustration 8. The cost accountant of Y Ltd has computed labour turnover rates for the
quarter ended 31 March 2012 as 10%, 5% and 3% under Flux method, Replacement
method and Separation method, respectively. If the number of workers replaced during
that quarter is 30, find out the number of (1) workers recruited and joined, and (2)
workers left and discharged
Solution
No. of replacements
Labour turnover rate =
Average No. of workers
5 30
100 Average No. of workers
100
Average No. of workers = 30 = 600.
5
(i) Calculation of workers recruited and joined
No. of separations + No. joined
Labour turnover rate (Flux method) =
Average No. of workers
10 18 No. joined
100 600
6, 000
No. joined = 18 42
100
Thus number of workers recruited and joined = 42
83
Cost and Management
Accounting
Illustrations 9. The worker is paid 50 paise per hour and the 5 days working week
contains 42 hours. The daily allowance for approved absence from his place of work,
maintenance of machine, etc., is 12 minutes and his job cards show that his time
chargeable during the week to various cost centres is as follows :
Job No. 305 20 hrs.
,, 310 10 hrs.
„ 320 8 hrs.
Time unaccounted for is caused by a power failure. Show how his wages for the week
would be dealt with in the cost accounts.
(B.Com. Hons. Delhi)
Solution
Total wages payable to the worker for the week = Rs. 21 (42 hours @ 50 P.)
Worker's wages are to be dealt with in the cost accounts as follows : Rs.
Wages chargeable to Job No. 305 (20 hours @ 50 paise) 10.00
Wages debited to Job No. 310 (10 hours @ 50 paise) 5.00
Wages debited to Job No. 320 (8 hours @ 50 paise) 4.00
Wages for approved absence for 5 days @ 12 minutes per day taken as normal
idle time to be recovered as factory overhead (1 hour wages @ 50 paise) 0.50
Wages for time wasted due to power failure taken as abnormal loss transferred
to Costing Profit and Loss Account (3 hours @ 50 paise) 1.50
21.00
Illustration 10. X Ltd. provides you the following information for its factory working 6
days in a week:
Hours as per Time Card 48 hours
Hours booked to jobs as per Job Card 36 hours
Daily allowance for lunch, personal needs,
fatigue and maintenance of machine 40 minutes
Wage Rate Rs. 20 per hour
Time not booked was due to breakdown of machinery, power failure, waiting for jobs
materials and tools. [B.Com. (Pass), Delhi 2008]
84
Accounting for
Labour
Solution :
4.8 QUESTION
1. What are the advantages of time keeping and time booking system?
2. What is idle time? How would you treat normal and abnormal idle time wages in cost
account?
3. What do you mean by Labour turnover? What measures do you suggest to keep the
rate of labour turnover at low level?
4. Explain different methods for calculating labour turnover?
5. What is idle time? Give reasons for idle time. How do you treat idle time in cost
account.
SUGGESTED REFERENCE
Mittal & Maheswari, Elements of Cost Accounting, Shree Mahavir Book Depot
(Publishers) 2015.
Varshney J.C., Principles and Practice of Cost Accounting, Wisdom Publication
House (2009).
Mittal & Maheswari, Management Accounting, Mahavir Publication, 2015.
Arora M.N. Cost Accounting, Vikas Publishing House Pvt. Ltd. 2013.
Jain & Narang, Cost Accounting, Kalyani Publishers 1998.
Arora M.N., Management Accounting, Himalaya Publishing House, 2006.
85
UNIT-5 ACCOUNTING FOR OVERHEADS
LEARNING OBJECTIVES
STRUCTURE
5.1 Introduction
5.2 Meaning and definition of overhead
5.3 Classification of overhead costs
5.4 Segregation of Semi variable cost
5.5 Overhead distribution
5.6 Basis of Apportionment
5.7 Re-apportionment of Service
5.8 Method of Apportionment
5.9 Questions
5.1 INTRODUCTION
(i) Indirect Material: All material which is used for proposes anciliary to the
business and which cannot be conveniently assigned to specific physical units is
known as Indirect material for example consumable stores, oil and wastage etc.
86
Accounting for
Overheads
(ii) Indirect labour: Labour employed for the purpose of carrying out tasks
incidental to goods or services provided is known as indirect labour. Such labour
does not directly participate in producing the product. For example, director fees,
salaries of sales man etc.
(iii) Indirect expenses: There are expenses which cannot be directly, conveniently
and wholly allocated to cost control and cost units. It means, cost which cannot
be traced to the finished products manufactured are termed as indirect expenses.
From the above discussion, it is clear that all kind of indirect expenses may be termed as
overhead. This implies overheads can only be apportioned to the finished products.
There are three elements of cost-material labour and expenses. These elements can
further be divided into direct and indirect. Direct materials, direct labour and direct
expenses constitute direct or prime cost while indirect materials, indirect labour and
indirect expenses constitute indirect cost or overhead.
87
Cost and Management
Accounting
(i) Element Classification: Under it, the classification is done according to nature
and source of the expenditure. The expenses can be classified into three main
categories:
(a) Indirect material: Material costs which cannot be allocated but are
apportioned to cost units are known as indirect material. For example fuel
lubricants etc.
(b) Indirect wages : These wages cannot be allocated but are observed by cost
units are known as indirect wages, e.g. wages of sweeper, idle time wages
etc.
(c) Indirect expenses : Expenses which cannot be allocated but to be absorbed
by cost units are known as indirect expenses, such as depreciation, taxes, rent
and rates etc.
(a) Production overhead: All indirect expenses incurred in connection with the
production are known as productions overhead. It is an aggregate of factory
indirect material, indirect wages and indirect expanses.
(b) Administration overhead: These expenses are of general nature and consist of
all costs incurred in the administration of an undertaking which are not related
directly to production or selling. They are known as office overhead.
(c) Selling and distribution overhead: Selling overhead is the expenses of seeking
for securing orders such as advertising, showroom expenses. Travelling
expenses, etc. Distribution expenses include the expenses incurred from the time
the product is completed in the factory till it reaches to the customers.
Distribution overhead is the cost of the process which begins with making the
packed product available for dispatched and ends with making the reconditioned
returned empty package available for re-use. It comprises all expenditure
incurred from the time the product completed in the factory until it reaches in the
hands of customers. It include cost of warehousing charges, cost of packing,
transportation etc.
3. Behaviour wise classification: Overheads cost can be classified into (i) fixed (ii)
variable and (iii) semi variable cost.
(i) Fixed overheads: Fixed methods may be defined as the overheads, which do not vary
with the volume of production within the installed capacity of plant. Fixed overhead
means the overhead which remains constant for all volumes of production within certain
limits. The examples of fixed overheads are rent of factory, salary of factory manager etc.
(ii) Variable Overheads: Variable overheads; maybe defined as the, overheads which
vary in direct proportion to the volume of production. There is positive correlation
88
Accounting for
Overheads
between the variable overhead and quantity of output. The examples of variable
overheads are power, repairs etc.
The main aim of classifying overhead costs is divided into fixed and variable, as it will
help the management in making decision and in controlling the expenditure. The semi
variable costs are to be split up into fixed and variable expenditure. Methods of
Segregation: The various methods of segregation of semi-variable expenses are as under :
(i) Average Method, (ii) Scatter Diagram Method, (iii) High and low point Method and
(iv) Simultaneous Equation Method.
(1) Average Method : Under this method, the given data are divided into two parts and
then average of output and cost is separately computed for these two parts. Variable
expenses is calculated by the following method :
Difference in Average Costs
Variable expenses per unit =
difference in Average output
Illustration 1 : From the following information segregate the semi-variable cost into
fixed and variable expenses :
Month Output Semi-variable Costs
January 160 4400
February 80 3200
March 240 5600
April 320 6800
May 400 8000
June 280 6200
Solution.
I Average II Average
Semi-
Semi-variable
Units Units variable
expenses
expenses
Jan. Rs. 4400 April 320 6800
Feb. 3200 May 400 8000
89
Cost and Management
Accounting
Under this method, the semi-variable costs are plotted on a graph. On its x-axis the
volume of production and on y-axis, the amount of expenditure is shown. After plotting
the points on graph, a straight line is drawn which represents an average of all the points.
If is known as line interacts the y-axis, marks the fixed cost. The difference between
semi-variable cost line and fixed cost line represents-variable cost.
The following graph is given with the help of data given in illustration No. 1.
Under this method, the highest and lowest output difference and the difference between
the corresponding costs are worked out. Then the variable cost per unit is calculated by
applying the following formula:
Difference in Semi - variable cost
Variable cost per unit =
Difference in output (units)
90
Accounting for
Overheads
Illustration 2 : From the following data, segregate the semi-variable cost into fixed cost
and variable cost:
Month Output (in units) Semi-variable cost
in (Rs.)
July 240 6600
August 120 4800
September 360 8400
October 480 10200
November 600 12000
December 420 9300
Solution.
(units) Highest Production 600 units in November
Lowest Production 120 units in August.
Difference = 480 units. (600 120)
Rs.
(Semi-variable cost) November = 12000
August = 4800
Difference = 7200
7200
Variable cost per unit = = Rs. 15 Per unit
480
Variable expenses in August = 120 units x 15 = Rs. 1800
Fixed expenses in August = Semi-variable - variables.
August = 4800 1800 = Rs. 3000.
Thus, Fixed cost in Aug.= Rs. 3000
Variable in Aug. = 1800
Total Semi-variable in Aug. = Rs. 4800
In this method, overhead costs are segregated by means of an equation which is as under:
y = mx + c
Where y = Total Semi-variable cost
m = Scope of variable cost line
x = Volume of output
c = Fixed cost
Illustration 3 : Take illustration No 1 and find out fixed and variable expenses with the
help of Equation :
Solution. Let us take January and February and make two equations.
y = rnx + c
for January : 4400 = 160 m + c — (i)
91
Cost and Management
Accounting
The overhead are classified into various categories. Production overheads should be
collected under standing order numbers. The main sources of collection of overheads are
as under :
(i) Stores requisitions-for indirect materials.
(ii) Journal entries-for adjustment entries and record.
(iii) Invoice— for collection of indirect expenses.
(iv) Wages Analysis Sheet-for collection of indirect wages.
A Production department is one that is engaged in the actual manufacture of the product.
A service department, is one which is rendering a service to the production departments.
Allocation : Certain items of overhead costs can be directly identified with a particular
department. Allotment of such costs to department is known as allocation. Thus
allocation is charging to a cost centre those overheads that results solely for that cost
centre. Allocation can be made only when exact amount of expenses is defintely known.
Indirect materials can be easily allocated to various departments.
92
Accounting for
Overheads
overhead. When any item is common to various cost centres, it is allocated to different
cost centres proportionately on such grounds.
Distinction between the two : Cost allocation means the allotment whole items of cost
to cost centre or cost units. Cost apportionment means and involves allotment or
proportion of items of cost centres or cost units. In other words, cost allocation deals with
whole items of cost. Further in allocation, costs are directly allocated- But apportionment
of cost needs a suitable basis for the sub-division of the costs to various cost units or cost
centres. Allocation is a direct process. But apportionment may be made indirectly on
suitable bases.
Allocation is a direct process and apportionment is an indirect Process for which suitable
bases are to be selected. Any item of cost can be allocated or apportioned does not
depend upon the nature of cost but upon its relation with the cost centre.
93
Cost and Management
Accounting
Once the overheads have been apporioned to production and service departments, the
next step is to reapportion the service department costs to production Departments. The
cost of service Departments should be charged to production departments. It is called as
secondary distribution.
method does not take into account inter-departmental transfer of service i.e. service,
provided by one service department to other service departments.
Non-reciprocal basis. Under this method, the various service departments are first
arranged according to their serviceability. Serviceability may be either on the basis of
expenditure incurred in a service department or the number of departments served. The
most serviceable department is taken up first and its cost is distributed to other production
and service departments on a suitable basis. Its account is then closed. Then, the next
most serviceable department is taken up and its cost is apportioned to production and
service departments. This process goes on till the cost of last service department is
apportioned only to the production departments. This method is also called 'step method'.
If partly considers the service rendered by one service department to other service
departments.
Reciprocal basis. Inter-departmental services are taken into account on reciprocal basis.
The three methods which may be used for reciprocal distribution are:
(i) Simultaneous equation method
(ii) Repeated distribution method
(iii) Trial and error method.
(i) Simultaneous equation method. In this method, the true total overhead cost of
each service department is ascertained with the help of an algebric equation.
These are then redistributed to production departments on the basis of given
percentages.
(ii) Repeated distribution method. Under this method, the total overhead costs of
the service departments are distributed to other service and production
departments according to the given percentage till the expense of all service
departments are exhausted or become insignificant.
(iii) Trial and error method. Under this method, the cost of a service department is
apportioned to another service department. The cost of another service centre
plus apportioned cost from the first centre is apportioned back to the first service
centre. This process is repeated till the amount to be apportioned becomes
negligible.
95
Cost and Management
Accounting
Illustration 1: The following information was available from the books of ABC Ltd. for
half year ending on 30th September 15. Prepare a overhead Departmental Distribution
Summary.
Particular Production Department Service Department
A B C X Y
Materials Rs. 6,000 5,000 4,000 3,000 2,000
Wages Rs. 14,000 12,000 10,000 2,000- 2,000
Electricity Kwt. 16,000 12,000 12,000 4,000 6,000
Assets values (Rs.) 100,000 6,000 40,000 20,000 20,000
Area occupied 1,600 1,200 1,200 400 400
(Sq. metre)
Light Points (No.) 20 30 30 10 10
Employees No. 800 600 600 200 200
Solution.
Overhead Distribution Summary
Items Basis of Total Production Deptt. Service Deptt.
Appor- Rs. A B C X Y
tionment Rs. Rs. Rs. Rs. Rs.
Materials 4,000 — — — 2,000 2,000
Wages 5,000 — — — 3,000 2,000
Stores- Materials 800 240 200 160 120 80
overheads
Motive- KWT. 3,000 960 720 720 240 360
Power
Lighting Points Nos. 400 80 120 120 40 40
Labour- No. of- 6,000 2,000 1,500 1,500 500 500
Welfare employees
Depreciation Asset Value 12,000 5,000 3,000 2,000 1,000 1,000
Repairs Asset Value 2,400 1,000 600 400 200 200
General- Wages 2,000 7,000 6,000 5,000 1,000 1,000
overheads
Rent Area 1,200 400 300 300 100 100
Total 54,800 16,680 12,440 10,200 8,200 7,280
Deptt. X 4:3:3 3,280 2,460 2,460 - —
8,200
Deptt. Y 7:6:5 2,832 2,426 2,022 — (-) 7,280
(Wages)
Total 54,800 22,720 17,326 14,682 — —
96
Accounting for
Overheads
Illustration 2. From the following information, work out the production hour rate of
recovery of overheads in departments X, Y and Z.
Production departments Service
departments
Particulars Total X Y Z A B
Rs. Rs. Rs. Rs. Rs. Rs.
Rent 1,000 200 400 150 150 100
Electricity 200 50 80 30 20 20
Fire insurance 400 80 160 60 60 40
Plant depreciation 4,000 1,000 1,500 1,000 300 200
Transport 400 50 50 50 100 150
Estimated — 1,000 2,500 1,800 — —
working hours
Solution
Simultaneous equation method
Let x = Total expenses of department A to be apportioned
y = Total expenses of department B to be apportioned
x = 630 + 0.2y
y = 510 + 0.1x
or .
x 0.2y = 630 (i)
y 0.1x = 510 (ii)
Multiplying (i) by 5 and adding the two
5x y 3150
0.1x y 510
4.9x 3, 660
3, 660
x 747 (approx.)
4.9
Substituting this value in equation (i)
747 2y = 630
- 2y = 630 747
y = 585
97
Cost and Management
Accounting
Service
Total
Production departments departments
Particulars Amount
X Y Z A B
(Rs.)
Rs. Rs. Rs. Rs. Rs.
1. Rent 1,000 200 400 150 150 100
2. Electricity 200 50 80 30 20 20
3. Fire insurance 400 80 160 60 60 40
4. Plant depreciation 4,000 1,000 1,500 1,000 300 200
5. Transport 400 50 50 50 100 150
Total departmental
expenses 6,000 1,380 2,190 1,290 630 510
98
Accounting for
Overheads
Illustration 3. The New Enterprises Ltd. has production departments A, B and C and two
service departments D and E. The following figures are extracted from the records of the
company.
Rates and rates Rs. 5,000
General lighting Rs. 600
Indirect wages Rs. 1,500
Power Rs. 1,500
Depreciation of machinery Rs. 10,000
Sundries Rs. 10,000
The following further details are available:
Total A B C D E
Floor space (sq. ft.) 10,000 2,000 2,500 3,0'00 2,000 500
Light points 60 10 15 20 10 5
Direct wages (Rs.) 10,000 3,000 2,000 3,000 1,500 500
H.P. of machines 150 60 30 50 10 —
Value of machinery 2,50,000 60,000 80,000 1,00,000 5,000 5,000
(Rs.)
Working hours — 6,226 4,028 4,066 —
99
Cost and Management
Accounting
Solution
(i) Overheads Primary Distribution Summary
Items Basis of charge Total Production dept. Service dept.
Rs. A B C D E
Rs. Rs. Rs. Rs. Rs.
Direct wages Allocation 2,000 — — — 1,500 500
Rent and rates Re. 0.50 per
sq. ft. 5,000 1,000 1,250 1,500 1,000 250
General Re. 10 per
lighting point 600 100 150 200 100 50
Indirect 15% of direct
wages wages 1,500 450 300 450 225 75
Power Rs. 10 per H.P. 1,500 600 300 500 100 —
Depreciation 4% of the
of machinery value of ;
machinery 10,000 2,400 3,200 4,000 200 200
Sundries 100% of
direct wages 10,000 3,000 2,000 3,000 1,500 500
Total
departmental
overheads 30,600 7,550 7,200 9,650 4,625 1,575
100
Accounting for
Overheads
Required:
1. A statement showing distribution of overheads to various department
2. A statement showing re-distribution of service department's expenses to
production departments.
3. Machine hour rates of the production departments A, B and C.
101
Cost and Management
Accounting
Basis Total A B C X Y
Rs. Rs. Rs. Rs. Rs. Rs.
Direct material Direct — — — 2,000 1,000
Direct wages Direct — — — — 1,000. 2,000
Factory rent Area 4,000 1,000 500 1,000 500 1,000
Power H.PX
M/c hr. 2,500 500 800 800 150 250
Depreciation Cap. value 1,000 200 400 200 100 100
Other overheads M/c hr. 9,000 1,000 2,000 4,000 1,000 1,000
2,700 3,700 6,000 4,750 5,350
102
Accounting for
Overheads
The expenses of the boiler-house and pump-room are appointed to the production
departments on the following basis:
A B C B.H. P.R.
Expenses of boiler-house 20% 40% 30% 10%
Expenses of pump-room 40% 20% 20% 20%
Show clearly as to how the expenses of boiler-house and pump-room would be appointed
to A, B and C departments. Use algebraically equation.
103
Cost and Management
Accounting
Solution.
Statement of overhead Distribution
Items Production Deptt. Service Deptt.
A B C P Q
Rs. Rs. Rs. Rs. Rs.
Total Expenses 6,50,000 6,00,000 5,00,000 1,20,000 1,00,000
Servicer Deptt. P 36,000 48,000 18,000 (-) 1,20,000 18,000
Servicer Deptt. Q 47,200 35,400 29,500 5,900 (-) 1,18,000
Servicer Deptt. P 1,770 2,360 885 (-) 5,900 885
Servicer Deptt. Q 354 266 221 44 -885
Servicer Deptt. P 13 18 7 (-44) 6
Servicer Deptt. Q 3 2 1 — (-)6
Illustration 7: From the data given below. Calculate the Machine Hour Rate:
Rs.
(i) Rent of the Deptt. (Space occupied 780
by machine of 1/5 deptt.)
(ii) Lighting (No. of men in the Deptt. 12 two 288
men are engaged on the machine)
(iii) Insurance 36
(iv) Cotton, waste oil etc. 60
(v) Salary of foreman (% time is occupied by this machine) 6000
The cost of the machine is Rs. 9,200 and it has an scrap value of Rs. 200.
(1) The machine will work for 1800 hours per annum.
(2) Expenses on repairs Rs. 1125.
(3) Power consumption 8 units per hour at cost of Re 1 Per unit.
(4) Working life of machinery 10 years.[Delhi B. Com.]
Solution.
Computation of Machine Hour Rate
Standing Charges : Rs. Rs.
- Rent (780 x % ) 156
- Lighting (780 x 1/5) 48
- Foreman's Salary (780 x 1/5 ) 1,500
- Insurance 36
- Cotton Waste 60
Total 1,800
Hourly Rate : 1,800/1800
Variable Expenses : '
1125 : 9200 200
- Depreciation 0.50
1800 100
- Repairs 10 1800 0.60
- Power 8 units @ Re 1/- 8.00
Machine Hour Rate 9.56
104
Accounting for
Overheads
5.9 QUESTIONS
SUGGESTED REFERENCE
Mittal & Maheswari, Elements of Cost Accounting, Shree Mahavir Book Depot
(Publishers) 2015.
Varshney J.C., Principles and Practice of Cost Accounting, Wisdom Publication
House (2009).
Mittal & Maheswari, Management Accounting, Mahavir Publication, 2015.
Arora M.N. Cost Accounting, Vikas Publishing House Pvt. Ltd. 2013.
Jain & Narang, Cost Accounting, Kalyani Publishers 1998.
Arora M.N., Management Accounting, Himalaya Publishing House, 2006.
105
UNIT-6 ABSORPTION OF OVERHEADS
LEARNING OBJECTIVES
STRUCTURE
6.1 Introduction
6.2 Under and over absorption of overheads
6.3 Methods of absorption of production overheads
6.4 Treatment of special items of overheads
6.5 Overhead absorption Rate
6.6 Methods of absorption of factory overheads
6.7 Machine hour Rate overheads
6.8 Questions
6.1 INTRODUCTION
Overheads may be charged to production on the basis of actual rates based on actual
output and actual overheads or on the basis of pre-determined rate. In practice, overheads
are charged to production at pre-determined rates. Since pre-determined rates are based
on estimates, actual overheads are found to be different from absorbed overheads. This
arises a difference between overhead absorbed and overhead incurred. Such a difference
is known as under-absorption or over- absorption of overheads.
Under-Absorption: When actual overheads incurred are more than the overheads
absorbed, it is known as under-absorption. Thus,
106
Absorption of
Overheads
Over-Absorption : When actual overheads incurred are less than the ' overheads
absorbed, it is known as over-absorption. Thus,
Over-Absorption = Absorbed Overheads - Actual Overheads.
Under-absorption has the effect of understating the cost while over-absorp-tion has the
effect of overstating the cost. Over-absorption or under-Absorption of overheads may be
disposed of in any one of the following ways: (i) Transfer to Costing Profit and Loss
Account, (ii) Carry forward to the next year, (iii) Use of supplementary rate.
(i) Transfer to Costing Profit and Loss Account: The amount of under-absorbed or
over-absorbed overheads is transferred (credited or debited) to Costing Profit and Loss
Account at the end of accounting period when
(i) The amount of under-absorbed or over-absorbed overheads is minor and insignificant.
Or
(ii) The under-absorption or over-absorption has caused by abnormal circumstances i.e.
the factors beyond the control of management,
(ii) Carry Forward to the Next year : The amount of under-absorbed or over-absorbed
overheads is carried over to subsequent years when management is sure that an over-
absorption in the current period will be more or loss neutralised by under-absorption in
the next period. Accordingly, the amount of the current period is transferred to a Reserve
Account or Suspense Account. This method has a limited application because it is against
the costing principle that overhead should be absorbed in a particular year in which it is
incurred. However, this method may be followed when:
(i) The industry is seasonal with fluctuating demand and fluctuation in production.
(ii) Normal business cycle extends beyond one year,
(iii) The project is new and output initially low but there will be more output in the next
year which will absorb more overheads,
107
Cost and Management
Accounting
(iii) Use of Supplementary Rates : In case there is significant over or under absorption
due to anticipation about overheads cost or the basis not coming true, supplementary rate
may be used. It is calculated by dividing the amount of over-absorption or under-
absorption by the actual base. Thus.
Under-Absorption or Over-Absorption
Supplementary Overhead Rate =
Actual Base
Supplementary rates are of two types :
(i) Positive supplementary rates and
(ii) Negative supplementary rates. ,
The amount of under-absorbed overheads is, added through a positive supplementary rate
and the amount of over-absorbed overheads is deducted through a negative
supplementary rate. Overhead absorption is the apportionment of overheads of the cost
centres over cost units. There are two steps in the absorption overheads. (A) Computation
of these rates. and (B) Application of these rates.
(A) Computation of overhead Rates : Absorption rates are computed with the aim
of absorption of overheads in costs of the cost units. There are six methods of
determining absorption rates. In all these methods, the overhead rate is computed by
dividing the total amount of overheads by the number of units.
Total overhead
Thus, overhead absorption Rate =
Total units
(B) Application of Rates : The overhead rate is multiplied to the number of units in
the cost unit. Thus overhead absorbed = No. of units x overhead Rate
(1) Direct Material: It is computed by dividing the total overheads by the total cost
of direct materials Consumed in a particular department.
Production overhead
Formula : 100
Direct Materials
(2) Direct Labour cost Percentage Rate : It is computed by dividing the production
overhead by the direct labour cost.
108
Absorption of
Overheads
Production overhead
Formula : 100
Direct Labour Cost
(3) Prime Cost Percentage Rate: Overhead rate is calculated by dividing the
production overhead by Prime Cost.
Production overhead
Formula : 100
Prime Cost
(4) Direct Labour Hour Rate : It is calculated by dividing the total production
overhead by the total number of labour hours.
Production overhead
Formula :
Direct Labour Hours
(6) Output Rate Per Unit: It is determined by dividing the total overheads of
department by the number of units Produced.
Production overhead
Formula :
No. of units
Administration Overhead
All indirect expenses incurred in formulating the Policy, directing the organisation and
controlling the operations of an undertaking is known as administration overhead. They
have no direct connection with the production or sales. These are affected by any
fluctuations in the volume of production or sales.
For the absorption of these overheads, a single overhead rate is computed by any one of
the following methods :
109
Cost and Management
Accounting
(3) Percentage of Conversion Cost : It is the cost of converting raw material into
finished product.
Administrative overhead
Formula : 100
Total Conversion Cost
Selling Cost is the cost of seeking to create and stimulate demand and of securing orders.
Such as advertisement Show room expenses etc. Distribution cost is the cost of the
sequence of operations which begins with making the packed product available for
despatch.
Method of Absorption:
(1) Rate Per unit of Sales : This overhead are divided by the number of units sold.
(2) Percentage of Selling price :
(1) Depreciation : Depreciation is the diminution in the value of a fixed asset due to
lose of time. In cost accounts, depreciation is charged to cost of production on the
following grounds :
(a) Depreciation represents a charge for usage of the capital resources.
(b) The invested amount has to be recovered over a number of years.
110
Absorption of
Overheads
(2) Rent: Rent is an expenses to be taken into account as overhead. If any premises
is owned by the proprietor, and no rent is paid, in such a case, a charge in lieu of rent
should be made in cost accounts, so that the comparison of total cost may be made easily.
(4) Royalties and Patent fees : Royalties payable is a direct expense and included in
prime cost. If Royalties are payable on the basis of sales, then it is a selling cost.
(6) Director's fees and Salaries : Director's fees and salaries are to be apportioned
to production, administration and selling department on the basis of time devoted by the
directors.
(8) Market Research Cost: The expenses on market research is a matter of policy
costs and it is treated as selling overhead.
(9) After sales Service : This cost is a part of the selling overhead. Each case is
analysed and expenditure major repairs are treated as deferred charge and written
in Profit and loss account.
(10) Fringe Benefits: All the expenses of fringe benefits are apportioned on the basis
of quantum of benefits received by the employees,
(11) Cost of small Tools : It may be treated in any one of the following methods :
(a) They may be charged as an expense at the time of their purchase.
(b) If it is capitalised, it is debited to Small Tools A/c.
(c) It may be charged to Stores inventory.
(14) Drawing and Design office Costs: It may be treated as Direct Cost. If treated as
selling overheads. If it is related to service department, its costs to be apportioned
to production departments on the basis of technical estimates of service rendered.
(16) Set-up Costs : The cost of set-up of time is charged to that particular Job for
which preparation is being made. It is frequent, it may be apporioned to all Jobs
equitably.
(17) Bonus to employees : Normal bonus is a part of wages and any bonus paid over
and above the minimum amount should be charged to Profit and Loss account.
Bonus may be treated as overhead and apportioned to various departments on the
basis of wages bill.
(i) Interest on Capital. There are certain items of income, expenses and apportion
which are included in the financial accounts but do not find any place in the Cost
Accounts e.g., provision for taxation, debenture interest, income on investments,
fictitious assets written off, dividends paid etc. Similarly, some items belong to
Cost Accounts only. These items are mostly of notional character and are treated
by way of opportunity costs for purposes of managerial decision making. Interest
on Capital employed in the business is one such item. It may be included in cost
accounts through no payment for this Interest may have been made in actual.
(ii) Leave wages. There are always certain payments made to workers which are not
directly to production. These payments are : Leave Pay, HolidayPay, Stock pay,
Maternity Period Pay, Pension Scheme Payments, Employer's contribution
to the Provident Fund, Retirement cum Death Gratuity Payment, Medical
benefits etc. These are expenses besides wages and salaries, which the
management incurs by way of supplementary labour costs and these benefits are
enjoyed by the industrial labour. These costs are of such a nature that these can
not be allocated to cost units directly but may be allocated to the particular
departments or cost centres in which the workers are employed. Thus, it can be
said that the cost of fringe benefits are items of departmental overheads. In many
112
Absorption of
Overheads
cases, the cost of these benefits to labour is not incurred uniformly in each
accounting year. Therefore, benefits for the whole year are anticipated and a
proportionately uniform amount is charged to each accounting period. This helps
to avoid uneven charge of these costs over different accounting periods. So far as
pension benefits to workers are concerned, a reserve is usually created to meet
future payments. The amount of provision created is treated as an overhead.
Pension costs are usually allocated direct to a cost centre or apportionment may
be made on the basis of the number of employees or total wages paid.
(iii) Audit fees. It is just like any other normal item of expense and it is included as
an expense in the Cost Accounts as well as in the Financial Accounts In both the
accounts, it may be included as an expense on accrual basis even in it has not
been paid. This expense is part of the administration overheads and all these
office or general administration overheads are absorbed or charged to the product
cost at some or the other pre-determined rate.
(iv) Holiday with pay. Every worker is entitled to certain number of holiday during
the year for which he is paid. Holiday with pay is estimated in advantage for the
full year and is included in the cost. For direct workers, holiday will pay may be
treated as a direct cost by inflating the wage-rate. Alternatively, we may be
included in production overhead. For all indirect workers, holiday will pay is
treated as an overhead.
(v) Casual wages. Casual wages mean wages payable to casual workers. Casual
workers are those workers who are employed on daily basis and are not on the
regular payroll of the employer. Casual workers are mostly untrained and indirect
workers. Their wages therefore become a part of production overheads
(vi) Bad debts. There are different opinions on treatment of bad debts in Cost
Accounts. Some accountants exclude it as it is a financial loss. But some expert
say that bad debts should be included in Cost Accounts as selling overheads.
Only normal amount of bad debts should be included in Cost Accounts.
Abnormal amount of bad debts should not become a part of the cost but should
be transferred to Profit & Loss Account.
(vii) Expenses on staff welfare activities. These are also called 'Non-monetary'
benefits extended to the staff in the form of amenities or facilities These do not
offer, direct cash reward to the employees. However, these benefit or facilities
certainly go a long way in making the work-conditions quite lucrative as these
create a huge positive psychological effect upon the employees.
Expenses incurred by the employers on all such benefits or facilities are treated as part of
the factory overheads. These are apportioned over the different cost centres i.e., the
departments of the factory on the basis of number of employees employed by each
department. In this way, these costs are absorbed alongwith other items of the factory
overheads.
(viii) Wages paid for re-operation of defectives. Defective goods are that part of the
production which can be rectified with reasonable costs. Such rectification is not possible
in case of the 'Spoilage'. Spoilage is that part of the production which, like defectives,
does not come up to the standards of the normal output and it is as such rejected. Such
spoiled or damaged goods can not be brought back to the normal condition even after
repairs. Both of these i.e., spoilage and defectives can be controlled by classifying them
first into normal and abnormal. Reports on 'spoilage' and 'defectives' can go a long way in
this direction. effective steps should be taken to reduce the quantity of the abnormal
spoilage and defectives.
Cost of rectification of the defectives can also be classified into normal and abnormal.
Normal cost is treated as part of the factory overheads whereas abnormal costs, if any, are
charged to 'Costing Profit & Loss Account.'
'Research cost' includes expenses incurred on searching for new or improved products
or new or improved materials or methods. This expenditure usually comprises of the
wages and salaries of the research staff, materials and facilities provided in the laboratory
and research department, or payments to outside organisations. 'Development cost'
means expenses which are incurred on implementing the results of the research
programmes undertaken by the organisation. Usually, all these expenses incurred on
research and development activities are of the nature of pre-production costs as there is
always a considerable time lag between the incurring of such expenses and actual
realisation of their benefits. As there is no immediate production, therefore it may not be
possible to charge these costs as part of the factory overheads. Such expenses may be
114
Absorption of
Overheads
capitalised and then written off in future against actual production. However, if research
and development costs are of recurring or continuous nature, then the same may be treated
as part of the 'Factory overheads'. Similarly, if research and development costs are related
to any specific job then the same may be charged exclusively to that particular job.
Overhead absorption rates may be of two types (i) Actual rate and pre determined rate.
Actual rates are those which are based on actual cost while pre-determined rates are those
which are based on estimated or pre-determined cost.
Pre-determined overhead absorption rates. The computation and use of such pre-
determined overhead absorption rates is more practical and useful because in case of such
rates, the rates relating to a particular period are available for costing purposes well in
advance. This helps in preparation of cost estimates and fixing prices for sales or
quotation purposes etc. However, there is one limitation as well of such pre-determined
rates and this is that such rates when actually applied for absorption of the factory or
other overheads, generally result in over or under absorption of the factory overheads.
Pre-determined overhead absorption rates are preferred over actual rates because of the
following factors:
• Pre-determined rates help in the preparation of tenders and quotations.
• These help in controlling the cost.
• These are of greater practical utility.
115
Cost and Management
Accounting
The following methods are commonly used for absorption of factory overheads:
1. Percentage of direct material cost
2. Percentage of direct labour cost
3. Percentage of prime cost
4. Direct labour hour rate t
5. Machine hour rate (M.H.R.) »"•
Sometimes, the factory overheads are charged on the basis of materials consumed by
products. The overhead rate is calculated by dividing estimated factory overheads for the
period by the cost of direct material.
Overhead expenses
Percentage of direct material cost = 100
Total direct material cost
This method is simple but has only limited use because in most cases no logical
relationship exists between the direct material cost of a product and factory overheads
incurred. Again one product might be made for costly materials while another requires
less expensive materials. However, both might require the same manufacturing process
and thus incur approximately the same amount of factory overheads. If the material cost
basis is used to charge overhead, the product using costly material will be charged with
more than its share of overheads.
(1) A percentage on direct wages. This is the oldest method and still appears to be the
most popular. The rate is calculated by dividing overheads cost by the amount of direct
labour. The overheads are applied by multiplying this percentage by the direct labour cost
of each job or cost unit.
116
Absorption of
Overheads
(III) Percentage of Prime cost: In this method, overhead rate is calculated by dividing
the factory overhead by prime cost.
Factory Overhead
Factory Overhead Rate = 100
Prime Cost
This is a simple method to apply, but as many overheads are related to time, and because
materials usually forms a large part of the total price costs it has no logical basis. It is
suitable only in case where one standard product made which uses a fixed quantity of
material at a constant price and time taken in production is constant. __
(IV) Direct Labour Hour Rate : Under this method, overheads is charged on the basis
of direct labour hours worked rather than direct wage paid. The overheads to be absorbed
are aggregated and rate cf overhead absorption is calculated by dividing the overhead
cost by the labour hours ex-pended.
Amount of Factory Overhead
Overhead Rate = 100
Total Direct Labour Hours
For example,
Labour = 4,000 hours
Factory Overheads = Rs. 10,000
10, 000
10,000 Factory Overhead Rate = = Rs. 2.50 per hour
4, 000
Thus, if 10 hours are spent on a job, factory overhead for that job will be Rs. 25.
This method gives full recognition to the time factor. It is easy to operate because labours
hours are readily available from the time sheet and job-cards etc. However, this method
does not take into consideration the factor of production other than labour and may lead
to faulty distribution of overhead to produce cost. It fails to take into consideration the
expenses which are not dependent on labour hours such as power, depreciation, fuel etc.
It is one of the most scientific methods of absorption of overheads. Machine hour rate
means the cost of operating a machine for one hour. It is on the basis of this rate that a
charge is made to the jobs for the overheads depending upon the number of hours for
which a machine has worked on that job machine has rate is computed by the following
formula.
Overheads pertaining to each machine cost centre
Machine hour rate (M.H.R.) =
Number of effective machine-hours
117
Cost and Management
Accounting
Effective machine hours = Normal working hours per year - loss of hours due to repairs
and maintenance and setting-up
etc.
Factory overheads
or Machine hours Rate =
Effective machine hours
1. While computing machine hour-rate, the first step is to determine effective machine
hours for the base period.
2. Overheads specific to machine are allocated to that machine. Such expenses may be
power, repairs, supervisor's salary etc. All other overheads are apportioned to each
machine on appropriate basis. For example, rent may be apportioned on the basis of
area occupied by the machines.
3. It is better to compute separate rate for fixed items and variable items, ience,
overhead costs are classified in following groups :
(a) Fixed or standing expenses.
(b) Variable or Machine Expenses.
Fixed or standing expenses are the expenses which are not affected by machine operation.
Such expenses include rent, rates, insurance, supervision etc. Variable or machine
expenses are the expenses which are affected by ma¬chine operation. Such expenses
include repairs, depreciation and power or fuel.
4. Sum total of standing charges or fixed charges is divided by total number of machine
hours. This will give us standing charges or fixed expenses per machine hour.
6. Standing charges per hour and machine expenses per hour are added to obtain
machine hour rate.
118
Absorption of
Overheads
Advantages :
Some suitable basis for apportionment of departmental expenses which relate to more
than one machine. The bases which may be adopted for apportioning the different
expenses for computation of machine hour rate are given below.
Advantages
1. It is a scientific, practical and accurate method of recovery of overheads where work
is done mainly by machines.
2. It .provides useful data for estimating cost of production, setting standards and for
fixing selling prices for quotation.
3. Under absorption, if any, will reveal the extent to which the machines have been idle.
4. It takes into consideration the time factor completely.
Disadvantages
1. Estimation of machine hours becomes difficult, particularly when the production /
programme is not available in advance.
2. The maintenance of detailed records for the running hours of the machines and time
Calculate machine hour rate if a machine runs for 1,200 hours in a year.
(B.Com. (Pass) Delhi)
Note: Interest paid on loan (being purely financial charge) has been ignored.
120
Absorption of
Overheads
Solution
Computation of Machine Hour Rated
121
Cost and Management
Accounting
Solution
Computation of Machine Hour Rate
(Machine hours: 50 x 40 = 2,000)
Particulars Per annum Per hour
Rs. Rs.
Standing or fixed charges
Rent for workshop (25,000 4) 6,250
160 12
General lighting for the workshop
480
4
Illustration 4: A manufacturing unit has added a new machine to its fleet of five existing
machines. The total cost of purchase and installation of the machine is Rs. 7,50,000. The
machine has an estimated life of 15 years and is expected to realise Rs. 30,000 as scrap at
the end of working life.
122
Absorption of
Overheads
You are required to compute the machine-hour rate for recovery of the running cost of
the machine. (I.C.W.A. Inter, June 1993)
Solution
Computation of Machine Hour Rate
Particulars Per annum Rs. Per hour Rs.
Standing charges
Operator's wages (450 x 3 x 50) 67,500
Add 40% fringe benefits 27,000
94,500
Departmental and general works overheads (60,000 +
12.5%) 67,500
123
Cost and Management
Accounting
Compute factory overhead rates of recovery and the amount of overhead chargeable to
the order by the following method:
(i) Direct material cost percentage;
(ii) Direct labour cost percentage;
(iii) Labour hour rate;
(iv) Machine hour rate.
124
Absorption of
Overheads
6.8 QUESTIONS
Calculate Machine Hour Rate if a machine runs for, 200 hours in a year.
[B. Com. (Pass) Delhi, 1988] [Ans. Rs. 2.27]
7. From the following information compute machine hour rate in respect of 'Skylark'
machine for the month of May 2005:
Cost of the machine Rs. 3,00,000
Expenses on installation and freight charges 25,000
Estimated scrap value at the end 25,000
Insurance charges allocated to the machine Rs. 480 p.m.
Effective working life of the machine 24,000 hours
Hours worked during the month 240 hours
Repairs during the effective life of the machine Rs. 24,000
Power consumed by the machine is 20 units per hour at a cost of 25 paise per unit.
Additional charges at 5 paise per unit are payable on the total units consumed if the same
exceeds the basic consumption of 2,000 units per mr.ith. [B. Com. (Pass) Delhi, 1986,
adapted]
The estimated life of the machine is 10 years and scrap value at the end of 10th year is
Rs. 1,00,000. The machine is expected to run 20,000 hours in its lifetime. The machine
occupies 25% of total area. The foreman devotes l/6th his time for the machine.
Calculate machine hour rate for the machine. [B.Com. (Pass) Delhi, 1996]
[Ans. Rs. 32.87]
10. Calculate the machine-hour-rate from the following details provided by a Production
Department A-B-C Ltd.:
(1) Bought of machineryRs. 4,50,000
(2) Installation charges Rs. 50,000
(3) Life of machine 5 years
(4) Working hourse per year 2,500
(5) Repairs charges 75% of depreciation
(6) Electric power consumed : 10 units per hour @ Rs. 3
(7) Lubricating oil Rs. 40 per day of 8 hours.
(8) Consumable stores @ Rs. 80 per day of 8 hours.
(9) Wages of machine operator @ Rs. 200 per day of 8 hours.
[B.Com. (Pass), Delhi 2012]
[Ans. Rs.140]
SUGGESTED REFERENCE
Mittal & Maheswari, Elements of Cost Accounting, Shree Mahavir Book Depot
(Publishers) 2015.
Varshney J.C., Principles and Practice of Cost Accounting, Wisdom Publication
House (2009).
Mittal & Maheswari, Management Accounting, Mahavir Publication, 2015.
Arora M.N. Cost Accounting, Vikas Publishing House Pvt. Ltd. 2013.
Jain & Narang, Cost Accounting, Kalyani Publishers 1998.
Arora M.N., Management Accounting, Himalaya Publishing House, 2006.
127
BLOCK-3
Overheads
The present block refers to the concepts of various costing like single, output, job,
contract and process. The learners with learn about the process and steps of
calculating the costs of a business organisation. Further, they will have the
opportunities to enquire the specific knowledge about the single, output and job
costing. The present block refers the following unit;
Unit 7: Single or Output Costing
128
UNIT-7 SINGLE OR OUTPUT COSTING
LEARNING OBJECTIVES
STRUCTURE
7.1 Introduction
7.2 Method of determining unit cost
7.3 specimen of cost sheet
7.4 Treatment of scrup
7.5 Items to be excluded from cost sheet
7.6 Production account
7.7 Question
7.1 INTRODUCTION
(i) Cement, (ii) Steel, (iii) Floor mills and (iv) Bricks-making etc
Objectives of unit or output costing: the important objectives of unit or output costing
are discussed here below.
The know the total cost of production
To classify cost under related categories such as prime cost, work cost,
administration cost etc
To determine the effect of each elements of cost.
To determine proposed selling price to earn desired profit.
To determine tender price on the basis of cost data and future prospects elements
of cost under unit or output costing: the main elements of cost under unit or
output costing are
(i) Material , (ii) Labor, (iii) Direct expenses and (iv) Overheads.
130
Single or Output
Costing
131
Cost and Management
Accounting
SALES TAX, EXCISE DUTY AND ROYALITY: Sales tax: sales tax is levied on sales.
The firm has to deposite this amount with sales tax authorities. It is deducted from sales
to arrive at net sales to be shown in cost sheet / statement of cost. Excise duty: excise
duty is levied on production. It is treated as a direct charge to production and is included
132
Single or Output
Costing
in prime cost. Royalty: it is payment for the right to manufacture a product or the use of a
process. If royalties are paid on production, it treated as a direct charge to production and
included in prime cost. If royalties are paid on the number of units sold, the amount is
charged to selling overheads.
While preparing cost sheet, some broad categories of expenses are not to be included as
they are purely financial items not forming part of cost of production. These are the
following:
(iv) Abnormal gains and losses: It includes the following: (i) Abnormal losses of
materials and (ii) Abnormal idle time of labour.
Illustration 1 . Find out the cost of raw materials purchased from the data given below:
Prime cost = 2,00,000
Closing cost of raw material = 20,000
Direct labor cost = 1,00,000
Expenses on purchases = 10,000
[B.Com. (Delhi)]
133
Cost and Management
Accounting
Solution
Prime cost – direct labor cost = Raw material consumed
Rs. 2,00,000 – Rs. 1,00,000 = Rs. 1,00,000
Raw material consumed = opening stock + purchases +
expenses on purchase- closing
stock
Rs,1,00,000 = Nil + purchase + 10,000-20,000
Rs. 1,00,000 – 10,000 +20,000 = purchase of material
Purchase of raw material = Rs. 1,10,000
(vi) Postage
(vii) Audit fees
(viii) Depreciation of office equipment
(ix) Legal charges
(x) Director travelling expenses
(xi) Office lighting
(xii) Stationary and printing
Illustration 2. From the following information calculate (1) prime cost (2) factory
cost (3) cost production (4) total cost (5) profit:-
Rs
Stock of materials – opening 3,76,000
Stock of materials – closing 4,00,000
Materials purchased 16,64,000
Direct wages paid 4,76,800
Indirect wages 32,000
Salaries to administrative staff 80,000
Freight-inward 64,000
Freight outward 40,000
Sales 31,59,600
Cash discount 28,000
Bad debts 37,600
Repairs of palnt 84,800
Rent factory 24,000
135
Cost and Management
Accounting
The managers time is shared between the factory and the office in the ratio of 20:80.
Solution:
COST SHEET
PARTICULAR Rs Rs Rs
Opening stock of materials 3,76,000
Add: purchases 16,64,000
Freight inwards 64,000
21,04,000
Less: closing stock of materials 4,00,000
(1) Value of materials 17,04,000
Add: direct wages 4,76,800
(2) Prime cost 21,80,800
Factory overheads-
Indirect wages 32,000
Repairs of plant 84,800
Factory rent 24,000
Depreciation of plant 57,800
Electricity 96,000
Fuel 1,28,000
Manager salary
(96,000 * 20/100) 19,200 4,41,800
Less: sale of scrap 1,000 4,40,800
(3) Factory cost 26,21,600
Add: administrative cost
Salary to staff 80,000
Office rent 12,800
Depreciation of furniture 4,800
Director’s fees 48,000
General charges 49,600
Managers salary 96000*80/100 76,800 2,72,000
28,93,600
136
Single or Output
Costing
Production account: production account also presents cost details like the cost sheet
with the difference that it is in the form of larger account. Production account is prepared
in T-form under costing. It shows output of the product during a given period, its total
cost and per unit cost, with cost components and profit and loss. It is prepared to suit the
needs of the organization. It may contain information regarding total cost only, or per unit
cost also, or even details regarding units of output. It may be divided in two, three or four
parts – the first part gives prime cost, the second part gives cost of goods manufactured or
production cost, the third part shows gross profit, and fourth shows the net profit formal
of production account is given below:
Particulars Rs Particulars Rs
To opening stock of raw materials XXX By closing stock of raw materials XXX
To direct materials purchases XXX By net value of normal XXX
To direct labour XXX scrap of raw materials XXX
To direct expenses XXX By prime cost c/d XXX
In brief when cost sheet is prepared in T-shap account, it is known as production account.
in this account of debit side it shows the various items of cost, while its credit side shows
the sales of finished goods. Opening stock is written on debit side and closing stock on
credit side. The balance shows profit or loss.
Illustration 3.: prepare separate production account of coal and coke from the following
information:
Wages paid for coal 5,80,000 Salaries 36,000
Coal for colliery 45,000 coal sold (including colliery)
Timber used in coal 64,000 1,12,000 8,84,000
Ropes used in coal 12,000 wages of coke 50,000
Stores used in coal 76,000 stores for coke 37,000
Royalties paid 42,000 salaries for coke 8,000
General charges 70,000 coke sold (43,500 tons) 5,40,000
Coal stock at the beginning is 7000 tons valued at Rs. 5 per ton and at the end 15,000
tons valued at the same rate. The stock of coke in the beginning and at the end was 2,000
tons and 500 tons respectively at the rate of Rs. 10/-. The total production of colliery was
1,85,000 Tons of coal and 42,000 tons of coke. 65,000 tons of coal being used for coke
making [B.Com (Delhi)].
Solution:
COAL PRODUCTION ACCOUNT
Particular Rs Particular Rs
Rs
To wages 5,80,000 By cost of production 9,25,000
To coal for colliery 45,000
To timber 64,000
To ropes 12,000
To stores 76,000
To royal cities 42,000
General charges 70,000
To salaries 36,000
9,25,000 9,25,000
To opening stock By sales 8,84,000
7,000*5 35,000 “Coke Production .
A/c @5/- 3,25,000
To cost of production 9,25,000 (65,000*5)
To profit 3,24,000 By closing stock
15,000*5 75,000
12,84,000 12,84,000
138
Single or Output
Costing
Illustration 4: M works can produce 60,000 units per annum at 100% capacity. The
estimated cost of production areas under:
Direct material Rs 3 per unit
Direct labor Rs 2 per unit.
Indirect expenses-
Fixed Rs 3, 00,000 per annum.
Variable Rs 5 per unit.
Semi variable: Rs 100,000 per annum up to 50% capacity and an extra
Rs 20,000 for every 25% increase in capacity or part thereof.
If management wishes to get Rs 2, 00,000 profit for the year as production programmed.
What will be the average selling price per unit:
First 3 months 50% of capacity.
Remaining 9 months 80% of capacity. [C.A.]
SOLUTION:
PRODUCTION STATEMENT (43500 OUTPUT UNITS)
Particular Rs Rs
Direct materials 43,500 units @ Rs. 3 per unit 1,30,500
Direct Labour 43,500 units @ Rs. 2per unit 87,000
Prime cost 2,17,500
Indirect expenses :
Fixed 3,00,000
Variable 43,000 units @ Rs. 5/ 2,17,500
Semi-variable
For first 3 at 50% Capacity
1,00,000* 3/12 25,000
For 9 months at 80%capacity
1,40,000*9/12 1,05,000
Cost of
production 6,47,000
Profit 8,65,000
sales prices 2,00,000
per unit sale price: 10,65,000/43,5000 units = Rs. 24.5 10,65,000
139
Cost and Management
Accounting
Particular Kg Rs
140
Single or Output
Costing
WORKING NOTES:
(1) closing stock of raw materials:
Rs. 3, 60,000
2400 kg Rs.2700
Rs. 3,20,000
(2) closing stock of finished goods:
Rs. 10,32,400
900 kg Rs.3, 036
3,06.000
141
Cost and Management
Accounting
Illustration 4: The following data were obtained from the record of a manufacturing
concern:
Rs
Purchases of raw materials 1,20,000
Direct wages 1,00,000
Rent and rates 40,000
Carriage inward 2,000
Opening stock-
Raw materials 20,000
Finished goods 16,000
Closing stock-
Raw materials 22,000
Finished goods –
Work in progress-opening 4,800
Work in progress-closing 16,000
Cost of factory supervision 8,000
Sales of finished goods 3,00,000
Advertising @ 0.40 per ton sold. 64,000 tons of the commodities were produced.
Prepare cost sheet.
[Delhi B. Com.]
Solution:
Cost Sheet
Particular Per ton Total
Rs Rs
Opening stock raw material 20,000
Add: purchase 1,20,000
1,40,000
Less: closing stock 22,000
Material consumed 1,18,000
Direct wages 1,00,000
Prime cost 2,18,000
Factory overheads:
Rent and rates 40,000
Carriage inwards 2,000
Factory supervision 8,000 50,000
2,68,000
Add: opening work in progress 4,800
2,72,800
Less: closing work in progress 16,000
Cost of production 4.01 2,56,800
Add: opening finished goods 16,000
2,72,800
Less: closing finished goods
(4,000*4.0125) 32,100
Cost of goods sold 2,40,700
Selling overheads (4,000+64,000-8,000)
=60,000 ton @ 0.40 0.40 24,000
Cost of sales 4.41 2,64,700
Profit 0.59 35,300
5.00 3,00,000
143
Cost and Management
Accounting
The company plans to manufacture 2400 machines in 2016. Prepare a statement in which
the company will show a profit of 10% on selling price.
Additional Information:
(1) the price of materials will be rise by 20%
(2) wages rate will rise by 5%
(3) manufacturing expenses per unit will rise in proportion to the combined cost of
material and wages
(4) selling expenses per unit will remain unchanged.
(5) Other expenses will remain unaffected by the rise in output.
Solution:
Statement of Cost
Output 2,400units
Particulars Rs Rs
Materials (80+20%) = 96 2,30,400 96.00
Direct wages (120+5%) = 126 3,02,400 126.00
Prime cost 5,32,800 222.00
Manufacturing expenses 1,33,200 55.50
Work cost 6,66,000 277.50
Administrative expenses
Rent and rate 20,000 8.33
Staff salaries 1,20,000 50.00
General expenses 40,000 16.67
8,46,000 352.50
Selling expenses 72,000 30.00
Cost of sales 9,18,000 382.50
Profit – 10% on selling price {10/90} 1,02,000 42.50
Selling price 10,20,000 425.00
144
Single or Output
Costing
A work order has been received and the estimated expenses are as under-
Materials 16,000 wages Rs 10,000.
Assuming that the rate of factory expenses has gone up by 20%, distribution charges have
gone down by 10% and selling and administration charges have gone each up by 15%. At
what price should the product be sold as to earn the same rate of profit as before. Factory
expenses are based on wages, and administrative, selling and distribution expenses
are based on factory cost.
[Banglore B.Com.]
Solution:
COST SHEET
Particular Rs
Direct materials 12,00,000
Wages 10,00,000
Prime cost 22,00,000
Factory expenses: 60% of wages 6,00,000
Factory cost 28,00,000
Administrative expenses 24% of factory cost 6,72,000
Cost of production 34,72,000
Selling expenses (16% of factory cost) 4,48,000
Distribution expenses (10% of factory cost) 2,80,000
Cost of sales 42,00,000
Profit 20% on total cost 8,40,000
sales 50,40,000
7.8 QUESTIONS
146
Single or Output
Costing
SUGGESTED REFERENCE
Mittal & Maheswari, Elements of Cost Accounting, Shree Mahavir Book Depot
(Publishers) 2015.
Varshney J.C., Principles and Practice of Cost Accounting, Wisdom Publication
House (2009).
Mittal & Maheswari, Management Accounting, Mahavir Publication, 2015.
Arora M.N. Cost Accounting, Vikas Publishing House Pvt. Ltd. 2013.
Jain & Narang, Cost Accounting, Kalyani Publishers 1998.
Arora M.N., Management Accounting, Himalaya Publishing House, 2006.
147
UNIT-8 JOB BATCH AND CONTRACT COSTING
LEARNING OBJECTIVES
STRUCTURE
8.1 Introduction
8.2 Meaning of Job Costing
8.3 Meaning of Batch Costing
8.4 Meaning of Contract Costing
8.5 Main aspect of contract costing
8.6 Treatment of Profit and Loss on Contract Costing
8.7 Questions
The job cost record will report the direct materials and direct labour actually used plus
the manufacturing overhead assigned to each job. An example of an industry where job
order costing is used is the building construction industry. Job costing involves the
accumulation of th3e cost of material, labour and overhead for a specific job. This
approaches is an excellent techniques for tracing specific costs to individual jobs and also
examining them to see if the costs can be reduced in later job. It is applied where each job
may be different from the next and consumers different resources.
Steps in Job costing: Some of the important steps in job costing are discussed below
(i) Identify the job that is chosen cost object.
(ii) Identity the direct cost of the job i.e. direct material and direct labour.
(iii) Select the cost allocation base to use for allocating indirect costs to the job.
(iv) Match indirect cost to their respective cost allocation best.
(v) Calculate an overhead allocation rate by applies the following formula.
- Budgeted overhead costs
- Budgeted overhead allocation base.
- Actual overhead cost
- Actual overhead allocation base.
(vi) Allocate overhead costs to the job overhead allocation rate Actual base activity
for the job.
(vii) Compute total job costs by adding all direct and indirect costs together.
148
Job Batch and
Contract Costing
Job costing is used by organisations whose products or services can readily, identified by
individual units or batches. Each receives varying inputs of indirect materials, direct
labour and factory overhead. Industries that use job order methods include construction,
printing, aircraft, furniture and machinery.
Non-manufacturing organisations that use job costing include auto repair, auditing,
consulting engagements, hospital cases, social welfare cases am research projects.
Illustration: 1 A factory uses job costing. The following data are obtained front the
books
Rs. Rs.
Direct materials 180,000 Selling and distributive 105,000
expenses
Direct wages 150,000 Administrative expenses 84,000
Profit 121,000 Factory overheads 90,000
149
Cost and Management
Accounting
If selling and distirbution overheads have gone up by 15%. The factory intends to earn
the - same rate of profits on sales. The factory recovers factory overhead on direct
wages and administrative and selling expenses on works cost. Find out the selling price
of the order received. (CA Inter)
Solution :
Production Statement
Direct Material 180,000
Direct Wages 150,000
Prime cost 330,000
Factory overheads 90,000
Works cost 420,000
Administrative overheads 84,000
Cost of Production 50,4000
Selling and Distributive expenses 105,000
Cost of sales 609000
Profit (given) 121,800
Sales price 7,30,800
150
Job Batch and
Contract Costing
A batch is a cost unit consisting a group of identical items. Batch costing applies, when
production consist of limited repetition work and a definite number of goods are
manufactured in each batch for sale to the customers. Batch costing is applied in the
production of toys, readymade garments, cars parts radios, watches and shoes etc.
Procedure : Each batch is given a batch number. In Batch cost card, direct materials,
direct labour and direct expenses are recorded, overheads are absorbed on the basis of
certain criterion. When batch is completed the total cost of the batch is divided by the
quantity produced and per unit cost is arrived at.
Illustration 2. A component of a machine is made entirely in cost centre. Material cost is
12 paisa per component and each component takes 10 minutes to produce. The machine
operator is paid Rs 1.44 per hour and the machine hour rate is Rs 3,00. the setting up of
the machine to produce the component takes 2 hours 20 minutes. Prepare a comparative
cost sheet showing the production cost of a batch of (i) 10 components (ii) 100
components and (iii) 1000 components. (CA Inter).
Solution :
Comparative cost sheet
Component
Batch sizes of components
Units 10 100 1000
Setting up cost :
Labour 2 hours 20 minutes
Rs.
at Rs 1.44 per hour 3.36
overheads 2 hours 20 minutes at
Rs 3.00 per machine 10.36 10.36 10.36
hour: 7.00
Production cost : 1.20 12.00 120.00
Materials @ 12 paise per component 2.40
wages @ Rs. 1.44 per hours :
For 10 components 1 hour 40 minutes
24.00
For 100 components, 16 hrs 40 minutes
240.00
For 1000 components, 166 hrs 40 minutes,
overheads : @ Rs. 3.00 per
Machine hour
For 10 components : 1 hrs 40 minutes 15.00
For 100 components : 16 hrs 40 minutes 50.00
For 1000 components : 166 hrs 40 minutes 500.00
Total cost 18.96 96.36 870.36
Cost per units 1.88 0.96 0.8703
151
Cost and Management
Accounting
Economic Batch quality [EBQ]: In the industries, setting up cost is a fixed nature,
which remains unchanged with the increase or decrease in the batch size. Thus if the
number of units in a batch is larger, the setting up cost per unit will be lower. Hence in
the industries, the optimum quantity in a batch is determined, at which cost per unit may
be minimum. This is known as Economic Batch Quantity. While determining economic
batch Quantity, the following two types of costs are to be considered. Such as (i) Setting
up cost and (ii) Carrying cost
(i) Setting up cost: It is the cost of setting the machine and tools used for production.
This is of a fixed nature. When the size of the batch increase, setting up cost per article
becomes lower.
(ii) carrying cost: It includes the cost of storage, interest on capital etc. When the size
any batch is larger the carrying cost will also be higher and vice-versa.
Main consideration:
In determining the economic batch quantity, the main consideration are as under: (i) -
Cost and time taken in setting up
(i) Cost and time taken in manufacturing
(ii) Cost of storage
(iii) Rate of consumption
(iv) Interest on capital invested Formula:
Formula :
2AS
EBQ =
C
Where : EBQ = Economic Batch Quantity
A = No. of units to be produced
S = Set up costs per batch
C = carrying cost per unit
Illustration 3 : A manufacturer has to supply 10,000 unit per day to a mill. He finds that
when he starts production he can produce 50,000 units per day. The cost of holding on
unit in stock for one year is 4 paisa and the setting up cost of production run is Rs 36.
How frequently should production runs be made ? [I C. W.A Inter]
Solution :
2AS
EBQ =
C
A = 10,000 x 365 days = 365,000
S = Setting up cost per unit = Rs 36
C = Carrying cost per unit = 0.04
2AS
Thus EBQ =
C
152
Job Batch and
Contract Costing
2 36,5000 36
= 81,056 units
0.04
81, 056
Frequency of production = 8days
20, 000 units per day
Illustration 4: The following direct costs were incurred on a job of standard Ratio
company :
Materials Rs. 12,020
Wages :
Deptt X : 120 hrs @ Rs 30 per hour
Y : 80 hours @ RS. 20 per hour
Z : 40 hours @ RS. 50 per hour
Overhead costs:
Variable overheads
Deptt. X : Rs. 30,000 for 3,000 labour hours.
Y : Rs. 8,000 for 400 labour hours.
Z : Rs. 24,000 for 600 labour hours.
Fixed overheads:
Rs. 80,000 for 4000 normal working hours,
Charge profit 25% on selling price.
Contract costing is that methods of costing in cost according which is used to collect and
identity all the expenses relating to a specific contract. For this purpose, a contractor has
to maintain contract ledger in which he has a show contract account. Every clause in a
contract, regardless of whether if applies to economic or non-economic issues, can have
cost implications. However, the importance of contract costing depends on the share of a
company's total costs on labour. Therefore, contract costing is a special type of job
costing where the unit of cost is a single contract. According to ICMA terminology,
contract costing is that term of specific order costing which applies where work is
undertakes to customer's special requirements and each order is of long duration. The
work is usually of construction nature. Contract costing is also known as terminal
costing
Some of the important aspect of contract costing are discussed here below:
(1) Material: It includes the following items.
Cost of material – It includes material purchased for the contract and materials issued
from stores. The cost of both these materials is derived to the contract account.
(2) Material returned to store: Materials which are returned back to store are credited to
contract account.
Materials at site: When any material is lying unused at site, that material will be credited
to contract account.
154
Job Batch and
Contract Costing
(3) Labour: All wages of workers engaged on a contract are charged to the contract
account.
Plant and Machinery : Following methods are often used for charging the contract for
use of Plant and Machinery.
If machinery is used at contract site for along period of time, the particular
contract may be debited with the value of plant (original cost or written down
value) at the beginning and credited with the depreciated value at the end of the
accounting period. If plant and machinery is returned to head office before the
accounting period, the depreciated value of plant and machinery on that
particular date is credited to contract account.
Total depreciation during a period is calculated for each plant and concerned
contract account is debited with depreciation of plant and machinery.
When plant is sent to contract for short period contract account is debited by the
account of depreciation.
When a plant is hired for a contract, the hire charges are debited to contract
Account.
(v) Work Certified: It is that part of work-in-progress which has been approved or
certified or authenticated and valued by the experts called certifier known as
work certified. It is a part of work actually done and it is generally value at a
contract price.
(vi) Work uncertified: The work done but not certified by the surveyor or Architect
is termed as work uncertified . It is valued at cost and credited to contract account
and debited to work-in-progress account. Both work certified and uncertified are
a part of work-in-progress and are credited to contract account.
(vii) Work-ion-progress: It includes the amount of work certified and the amount of
work uncertified. The work in progress account will appear on the asset side of
the Balance-sheet. The amount of cash received from the contractor and reserve
for contingencies will be deducted out of this amount.
155
Cost and Management
Accounting
(b) Completion of contract is upto 50% or more but less than 90% = In this case 2/3 rd of
the notional profit should be transformed to profit and loss account. It can be
expressed as
2 Cash received
Notional Profit
3 Work certified
(c) Completion of contract is upto 90% or more than 90% i.e. Near completion contract
When contract is near completion, then the estimated profit should be calculated on the
whole contract. The proportion of estimated profit to be transferred to Profit and Loss
Account is computed by any one of the following formulas:
Work certified
Estimated profit
Contract price
Work certified Cash received
Estimated profit
Contact price Work certified
Cost of work to date
Estimated profit
Estimated total cost of work
Cost of work to date Cash received
Estimated profit
Estimated total cost of work Work certified
(d) Loss on Uncompleted contract : In the event of a loss on uncompleted contract. This
should be transferred in full to the profit and loss account.
Notional Profit: Notional profit is the difference between the value of work-in-progress
certified and the cost of work-in-progress certified. Notional profit = Value of work
certified + Cost of work not yet certified Cost of work to date.
Estimated profit: It represents the excess of the contract price over the estimated total
cost of the contract. Estimated profit = contract price total cost already incurred
estimated additional costs to complete the contract.
Retention Money: Fuller amount of work certified is never paid to the contractor.
contract may pay a fixed percentage of the work certified and it is known as cost
ratio. The balance amount which is not paid is known as Retention money. This
156
Job Batch and
Contract Costing
retention money is treated as security money and may be utilised for any defective
work done or found during the contract work. The retention money may be adjusted
against any penalties imposed, if any contract is not completed within the stipulated
time.
Extra Work:- Sometimes, the contractor is required to do some extra work is
addition to the work originally assigned to him. For such extra work the contractor
will charge extra money which will be credited in the contract account and whatever
cost of such extra work is made, it will the debited to contract account.
Esclation De-Esclation class: This clause is generally provided in long term contract
with a view to protecting both the contractor and the contractee against fluctuations
in the price of inputs to the contract, mainly material and labour. The clause provides
for changes in the price of contract to cover jphanges in the price of raw materials
and labour. The contract price is revised in an agreed proportion to the changes in the
price of inputs.
Edelation clause aims at safeguarding the interest of the contractor against unexpected
rise in cost. On the other-hand dc-esclation clause provides for a decrease in the contract
price due to a decrease in the price of inputs so that the benefit of price decrease is passed
on the contractee.
Cost plus Contract: These contracts provide for the payment by the contractee of
the actual cost of manufacturing plus a stipulated profit. The profit to be added to the
cost may be a fixed amount or it may be a stipulated percentage of cost.
Plant purchased Rs. 60,000; Materials issued to site Rs. 3,36,000; Wages paid Rs.
3,40,000; Direct expenses Rs. 8,000; General overheads apportioned Rs. 32,000; Wages
accrued due as on 31.12.15 Rs. 2,800; Materials at site as on 31.12.2015, Rs. 4,000;
Direct expenses accrued as On 31.12. 2015 Rs. 1,200; Work not yet certified at cost Rs.
14,000; Cash received being 80% of work certified Rs. 6,00,000. Life of plant purchased
is 5 years and scrap value is nil. Prepare the contract account for .the year ended
31.12.2015 and show the amount of profit which you consider might be fairly taken on
the contract. Show complete calculations. (B,.Com, Delhi)
Working Notes:
Cash received 2
Profit to be taken = Notional Profit
Work certified 3
6, 00, 000 2
36, 000 Rs.19, 200
7,50, 000 3
Plant at site = Cost less depreciation = 60,000 20% of 60,000 = Rs. 48,000.
Illustration 6: Surya Construction Ltd. started business with a paid up capital of Rs. 50
lakhs. On 1st April, 2015, it undertook a contract to construct a building for Rs. 60 lakhs.
Cash received on account of the contract upto 31st March, 2015 was Rs. 18 lakhs (being
90% of work certified). Work uncertified as on 31st March, 2015 was estimated at
Rs.1,00,000. As on 31st March, 2015, the cost of materials at site was Rs.30,000 and
outstanding wages were Rs.5,000. Of the plant and machinery charged to the contract,
machinery costing Rs.2,00,000 was returned to stores on 31st March, 2015. Plant and
machinery charged to the contract is to be depreciated at 5%. The following were the
ledger balances as per the trial balance as on 31st March, 2015.
Rs.
Land and building 23,00,000
Plant and machinery (60% at site) 25,00,000
Furniture 60,000
Materials 14,00,000
Fuel and Power 1,25,000
Site expenses 5,000
Office expenses 12,000
Rates and taxes 15,000
Cash'at bank 1,33,000
Wages 2,50,000
158
Job Batch and
Contract Costing
Prepare Contract Account and Balance Sheet for the year ending 31st March, 2016
Contract Account for the year ended 31st March, 2016.
Particulars Amount Particulars Amount
(Rs.) (Rs.)
To Materials 14,00,000 By Materials at site 30,000
To Wages By Plant returned
2,50,000
Add : Outstanding 5,000 2,55,000 (2,00,000 - 5% of 2,00,000) 1,90,000
To Plant and Machinery at site By Plant at site
(60% of Rs. 25,00,000) 15,00,000 [(15,00,000 - 2,00,000)
To Fuel and Power 1,25,000 -5% of 13,00,000] 12,35,000
To Site expenses 5,000 By Work-in-progress
159
Cost and Management
Accounting
Working Notes
(1) . Contract price = Rs. 60,00,000
Work certified = Rs. 20,00,000
20, 00, 000 1
Percentage of work certified to the contract price = 100 33 %
1
Since percentage of work certified to the contract price is 33 % , 3the profit to be
60, 00, 000
3
transferred to the Profit & Loss Account would be calculated as follows :
Cash Received
Notional Profit =
1 18,00,000
i.e. 2'43'000 Work
Certified
Rs.72,900
3 20,00,000
Illustration 7: The following information is available to a building contract for Rs.
20,00,000 for two years. (Delhi, B.Com.)
I year II Year
Rs Rs.
Materials 6,00,000 1,68,000
Direct Wages 4,60,000 2,10,000
Direct expenses 44,000 20,000
Indirect expenses 12,000 2,800
Work certified 15,00,000 20,00,000
Work uncertified 1,600 —
Materials at site 1,000 14,000
Plant issued 28,000 4,000
Cash received 12,00,000 20,00,000
160
Job Batch and
Contract Costing
Contracte's A/C
Rs.
Rs.
T Balance c/d 1200000 By Cost
1200,000
12,00,000
12,00,000
Contract Account
Rs. Rs.
To Work in progress :- By Contrect A/C 20,00,000
Certified 15,00,000 By Materials at site 14,000
uncertified 16,000
Less: Reserve 1,84,800 13,31,200
To Materials at site 10,000
To Materials issued 1,68,000
To Direct Wages 2,10,000
To Direct expenses 20,000
To Indirect expense 2,800
To Depreciation on plant 8,000
(28,000 + 4,000 - 14,000 - 10,000)
To P/LA/C 2,64,000
20,14,000 20,14,000
Contracted A/C
By Balanceb/d 12,00,000
To Contract 20,00,000 By Cash (Balance) 800,000
20,00,000 20,00,000
161
Cost and Management
Accounting
The contract price is Rs 6,00,000 and work certified was Rs. 30,000. the work uncertified
being Rs 2,000. Machinery costing Rs 4,000 was returned to store at the end of the year.
Stock of materials at site on 31st December was rs 10,000. Wages outstanding were Rs
400. Depreciate machinery at 10% Prepare contract Account and Balance sheet.
[Delhi B. Com. (Hons)]
Solution :
Contract Account
Rs. Rs.
To Materials 80,000 By Work in Progress 3,00,000
To Direct Wages 1,10,000 capital in 2,000
To expenses 4,000 certified
To Wages outstanding 400 10,000
To Fuel and power 5,000 By Materials at site 50,400
To Machinery 60,000 By Machinery at site 3,600
To Balance c/d 1,06,600 By Mac binary returned
3,66,000 3,66,000
To P/L A/C By Balance b/d 106,600
1,06,600x2/380% 56,854
To Reserve 49,746
1,06,600 1,06,600
Balance Sheet
Liabilities Rs. Assets Rs.
Work in progress 300,000
Certified 2000
Uncertified
Less : Cash
Received
Less : Reserve
302000
240000
62000
49746
12254
Illustration 9: A contract was commenced on 1st April. The following information is
available of a contract on 31st December:
Rs.
Material 5,16,200
Labour 11,21,000
Formen's Salary 15,86,000
162
Job Batch and
Contract Costing
Plant costing Rs. 5,20,000 had been on site for 146 days. The scrap value being Rs
30,000 and working life 7 years. A supervisor salary Rs 8,000 Plant and Machinery has
devoted 3/4 of his to this contract. The administrative expenses being Rs. 2,80,000.
Materials at site Rs. 50,500. Materials costing Rs. 9,000 was sold for Rs. 8,000 and a
plant costing Rs. 11,000 found unsuitable and was sold at a profit Rs. 2,000.
The contract price was RS 44,00,000 but is was accepted for Rs 40,00,000.2/3 of the
contract was completed and work certified was 50% of contract price and Rs 15,00,000
was received in cost.
Prepare contract Account and Balance sheet as on 31st Decemeber. [CA. Inter]
Solution:
Contract Account
Rs. Rs
.
To Materials Wages 5,16,200 By Materials at site 50,800
To Wages 11,21,000 By Materials sold 8,000
To Formens salary 1,58,600 By Loss on sale of 1,000
To Supervisor salary By materials Work in
3 54,000 progress
3 8,000 9
4
To Depreciation of Certified 20,00,000
plant 2,80,000
146 (Balance Amount) 5,24,500
(5, 20, 000 30, 000)t
7 365 incertified
25,84,300 25,84,300
To P/LA/C By Balance b/d 2,26,500
150000 2,13,250
426500 2 / 3
2000000
To Reserve 2,13,250
4,26,500 4,26,500
Work uncertified:
Contract completed 2/3, work certified 50%
1
Work uncertified = 2/3 1/ 6
2
Contractee Account
Rs. Rs.
To Balance 15,00,000 15,00,000
By Cash c/d
Balance Sheet
Liabilities Rs. Assets Rs.
Profit & Loss A/c Material at sit 50,800
Profit on contract 2,13,250 Plant at site 4,81,000
Add: Profit on Work in progress
Work certified
20,00,000
8,11,250
1000 work uncertified
Sale of plant
214,250 5,24,500
Less : Loss on 25,24,500
Add of 1000
Materials
213,250
Less Reserve
2,13,250
23,11,250
Less cash
15,00,000
164
Job Batch and
Contract Costing
The value of the contract is Rs. 2,15,000 and it is the practice of the contractee to retain
10% of work certified. From the above, prepare a Contract Account and show how the
various items would appear in the Balance Sheet.
165
Cost and Management
Accounting
Rs.
A B A B
To Materials 1,70,698 1,46,534 By Materials 1,098 1,264
To Labour 1,48,750 1,37,046 By Materials in hand 3,472
3,766
To Plant 30,000 25,000 By Plant in 22,000 19,000
hand
To Direct expenses 6,334 5,718 work
certified
To Establishment exp. 8,252 7,704 Work 3,90,000 2,90,000
certified
To Wages accrued 4,800 4,200 By Balance loss
to
To Direct expenses
accured 480 360 P/L A/C — 6,826
To Balance c/d 56,550 —.
42,58,864 3,26,562 42,58,864 3,26,562
To P/L /A/C By Balance b/d 56,550 —
2 360000 34,800 —
56550
3 395000
To Reserve 21,750
56,550 — 56,550
166
Job Batch and
Contract Costing
Contractee's Account
A B A• B
17,250
16,000
Direct expenses Accrued
480 + 360
Profit: Contract A : 34,800
Less: Loss of
Contract B 6,826
3,99,000
Less : Reserve
2,17,750
3,77,25
0
Less : Cash Received
3,60,000
Contract B :
Work certified:
2,90,000
Work uncertified
6,000
2,96,000
Less Cash Received
2,80,000
167
Cost and Management
Accounting
(i) Contract A/c showing profit transferred to Profit & Loss A/c «tad
(ii) ContracteeA/c
168
Job Batch and
Contract Costing
Contractee A/c
Particulars Rs. Particulars " Rs.
By Ban A/c 80,000
To Contractee A/c 195000 By Balance A/c 15000
195000 195000
Illustration 13: Two contracts, commenced on 1st January and 1st July, 2025
respectively, were undertaken by a contractor and their accounts on 31st December
2015 showed the following position:
Contract Contract
I II
Contract price 4,00,000 2,70,000
Expenditure: Materials 72,000 58,000
Wages paid 1,10,000 1,12,000
General charges 4,000 2,800
Plant installed 20,000 16,000
Materials on hand 4,000 4,000
Wages accrued Work 4,000 4,000
certified 2,00,000 1,60,000
Cash received in 1,50,000 1,20,000
Work uncertified 6,000 8,000
Additional Information: The plant was installed on the date of commencement of each
contract; depreciation thereon is to be taken at 10% per annum. Prepare the Contract
Accounts in the tabular form and ascertain the profit or loss to be taken to Profit & Loss
Account.
Solution
Contract Account
Particulars Contact Contact Particulars Contact Contact II
1 (Rs.) II (Rs.) 1 (Rs.) (Rs.)
Materials 72,000 58,000 Materials in hand 4,000 4,000
Wages. 1,10,000 1,12,00 Plant at site 18,000 15,200
Wages accrued 4,000 4,000 Work-in-progress
General charges 4,000 2,800 - Certified 2,00,000 1,60,000
Plant installed 20,000 16,000 - Uncertified 6,000 8,000
To National profit 18,000 By Loss transferred to
Profit & Loss A/c 5,600
By Notional Profit
2,28,000 .1,92,800 2,28,000 1,92,800
9,000 18,000
169
Cost and Management
Accounting
9,000
18,000 18,000
Working notes:
- Contract II was undertaken on 01.07.15
- Depreciation has been charged for six months
- In Contract I, transfer to Profit & Loss Account is calculated as under:
2 1,50, 000
Rs.18, 000 Rs.9, 000
3 2, 00, 000
8.7 QUESTIONS
1. What is the difference between job costing and contract Costing Rs. Explain the
objectives and procedure of job costing system.
2. What do you understand by Batch Costing Rs. In which industries is applied ?
3. Explain briefly the distinguishing features of contract costing.
4. Write short notes on : (i) Escalation Clause, (ii) Cost Plus Contracts. (iii) Features of
Job Costing.
5. Mayur Engineering, engaged in job work, has completed all jobs in hand on 30th
December, 2006 except Job No. 447. The cost sheet on 30th December showed direct
materials and direct labour costs of Rs. 40,000 and Rs. 30,000 respectively as having
been incurred on Job No. 447.
The costs incurred by the business on 31st December, 2015, the last day of the accounting
year, were as follows:
Direct Materials (Job 447) Rs. 2,000
Direct Labour (Job 447) Rs. 8,000
Indirect Labour Rs. 2,000
Miscellaneous Factory Overheads Rs. 3,000
It is the practice of business to make the jobs absorb factory overheads on the basis of
120 per cent of direct labour cost.
Calculate the value of Work-in-progress of Job No. 447 on 31st December 2015.
[AnsRs.1,25,600]
6. From the following particulars relating to four jobs of a manufacturer, ascertain the
total cost each job :
Job Job Job Job
No. 1 No. 2 No: 3 No. 4
Direct Materials 800 1,000 1,200 1,400
Direct Wages 400 500 600 700
Direct Expenses 80 100 120 140
Works overhead is 45% on prime cost, and office overhead is 15% on works cost.
170
Job Batch and
Contract Costing
7. How much profit will be credited to profit and loss account in the following case :
Contract price Rs. 20,00,000
Cost incurred Rs. 1 1,20,000
Cost received (90% of work certified) Rs. 10,80,000
Work not certified Rs. 1 ,20,000
[IB. Com. Delhi] [Ans.Rs. 1,20,000]
8. The following expenditure was incurred for the contract on construction of the
building of 12,00,000 for the year ending 31-12-2011 :
Materials 2,40,000
Wages 3,28,000
Plant 40,000
Overheads 17,200
Cash received on account of the contract to 31 st Dec. 2011 was Rs. 4,80,000, being 80%
of the work certified. The value of materials in hand was ^ 20,000. The plant had
undergone 20% depreciation. Prepare Contract Account.[B.Com. (Pass), Delhi, 2012]
[Ans. Profit to P&L A/c Rs. 14,293; Work-in-Progress A/c Rs. 12,507]
9. Following amounts have been spent on a contract still unfinished on 31st December,
2006 :
Materials Rs. 80,000
Wages Rs. 70,000
Direct Charges Rs. 50,000
Rs. 2,00,000 have been received from the contractee being 80% of the work certified.
Calculate profit to be credited to Profit and Loss Account, uncertified work in progress
being Rs. 10,000. Total value of the Contract is Rs. 4,00,000.
[Ans. Profit to P.& L. A/c Rs. 32,000, Balance being Reserve Rs. 28,000.]
SUGGESTED REFERENCE
Mittal & Maheswari, Elements of Cost Accounting, Shree Mahavir Book Depot
(Publishers) 2015.
Varshney J.C., Principles and Practice of Cost Accounting, Wisdom Publication
House (2009).
Mittal & Maheswari, Management Accounting, Mahavir Publication, 2015.
Arora M.N. Cost Accounting, Vikas Publishing House Pvt. Ltd. 2013.
Jain & Narang, Cost Accounting, Kalyani Publishers 1998.
Arora M.N., Management Accounting, Himalaya Publishing House, 2006.
171
UNIT-9 PROCESS COSTING
LEARNING OBJECTIVES
STRUCTURE
9.1 Introduction
9.2 Procedure of Process Costing
9.3 Accounting for Losses and Gains in Process costing
9.4 Process Losses and Wastages
9.5 Questions
9.1 INTRODUCTION
The method of cost accounting used by processing firms is called process costing system.
For each process function, product costs includes direct materials, direct labour and
factory overhead system. Process costing is a special branch of costing used by the
manufacturing industries; who are involved in converting the raw materials into the
finished product. Such work of conversion is done step by step. Each step is called as a
process. Process costing is a method of allocating manufacturing cost to products to
determine an average cost per unit. Process costing is a form of operation costing used
where production follows a series of sequential process. Under this method of costing all
costs are accumulated for each stage of production also called process of production) and
the cost per unit of product is ascertained at each stage of production by dividing the total
cost of each process by the normal output of that process.
Accordign to CIMA, London, process costing as "that form of operation costing which
applies where standardised goods are produced."
Features : The following are the most common features in the process costing systems:
The production is continuous and homogeneous.
The processes are standardised.
The output of one process becomes the input of another process.
The output of the last, process is transferred to Finished Stock Account.
Costs are collected process-wise. Process cost centres are clearly defined and all
costs are accumulated by the cost centres.
Cost per unit, is calculated at the end of period by dividing the total process cost
by the normal output produced.
172
Process Costing
173
Cost and Management
Accounting
(ii) Effectiveness of each process is decided on the basis of cost incurred at individual
process.
(iii) Cost data are available process-wise for exercising managerial control.
(iv) Since output at each process is homogeneous average "cost per unit can be easily
computed.
(v) Make or buy decision for different processes can be taken in the light of costs at
different process.
(vi) Valuation of inventory of work-in-progress at different processes and finished
products in facilated by process accounts.
(vii) It helps the management in determining or fixing up price quotations. -
Illustration 1. A product passes through three distinct processes in completion. 100 units
are produced. The following information is available :
Process A Process B Process/C
Rs Rs Rs
Materials 13000 6,000 4,000
Labour 10,000 8,000 10,000
Direct expenses 3000 1400 2,000
174
Process Costing
The indirect expenses are Rs. 5,600 to be apportioned on the basis of labour
Prepare process accounts Solution:
Process A Account
Particulars Amount Particulars Amount
(Rs.) (Rs.)
To Materials 13000 By Process B 28000
To Labour 10,000
To Direct expenses 3000
To Indirect expenses 2,000
28000 28000
Process B Account
Particulars Rs. Particulars Rs.
To Process A 2000 By Process C 59400
To Material 6,000
To Labour 8,000
To Direct expenses 1400
To Indirect expenses 16.,000
59400 59400
Process C Account
Particulars Rs. Particulars Rs.
To Process B 59400 By Finished stock 79400
To Materials 4,000
To Labour 10,000
;
To Direct expenses 2,000
To Indirect expenses 2,000
77,400 77,400
175
Cost and Management
Accounting
In most of the industries a certain amount of loss occurs at various stages of production.
This loss might be due to chemical reaction. Proper record is to be maintained of such
type of losses. This loss may be of two types as under:
Abnormal loss is an avoidable loss which occurs due to abnormal reasons like sub
standard materials, carelessness of work. Machinery breakdown, unplanned operations
etc. Such losses are in excess of pre-determined normal losses. Such losses can not be
estimated in advance. Thus, abnormal losses arise when actual losses are more than
expected losses.
Its value is credited to the concerned Process Account and debited to Costing Profit and
Loss Account.
Illustration 2. The following data have been collected from the Ledger Accounts :
Input - Materials - 1000 tonnes Rs 150 per tonne
Wages Rs. 20,000
Overheads Rs. 10,000
Normal wastage @ 5% which can be sold at Rs. 5 per tonne. Actual production is 900
tonnes.
Compute the value of abnormal loss and show the relevant accounts.
177
Cost and Management
Accounting
Abnormal Gain or Effectiveness: Abnormal Gain arises when the actual loss is smaller
than estimated loss.
Abnormal Gain = Normal Loss Actual Loss
178
Process Costing
Cr.
Particulars Units Amount Particulars Units Amount
(Rs. ) (Rs. )
To Normal Loss A/c -------- -------- By Process A/c ------- ------
To Costing P & L --------- --------
A/c
-------- -------- -------- --------
Illustration 3. The product of Company A passes through two processes A and B and
then to finished Stock Account. In each process 5% of the total weight is lost and 10% is
scrap which realises from process A Rs. 80 per tonne and Process B Rs. 200 per tonne
respectively.
The following are the figure relating to both the processes :
Process A Process
B
Materials (tonnes) 1,000 70
Cost of material per tonne ( Rs. ) 125
200
Wages ( Rs. ) 28,000 10,000
Expenses ( Rs. ) 8,000 5,250
Output (tonnes) 830 780
Prepare Process Accounts, Abnormal Loss Account and Abnormal Gain Account.
Solution.
179
Cost and Management
Accounting
Working Notes:
Normal Gain
Value of Abnormal Gain = Units of Abnormal Gain
650 Rs.18, 000
Rs.1, 78,Output
Normal
15
45 90
900650
Rs.1, 60,
15 Rs.3,150
765
Dr. Abnormal Loss Account Cr.
Particulars Tonnes Amount Particulars Tonnes Amount
Rs. Rs.
To Process A 20 3,600 By Cash/ Bank A/c 20 1,600
By Costing 2,000
20 3,600 P&LA/c 20 3,600
180
Process Costing
Process B Account
Particulars Units Amount Particulars Units Amount
( Rs.) (Rs.)
To Process A 4,700 28,200 By Normal Loss 470 2,350
To Direct Wages 5,000 (10% of 4,700
To Direct Expanses 9,910 @Rs.5p.u.)
To overheads (5/8 x 16,000) 10,000 By Finished Stock 4,300 51,600
To Abnormal Gain 70 840
4,770 53,950 4,770 53,950
Illustration 5. The output of Process X was 5,000 units. Normal loss allowed was 10%
of input. Abnormal loss was 400 units. The following further information is obtained :
Material @ Rs. 5 per unit
Labour Rs. 8,000
Overheads Rs. 6,700
Wastage realized Rs. 2.50 per unit
Prepare Process X account and Abnormal Loss account.
5
Solution. (a) Let the input be x units
Normal loss = 10% of input
10 x
x units
100 10
Abnormal loss = 400 units
Output = 5,000 units
Now Input = Output + Abnormal loss + Normal loss
181
Cost and Management
Accounting
x
x = 5,000 + 400 +
10
x
x = 5,400 +
10
10 x= 54,000 + x
9x = 54,000
x = 6,000
Units introduced = 6,000
Process 'X' Account
Particulars Units Amount Particulars Units Amount
Rs. Rs.
To Materials 6,000 30,000 By Normal loss 600 1,500
@ Rs. 5 per unit @ Rs. 2.50 per unit
To Labour 8,000 By Abnormal loss 400 3,200
To overheads 6,700 By output 5,000 40,000
transferred
to process Y a/c:
@ Rs. 8 per unit
6,000 44,700 6,000 44,700
Working Notes:
Calculation of Normal Loss:
10 6000
Normal loss = 10% of 6,000 = = 600 Units.
100
Calculation of Value of abnormal loss
Total cost – Value of Normal Loss
= Units of Abnormal Loss
1500– Units of Normal Loss
Total Input
44700
400 Rs.3200
600 600
Abnormal Loss Account
Particulars Units Amount Particulars Unit Amount
(Rs.) (Rs.)
To Process 400 3200 By Sale of Scrap @250 per unit 400 1000
A/c
By Costing Profit & Loss A/c - 2200
400 3200 400 3200
Illustration 6: In a factory, the product passes through two processes, A and B. Loss of
5% is allowed in Process A and 2% in Process B. Nothing being realized by disposal of
the wastage. During April, 2015, 10,000 units of material costing Rs, 6 each were
introduced in Process A. The other costs were as follows :
Process A Process B
Rs. Rs.
Materials - 6,140
Labour 10,000 6,000
Overheads 6,000 4,600
182
Process Costing
The output was 9,300 units from Process A. The output of Process B was 9,200 units,
8,000 units of the finished product were sold.
Prepare Process Accounts and Abnormal Loss/Gain Accounts.
Rs.
Raw materials 6,000
Direct wages 5,000
Factory overheads 2,400
Opening stock of finished goods 800 [200 kg]
Closing stock of finished goods [400 kg]
Sale of finished product 20,000 [3,000 kg]
Advertising and Selling expenses 1,475
Profit desired is 30% on sales.
Solution:
Process 'A' Account
Particulars Units Amount Particulars Units Amount
Rs. Rs.
To Units introduced - @ 10,000 60,000 By Normal Loss 500 -
Rs. 6 per unit By Abnormal Loss 200 1600
To Labour 10,000 By Transfer to 9,300 74,400
To Overheads 6,000 Process 'B' A/c
10,000 76,000 10,000 76,000
183
Cost and Management
Accounting
Illustration 7: A product passes through two distinct processes A and B and then to
finished stock. The output of A passes direct to B and that of B passes to finished
product. From the following information you are required to, prepare Process Account.
Process A (Rs.) Process B (Rs.)
Material consumed. 12,000 6,000
Direct Labour 14,000 8,000
Manufacturing Expenses 4,000 4,000
Input in Process A (Units) 10000 units
Input in Process A (Value) 10,000
Output (Units) 9400 units 8300 units
Normal wastage . 5% 1 0% _
.(percentage of input)
Value of normal wastage 8 10
(per 100 units)
Process A Account
Particulars Units Rs. Particulars Units Rs.
To Input . 10,000 10,000 By Normal Wastage 500 40
To Material Consumers 12,000 By Abnormal Loss A/c 100 421
To Direct Labour 14,000 By Process B (Output 9,400 39,539
To Manufacturing 4000 transfer to next process)
Expenses
10,000 40,000 10,000 40,000
184
Process Costing
Process B Account
Particulars Units Rs. Particulars Units Rs.
To Process A Account 9,400 39,539 By Normal Wastage 940 94
To Material 6,000 By Abnormal Loss 160 1,086
To Direct Labour 8,000 By Finished Stock A/c 8,300 56,359
To Manufacturing 4,000 transfer to next process)
Expenses
9,400 57,539 9,400 57,539
Working Notes:
Rs.57539 Rs.94
Cost of Abnormal Loss = 160
9400 940
Illustration 8. D. Ltd. introduced 5,000 units in a process at a cost of Rs. 10,000. .'he
wages and overheads incurred are Rs. 10,000 and Rs. 8,000 respectively. It is expected
that 10% of the output is likely to be defective. Actual output of goods is 4,400 units. The
rectification of defective units costs Rs. 4 per unit. Calculate the cost per\unit and show
how will you deal with the cost of rectification of abnormal defective units.
Process Account
Units Amount Units Amount
(Rs.) (Rs.)
To Units introduced 5,000 10,000 By Abnormal Loss 100 600
To. Wages 10,000 By Finished Stock A/c
To Overheads 8,000 @ Rs. 6 4900 29,400
To Cost of Rectification — 2,000
(500 units @ Rs. 4)
5,000 30,000 5,000 30,000
Working Notes:
1 Rectification of normal defective units is an item of factory overheads. Hence 10% of
5,000 units, i.e., 500 units multiplied by Rs. 4, i.e., Rs. 2,000 has been added to the
cost.
2. Total Output = Actual output + Rectified units = 4,400 + 500 = 4,900 units
3. There is no normal loss
Normal Cost 30, 000
Cost per unit = Rs.6
Normal Output 5, 000
Normal Cost = Total cost Scrap Value of Normal Loss
= 30,000 0 = Rs. 30,000
Normal Output = Units introduced - Units of normal loss
= 5,000 - 0 = 5,000
4 Units of Abnormal loss = 5,000 4,900 = 100 units
Value of Abnormal Loss = Units of abnormal loss Cost per unit
= 100 6 = Rs. 600
185
Cost and Management
Accounting
5 Cost of rectification of abnormal defective units is debited tp the Costing Profit & Loss
Account.
Illustration 9 : A product passes through two process A and B. The normal wastage for
each process is as follows:
Process A = 3 Percent and Process B = 5 percent
Wastage of process A was sold at 25 paise per unit and that of process B of 50 paise per
unit. 10,000 units were introduced in process A @ Re 1 per unit.
Other expenses were :
Process A Process B
Material 1000 1500
Labour 5000 8000
Direct expenses 1050 1188
Actual output 9500 9100
Prepare (1) Process Account (ii) Normal Loss A/c, (iii) Abnormal Loss
A/c and (iv) Abnormal Gain A/c
Process A account
Amount Amount
Particulars Units (Rs.) Particulars Units (Rs.)
To Units introduced 10,000 10,000 By Normal Loss A/c
(10,000 units @ Re.l (Normal wastage)
Per unit) (3% of 10,000)
To Material 1,000 Units sold at 300 75
To Labour 5,000 25 paise per unit)
To Direct Expenses 1,050 By Abnormal Loss A/c 200 350
(Abnormal wastage)
By Process B (Output
transferred) 9,500 46,625
10,000 17,050 10,000 17,050
186
Process Costing
Rs.27, 075
75 Rs.225
9025
Abnormal Loss A/c
Amount Amount
Particulars Units (Rs.) Particulars Units (Rs.)
To Process A 200 350 By Sale wasted units :
Process A @ 25 paise 200 50
per unit
By costing profit 300
/ & loss A/c
200 350 200 350
Illustration 10: The product XYZ of a company passes through three distinct processes
before completion. From the past experience it is ascertained that wastage is incurred in
each process as under:
Process A 2%; Process B 5%; Process C 10%.
The wastage of Processes A and B is sold at Rs. 10 per unit and that of Process C at Rs.
80 per 100 units. The following is the information regarding the production as on 31"
March 2015 :
Process A Process B Process C
Materials 12,000 8,000 4,000
Direct Labour 16,000 12,000 6,000
Machine Expenses 2,000 2,000 3,000
187
Cost and Management
Accounting
188
Process Costing
189
Cost and Management
Accounting
Normal Cost
= Units of Abnormal Loss
Normal Output
Total Cost - Scrap Value of Normal Loss
= Units of Abnormal Loss
input Units - Units of Normal Loss
83,467 1,504 81,963
= 9209 920
18,800-1,880 16,920
= Rs. 4,457 (app.)
77,506
Rs.4.84 (app.)
16,000
Illustration 11. The output from Process A transferred to Process B was 2,500 units.
Normal Loss being 10% of input in Process A was 300 units. 200 units were reported to
be as abnormal loss. Material introduced @ Rs.5 per unit, labour cost Rs. 4,000 and
overheads Rs. 3,350 and normal loss realised Rs.2.50 per unit.
You are required to prepare: (i) Process Account A and (ii) Abnormal Loss Account
Solution: Input = Output + Normal Loss + Abnormal Loss
= 2,500 + 300 + 200 = 3,000 units '
OR
Normal Loss = 10% of input
= 300 units
300
Input = = 3,000 units
10%
Process 'A' Account
Particulars Units Amount Particulars Units Amount
(Rs.) (Rs.)
To Materials 3,000 By Normal Loss 300 750
@ 5 per unit 15,000 By Abnormal Loss 200 1600
To Labour Cost 4,000 By Transfer to Process 'B' 20,0003
To Overheads 3,350 Account 2,500
— — @Rs.8 per unit
3,000 22,350 3,000 22,350
190
Process Costing
20,000
Rs.8
2,500
Illustration 12: 600 units were introduced in Process X at Rs. 20 per unit. 500 units
were completed and transferred to process Y. The normal process loss was 20% of the
input and the scrap is sold at Rs. 3 per unit. The labour and overhead expenditure
incurred in the process amounted to Rs.600. You are required to show the Process
Account, Normal Loss Account and Abnormal Gain Account.
Particulars Units Amount Particulars Units Amount
(Rs.) (Rs.) (Rs.) (Rs.)
To Units introduced 600 12,000 By .Normal loss 120, 360
By Transfer to process
To labour and overhead 'Y' Account @ Rs.
expenditure . 600 25.50
To Abnormal Gain 20 510 per unit 500 12,750 ,
620 13,110 620 13,110
To Process 'X' Account 120 360 By Cash : Sale of .scrap 100 300
By Abnormal Gain 20 60
120 360 Account 120 360
Illustration 13. product passes through two processes A and B. Normal loss in process A
is 10% of the input and in process B it is 7.5% of the input. The scrap value in the process
A is 5 paisa per unit and in process B it is 10 paisa per unit. Other information is as
follows:
Process .
A B
Materials 5,000 2,500
Wages 5,000 3,000
Other expenses 2,500 1,073
l,000 units were introduced into process A at a cost of Rs. 5,000.
The outputs were : Process A — 8,400 units
Process B — 7,300 units
Prepare process cost accounts.
Solution: Process 'A' Account
Particulars Units Amount Particulars Units Amount
(Rs.) (Rs.) (Rs.) (Rs.)
To units introduced 10,000 5,000 By Normal loss 1,000 50
To Materials 5,000 (10% of 10,000)
To Wages 5,000 By Abnormal loss 600 1,200
To Other expenses 2,500 By Transfer to Process
'B' Account 8,400 16,800
@ Rs. 2 per unit
10,000 18,050 10,000 18,050
18,050 50
= 600
10,000 1,000
18,000
600 = Rs. 1,200
9,000
Process 'B' Account
Particulars Units Amount Particulars Units Amount
(Rs.) (Rs.) (Rs.) (Rs.)
T6 Transfer from By Normal loss 630 63
Process 'A' Account 8,400 16,800 (7.5% of 8,400)
To Materials 2,500 By Abnormal loss 470 1,410
To Wages 3,000 By Transfer to Finished
To Other Expenses 1,073 Stock Account @ Rs. 7,300 21,900
3
per unit
8,400 23,373 8,400 ^23,373
Illustration 14: From the following information relating to Process A, prepare Process A
Account, Abnormal Gain Account and Normal Loss Account.
Units introduced 840
Cost per unit Rs. 40
Material Introduced Rs. 5,924
Direct wages Rs. 8,000
Overheads Rs. 8,000
Actual output 750 units
Normal loss 15% of input
Value of scrap Rs. 10 per unit.
193
Cost and Management
Accounting
Normal Cost
Value of Abnormal Gain = Units of Abnormal Gain
Normal Output
Total Cost - Scrap Value of Normal Loss
= Units of Abnormal Gain
Units introduced - Units of Normal Loss
194
Process Costing
Since actual loss is only 90 units, only 90 units could be sold as scrap. Abnormal Gain
Account
Particulars Units Amt. Particulars Units Amt.
Illustration 15. The product of a manufacturing concern passes through two processes —
A and B. The normal losses and abnormal losses are defective units having a scrap value
of Rs. 2 and Rs. 5 per unit in processes A and B respectively. The following
information relates to the month ending 31-03-2010:
Process A Process B
Raw materials issued @ 3000 units -
Rs. 5 10% of input 5% of input
Normal loss 2800 units 2600 units
Output Rs.1,000 Rs. 780
Additional components Rs.4,000 Rs. 3,000
Direct wages Rs. 10,000 Rs. 14,000
Direct expenses 75% 125%
Production overheads
(as a percentage of direct
wages)
There was no opening or closing work in progress but opening and closing stocks of
finished goods were Rs. 20,000 and Rs.23,000 respectively. Prepare Process Accounts,
Finished Stock A/c, Normal Loss A/c, Abnormal Loss A/c and Abnormal Gain A/c.
Solution Process A Account
Particulars Units Ami. ( Rs. Particulars Units Amt. ( Rs.
) )
To Raw Materials 3000 15,000 By Normal Loss 300 600
To Additional 1,000 By transfer to Process B 2,800 33,600
Components 4,000 @ Rs. 12
To Direct Wages 10,000
To Direct Expenses 3,000
To Production 33,000
Overheads 1,200
100
3,100 34,200 3,100 34200
To Abnormal Gain
195
Cost and Management
Accounting
Working note:
Rs.33, 000 600 Rs.32, 400
Cost per unit = Rs.12
3000 300 units 2, 700 units
Process B Account
Particulars Units Amount Particulars Units Amount
(Rs.) (Rs.)
To Transfer from 2,800 33,600 By Normal Loss 140 700
Process A By Abnormal Loss @
To Additional Rs. 20.46* 60 1,228
Components 780 By transfer to Finished 2,600 53,202
To Direct Wages 3,000 Stock A/c
To Direct Expenses 14,000
To Production
Overheads 3,750
2,800 55,130 2,800 55,130
Working note:
Rs.55,130 700 Rs.54, 430
Cost per unit = Rs.20.46
2800 140 units 2, 660 units
Finished Stock Account
Particulars Rs. Particulars Rs.
To Balance b/d 20,000 By Cost of Sales By 50,202
To Process B A/c 53,202 Balance b/d 23,000
73,202 73,202
196
Process Costing
9.5 QUESTIONS
1. Distinguish between :
(i) Normal loss and abnormal
(ii) Job Costing andProcess
(iii) Abnormal Loss & Abnormal
2. Explain normal wastage and abnormal wastage and state how they should be
dealt with in Process Cost Accounts.
3. Calculate the cost of each process and the total cost where production per month
was 480 radios :
Process A Process B Process C
Materials 750 250 100
Wages 400 1,000 300
Factory Overheads 130 360 125
Indirect costs 7 425 should be apportioned on the basis of wages.
[B.Com. (Pass) Delhi, 1982, 1983, 1984 adapted]
[Ans. Cost Process A 7 1,380. Process B 7 3,240, Process C 7 3,840]
4. In a factory, a product passes through two processes. A and B. During the month
of March 2012, certain number of units costing 7 10 per unit were introduced in
process A. The other costs were 7 2850. 855 units were completed and
transferred to process B at the end of the month and nothing was in process
Assuming that a normal loss of 10% of input is allowed in process A and realises
7 4 per unit; and assuming that there was no abnormal loss or gain in the process.
(i) State the number of units introduced in the beginning of the process, and
(ii) Complete Process^ Account, [B.Com. (Pass) Delhi]
[Ans. 950 units, Cost of output transferred 7 11,970]
5. Find out the number of units initially introduced in Process /when Normal
Wastage is 10%, 5% and 8% of the input respectively. There are 3,933 units of
Finished Stock. There was no Abnormal Loss of Effectives in any of the
processes. [B.Com. (Pass), Delhi, 1997]
[Ans. 5,000 units]
6. 600 kg of material was charged to process A @ Rs. 4 per kg. The direct labour
accounted for Rs. 200 and the other departmental expenses to Rs. 760. The
normal loss is 10% of input and the net production was 500 kg. Assuming that
the process scrap itself is saleable at Rs. 2 per kg, prepare Process A Account
clearly showing the value of normal and abnormal loss. Also prepare normal and
abnormal loss account. [B.Com. (Pass), Delhi, 2009]
197
Cost and Management
Accounting
Process I Process II
Rs. Rs.
Indirect Material 10.000 13,000
Direct Labour 10,000 13,100
Production overheads 20,000 10,000
% Normal Loss (of input) 5 5
Scrap Value per unit 5 5
Output (Units) 19.000 18,800
20,000 units costing Rs. 60,000 are introduced in Process I.
[B. Com. (P'ass), Delhi, 1997]
[Ans. Cost of Process I 19,000 units of Rs. 95,000; Cost of Process II 18,800
units of
Rs. 1,31,600 Cost of Abnormal Effectives 750 units of Rs. 5,250.]
8. The output from process X transferred to process Y was 2,500 units. Normal
Loss was 10% of input in process A"and was 200 units were reported to be as
abnormal loss. The other information is given below :
Materials introduced @ Rs. 5 per unit. Labour Cost Rs. 4,000 and Overheads
Rs. 3,350 and normal loss realised Rs. 2.50 per unit.
You are required to prepare :
(i) Process A' Account
(ii) Abnormal Loss Account.
[B. Com. (Pass) Delhi, 1986 & 1987, adapted]
[Ans. (i) 2,500 units of Rs. 20.000; (ii) Net Abnormal Loss Rs. 1,100].
9. The output of Process A" was 2,500 units. Normal loss allowed was 10% of
input. Abnormal loss was 200 units.
Material @ Rs. 5 per unit
Labour Rs. 4,000
Overheads Rs. 3,350
Wastage realised Rs. 2.50 per unit
You are required to prepare Process X Account and Abnormal Loss Account.
[B. Com. (Pass) Delhi, 1996]
[Ans. Cost of Process A Rs. 2,500 units of Rs. 20,000 Net Abnormal Loss Rs.
1,100]
10. 2,000 units costing Rs. 4 per unit were introduced to Process /. Labour Costs
and other expenses were Rs. 1,080 and Rs. 120 respectively. Its output was 1,900
units. The Normal Scrap was 10% of the input and had a realisable value of Re. 1
per unit.
Prepare Process / Account, Normal Loss Account and Abnormal Gain Account.
[B.Com. (Pass) Delhi, 1994]
198
Process Costing
[Ans. Cost of Process I 1,900 units of Rs. 9,500 Net Abnormal Gain Rs. 400]
SUGGESTED REFERENCE
Mittal & Maheswari, Elements of Cost Accounting, Shree Mahavir Book Depot
(Publishers) 2015.
Varshney J.C., Principles and Practice of Cost Accounting, Wisdom Publication
House (2009).
Mittal & Maheswari, Management Accounting, Mahavir Publication, 2015.
Arora M.N. Cost Accounting, Vikas Publishing House Pvt. Ltd. 2013.
Jain & Narang, Cost Accounting, Kalyani Publishers 1998.
Arora M.N., Management Accounting, Himalaya Publishing House, 2006.
199
BLOCK-4
Methods of Costing
The present block refers to the details concepts of operating as well as service
costing and reconciliation of cost and financial accounts. Further, the learner will
have the opportunity to learn about the basic of budgetary control. How the
budgets of an organisation prepared and managed. The present block refers the
following unit;
Unit 10: Operating or Service Costing
200
UNIT-10 OPERATING OR SERVICE COSTING
LEARNING OBJECTIVES
STRUCTURE
10.1 Introduction
10.2 Application of operating costing
10.3 Transport costing
10.4 Operating cost sheet
10.5 Classification of Operating Cost
10.6 Questions
10.1 INTRODUCTION
Definitions
According to ICWA, "Operating cost is, "the cost incurred in conditions a business
activity. Operating cost 5refers to the cost of undertakings which do not manufacture any
product but which provide services.
CIMA London, defines Operating Costing as "that form of operation costing which
applies where standardised services are rendered either by an undertaking or by a service
cost centre within an undertaking."
The costs incurred in providing a service are called 'Operating costs' and the method used
for computing such costs is called 'Operating Costing'.
201
Cost and Management
Accounting
(i) It deals with determination of cost of repetitive services, not tangible products.
(ii) It resembles with unit costing in that the total cost incurred during a period in a
service divided by the total number cost units of the service gives cost per unit of
service,
(iii) Services are standardised.
(iv) Investment in fixed assets is high and in working capital is low.
(v) Major portion of the total cost is fixed cost. Hence, the cost per unit of service
rendered is affected by the economies and scale of operations,
(vi) Various costs in providing services may be categorised as :
(i) Standing changes and Maintenance charges.
Operating Costing is applied in those undertakings which are engaged in providing services
rather than manufacturing of tangible products. It is generally applied in : (a) Road
Transport, Railways, Airlines, (b) Hotels, (c) Hospitals, (d) Electricity Supply Companies,
(e) Water Supply Companies, (f) Gas Supply Companies, (g) Cinema, (h) Canteen
Cost unit: There are large varieties of services and therefore, there are different cost
units for different services. The unit for which the operating cost is to be computed
should be decided after considering all the technical and other factors affecting the
operating cost. The cost unit may be of two types as follows :
(i) Simple cost unit: A cost unit is said to be simple cost unit when it involves one
characteristic, for example :
202
Operating or
Service Costing
(ii) Composite cost unit : A cost unit is said to be the composite cost unit when it reflect
two or more characteristic, for example :
Undertaking Cost Unit
Passenger Transport per passenger-km
Goods Transport per tonne-km
Hotel per room per day
Hospital per bed per day
Electricity per kilowatt hour
In short, all the variable charges for running the vehicle are included in this group.
Normally, the life of a vehicle is given in terms of mileage to be run. Accordingly,
depreciation is to be allocated on the basis of mileage run and hence should be treated as
'variable expense'. A clear-cut classification of the various items of costs into the above
three groups should not attempted to be made as a matter of rule. It should be, rather,
based upon. circumstances of each case. For example, the wages payable to a driver by a
private owner on a 'no work no pay' basis should be always treated as an item of
203
Cost and Management
Accounting
'operating or variable expenses'. But in case of employees of, say, a State Transport
Corporation, who are employed on a permanent basis, same item of cost should be treated
as part of standing or fixed charges'.
3. Ascertainment of costs : Total fixed, maintenance and operating costs are collected
under respective heads and these are then divided by total units e.g. ton-miles or
passenger miles to arive at average unit cost. A performa of cost sheet is given below :
204
Operating or
Service Costing
The classification of various items of costs into the above three groups should not be
attempted as a matter of rule. It depends basically on the circumstances of each case. For
example, the wages payable to a driver by a private owner on a no work no pay basis
should be treated as an item of operating or variable expenses. But in case of employees
of State Transport Corporation and employed on permanent basis, the same item of cost
should be treated as pan of standing or fixed charges.
Illustration 1: From the following information calculate total kilometer and total
passengers kilometers
No. of Buses = 6
Days operated ion the month = 25
Trips By Each Bus = 4
Distance of rate 20 kms one way.
Capacity of Bus = 40 passengers
Normal passenger travelling 90% of the capacity.
Commercial (simple average) tonnes kms – commercial tonnes kms are arrived at by
multiplying total distances kilometers by average load quantity.
205
Cost and Management
Accounting
Illustration 2: A lorry starts with a load of 30 tonnes of goods from Station A. It unloads
8 tonnees at station B and rest of goods at station C. It reaches back directly to station A
after getting reloaded with 16 tonnes of goods at station C. The distance between A to B,
B to C and then from C to A are 100 kms, 120 kms and 160 kms, respectively. Compute
Absolute tonnes kms and commercial tonnes kms.
Solution: Absolute tonnes kms = 30 tonnes 100 kms + 12 tonnes 120 kms + 16
tonnees 160 kms = 7000 tonnees kms.
Commercial tonnees kms = Average load Total Kilo metres travelled
30 12 16)
tonnes 380 kms
3
58 22040
380 7346 tonnes kms.
3 3
Illustration 3: From the following data, calculate the cost per kilometer of a vehicle.
Value of Vehicle = Rs. 15,00,000
Road licence fee per year = 500
Insurance charges per year = 100
Garage rent per year = 600
Driver wages per month = 200
Cost of petrol per litre 3.60
Kilometers per litre 8
Proportionate charges for tyre and maintenance per km.
Estimated life 1,50,000 km.
Estimated annual kilometreage— 6,000 km.
Ignore interest on capital
206
Operating or
Service Costing
Illustration 4: Mr. Sharma is running a taxi for which the following information is given.
You are required to compute the rate per kilometre to be charged to earn a profit of 20% cash:
Solution:
Particulars Per Annum Per km
Standing charges :
Road Licence 7,500
Insurance 7,000
Garage Rent 6,000
Supervision and Salaries 12,000
5
Interest 5,00,000
100
25,000
Total Standing Changes
47,500
Standing Charges per km.
47,500
10, 000 4.75
Variable Charges :
5, 00, 000
Depreciation
10, 000 5.00
Repairs and Maintenance 1.75
Cost of fuel
30
20 1.50
Tyre allocation 0.70
Driver's wages
30 3.00
10
Operating cost per km. 17.70
207
Cost and Management
Accounting
Working note:
Profit (20% on cost)
Operating cost per km = Rs. 17.70
Profit (20% on cost) = Rs. 3.54
Rate to be charged = Rs. 21.24
Profit (20% on rate / fare)
Operating cost per km= Rs. 17.70
Profit (20% on Rate or 25% on cost) = 4.43
Rate to be charged 22.13
Illustration 5. Ganeev Transport Service Company Ltd. is running 4 buses between two
towns which are 50 km apart. Seating capacity of each bus is 40 passengers. The
following particulars were obtained from their books as on 31" March 2015:
Actual passengers carried were 75% of the seating capacity. All the four buses ran on all
the days of the Month (30 days). Each bus made one round trip per day. Find out the cost
per passenger km.
Solution:
Number of passenger kilometers
= Number of buses x Distance x capacity of each bus x Actual capacity
utilised x Round trip x Number of days
= 4x 50 x 40 x 75% x 2 x 30
75
= 4 x 50 x 40 x x 2 x 30 = 3,60,000
100
208
Operating or
Service Costing
Illustration 7: Varun Ltd. is running four buses between Delhi and Alwar, covering a
distance of 100 kms. The seating capacity of each bus is 40 passengers. The following
particulars are obtained from its Books for the month of Oct. 15 :
Rs.
Wages of drivers, conductors 9,600
Salaries of office staff 3,000
Honorarium of accountant 1,000
Diesel, oil etc. 16,000
Repairs and maintenance 3,200
Road tax and insurance 6,400
Depreciation 10,400
Interest and other charges 8,000
Actual passengers carried were 75% of the seating capacity. All the buses ran for 30
days. Each bus made one round trip per day. Find-out the fare the company should charge
per passenger/km, if it wants a profit of 20% on the taking.
75
Solution: Effective Passenger kms per bus = 2 100 30 40
100
= 1,80,000 passengers kms
Operating Cost Sheet for Four Buses for Delhi-Alwar
Particulars Rs.
Fixed Costs:
Wages of drivers, conductors 9,600
Salaries of office staff 3,000
Honorarium of accountant 1,000
Repairs and maintenance 3,200
Road tax and Insurance 6,400
Depreciation 10,400
Interest and other charges 8,000
Total Fixed cost for 4 buses 41,600
Variable Costs :
Diesel, Oil etc. for 4 buses 16000
Total Cost for 4 buses 57600
Profit 20% of taking i.e. 25% of cost 1,4400
Total fair collection from 4 buses 72000
72, 000
Total fair from one bus Rs. Rs.18, 000
4
Fair per passenger km = Rs. 18,000 = 1,80,000 = 10 paise.
210
Operating or
Service Costing
Illustration 8: The Madras Transport Company which keeps a fleet of lorries, shows the
following information :
Kilometres run for April 2015—30,000
Rs.
Wages for April 2015 2,000
Petrol, oil, etc, for April 2015 4,000
Original cost of vehicles 1,00,000
Repairs for April 2015 6,000
Garage rent etc. for April 2015 1,000
Licence, insurance etc. for the year 2015 6,000
Depreciation is to be allowed @ 25% per annum on original cost. Prepare a statement for
April 2015 showing the cost per running kilometre.
Solution:
Madras Transport Company
Operating Cost statement for the month of April, 2015
Kilometers run = 30,000
Particulars Per Month (Rs.) Per Km.
(Rs.)
Rs.
Fixed Costs
Wages Garage Rent 2,000
Licence, insurance etc. 1,000
(6000 + 12) 500 3,500 0.117
Variable or Running Charges :
Petrol, oil etc. 4,000
Repairs 6,000
Depreciation 2,083
25 1
1, 00, 000 12,083 0.403
100 12
15,583 0.520
The truck carries goods to and from city covering a distance of 50 miles each way. While
going to the city, freight is available to the extent of full capacity and on return 20% of
capacity.
Assuming that the truck runs on an average 25 days a month, work out: Operating cost
per tonne mile.
Illustration 10. A transport operator runs 4 buses between 2 cities, the distance between
the two being 100 kms. Each bus has a seating capacity of 50 and actual passengers
carried were 75% of the capacity. Each business takes 1 round trip every day and 30 days
in the month. Other details of the month:
Wages of drivers, conductors, cleaners Rs. 36,000
Salaries of office staff Rs. 14,000
Diesel, oil etc. Rs. 56,000
Repairs and maintenance Rs. 10,000
Road-tax and insurance Rs. 21,000
Cost of each bus Rs. 3,50,000
(charge depreciation @ 20% p.a.)
Interest and other charges Rs. 28,000
Calculate cost per passenger km.
212
Operating or
Service Costing
Illustration 11. A person owns a bus which runs between Delhi and Chandigarh and
back, for 10 days in a month. The distance from Delhi Chandigarh is 240 kms. The bus
completes the trip from Delhi to Chandigarh and back in the same day. The bus goes
another 10 days month to Agra and the distance covered being 200 kms. The trip also
completed in the same day. For the rest of 4 days it runs in the Delhi city. Daily distance
covered in local city is 60 kms. Calculate the amount person should charge from
passenger when he wants to earn profit of 33% on his takings. The other particulars are
given be:
Cost of bus Rs. 2,00,000.
Depreciation 20% per annum.
Salary of driver Rs. 1,600 per month.
Salary of conductor Rs. 1,500 per month.
Salary of part time accountant Rs. 400 per month.
Insurance Rs. 2,000 per annum.
Diesel consumption : 6 kms. per litre costing Rs. 4.00 per litre.
213
Cost and Management
Accounting
The bus is generally occupied 90% of the capacity when it goes to Chandigarh and 80%
when it goes to Agra. It is always full when it runs within the city.
Solution:
Delhi to Chandigarh
Selling Capacity = 50 passengers
Actual passengers carried = 50 x 90% = 45
Actual passengers carried for 10 days = 45 x 10 = 450 ,
Delhi to Agra
Actual passengers carried = 50 x 80% = 40
Actual passenger for 10 days = 40 x 10 = 400
Actual passenger kms. = 400 x 200 x 2 = 1,60,000
Local City
Actual passenger carried = 50
Actual passenger for 4 days = 50 x 4 = 200
Actual passenger kms. = 200 x 600 = 12,000
Total passenger kms. = 2,16,000 + 1,60,000 +
12,000
= 3,88,000
Total Kms. travelled in a month
Delhi to Chandigarh (240 x 2 x 10) = 4,800 kms
Delhi to Agra (200 x 2 x 10) = 4,000 kms
Local (60x4) = 240 kms = 9, 040 kms
214
Operating or
Service Costing
Illustration 12. Union Transport Company supplies the following details in respect of a
truck of 5-tonne capacity
Cost of truck Rs. 90,000
Estimated life 10 years
Diesel, oil, grease Rs. 15 per trip each way
Repairs and maintenance Rs. 500 per month
Driver's wages Rs. 500 per month
Cleaner's wages Rs. 250 per month
Insurance Rs. 4,800 per year
Tax Rs. 2,400 per year
General supervision charges Rs. 4,800 per year
The truck carries goods to and from city covering a distance of 50 miles each way.
While going to the city, freight is available to the extent of full capacity and on return
20% of capacity.
Assuming that the truck runs on an average 25 days a month, work out-
(i) Operating cost per tonne-mile, and
(ii) Rate per tonne per trip that the company should charge if profit of
50% on freightage is to be earned.
215
Cost and Management
Accounting
Illustration 13: A toy manutacturer earns an average net profit of Rs. 3 per piece at a
selling price of Rs. 15 by producing and selling 60,000 pieces at 60% of capacity.
Composition of his cost of sales is :
Rs. per Piece
Direct Material 4.00
Direct Wages 1.00
Work Overheads 6.00 (50% fixed)
Sales Overheads 1.00 (25% varying)
During the current year he intend to produce the same number but anticipated that: (a) his
fixed charges will go up by 10%; (b) rates of direct labour will increase by 20%; (c) rates
of direct material will increase by 5%; (d) selling price cannot be increased.
Under these circumstances he obtains an order for a further 20% of his capacity. What
minimum price will you recommend for accepting the order to ensure the manufacturer
an overall profit of Rs. 1,80,500 Rs.
216
Operating or
Service Costing
Solution
Operating Cost Statement
(Annual mileage 6,000 miles)
Annual Cost, Cost per mile
Rs. Rs.
Fixed or Standing Charges :
Road licence fee 500
Insurance charge 100
Garage rent 600
Driver's wages per year 2,400
Total/per mile fixed cost 3,600
Variable or Running Charges : 0.60
Budget for
Particulars Previous Year Current rear
Per Piece Amount Per Piece Amount
Rs. Rs.
Sales (A) 15.00 9,00,000 15.00 9,00,000
Less : Marginal Cost Direct 4.00 2,40,000 4.20 2,52,000
material
Direct Wages 1.00 60,000 1.20 72,000
Variable Works Overheads 3.00 1,80,000 3.00 1,80,000
Variable Sales Overheads 0.25 15,000 0.25 15,000
Total Marginal Cost (B) 8.25 4,95,000 8.65 5,19,000
Contribution (A - B) 6.75 4,05,000 6.35 3,81,000
Less : Fixed Costs
Works Overheads 80,000 1,98,000
Sales Overheads 45,000 2,25,000 49,500 2,47,500
Profit 1,80,000 1,33,500
217
Cost and Management
Accounting
Charge interest at 5% per annum on cost of vehicles. The vehicles run 20 Km. per hour
on an average.
Operating Cost Sheet
Particulars Vehicle A Rs. Vehicle B
Rs.
A. Fixed Cost per annum :
Road licence 750 750
Insurance 700 400
Garage rent 600 500
Supervisory salaries 1,200 1,200
Interest 1,250 750
Total Fixed Cost 4,500 3,600,
Km. run per annum 15,000 6,000
Fixed cost per Km. 0.30 0.60
B. Variable Cost per Km.
Drivers' wages (Rs. 3 per hour for (20 Km.) 0.15 0.15
Fuel cost per Km. 0.15 0.20
Repairs & maintenance 1.65 2.00
Tyre allocation 0.80 0.60
Depreciation (cost + estimated life) 0.25 0.20
Variable cost per km. 3.00 3.15
Total Cost per Running Km. (A+B) 3.30 3.75
218
Operating or
Service Costing
Illustration 15. SR Airlines has been permitted to operate three flights per week,
between A and B cities (both sides). The Airline operates a single aircraft of 160 seating
capacity. The normal occupancy is estimated at 60% throughout the year of 52 weeks.
The one way fare is Rs. 7,200. The cost of operation of flights are:
219
Cost and Management
Accounting
Rs.
fixed Costs: . Per flight
As per Statement (z) — (A) 4,22,000
Variable Cost:
Fuel Cost 76,000
Crew charges 24,000
Food served (Rs. 125 120) 15,000
Commission (Rs. 6,500 5% 120) 39,000
Total (B) 1,54,000
Total Operating Cost (A + B) 5,76,000
Gross Sales (Rs. 6,500 x 120) 7,80,000
Net Operating Income (Gross Sales Total Cost) 2,04,000
Rs.5, 76, 000 Rs.4,800
Operating Cost per passenger per flight =
120
Illustration 16. A Transport Co. charges Rs. 120 per ton for a 5 tons lorry load from A
station to B station. The charges for return trip are Rs. 110 per ton. In the month of July,
2015, a truck has made 10 outward journeys with full load out of which 3 tons were
unloaded twice at C station on the way. It returned without any load once only from C
station to A station. The expenses incurred were :
Annual fixed charges : Rs. 38,400
Annual maintenance : Rs. 19,200
Monthly running charges : Rs. 2,404
You are required to find the cost per ton-kilometre (absolute) and the profit are the month
of July, 2015, assuming that no concession is made for delivery t the intermediate
stations. Distance from A station to B station is 210 kms, and from A to C station 120 ms.
The truck carried a load of 8 tons 5 times in the month while returning on B station but
was once caught by the police and was fined Rs. 2,000.
Solution:
(i) Calculation of tonne km
Outward Journey Tonne Rs.
A to C (120 km x 5 tons x 2 times) 1,200
220
Operating or
Service Costing
Income Rs.
Outwards (10 trips x 5 tons x Rs. 120) 6,000
Return (5 trips x 8 tons x Rs. 110) 4,400
Return (5 trips x 5 tons x Rs. 110) 2,750
Total 13,150
Less: Fine 2,000
Net Receipts 11,150
Cost
Per annun
Rs.
Fixed charges 38,400
Maintenance 19,200
Total 57,600
Charges per months (57,600/12 months) 4,800
Running charges 2,404
Total 7,204
Cost per ton km Rs. 7,204/23,010 0.31
Total Receipts for the month of July 11,150
Less: Cost for the month of July 7,204
Profit 3,946
Illustration 17. A. transport company has been given a 20 km long route to run a bus.
The bus costs Rs. 50,000 and has been insured @ 6% p.a. while annual taxes amount to
Rs. 2,000. Garage rent is Rs. 100 p.m. Yearly repairs will be Rs. 2,000, • and the bus is
likely to last for 5 years. The driver's salary will be Rs. 3,000 p.a. and that of conductor's
Rs. 1,800 p.a. is in addition to 10% of the takings as commission (to be shared by the
driver and the conductor equally). Cost of stationery will be Rs. 600 p.a., Manager's
salary is Rs. 400 p.m. who also looks after accounts.
Diesel and oil will be Rs.25 per 100 km. The bus will make 3 round trips arriving on the
average 40 passengers on each trip. Assume 25% profit, calculate the bus fare to be
charged from each passenger. The bus runs on an average 25 days in a month.
Solution:
Operating Cost Statement
Standing Charges: (per Annum)
Taxes 2,000
Insurance (6% of 7 50,000) 3,000
Garage rent (Rs. 100 x 12) 1,200
Repairs 2,000
Salary of driver 3,000
Salary of conductor 1,800
Stationery 600
Salary of Manager (Rs. 400 x 12) 4,800
221
Cost and Management
Accounting
Total 18,400
Standing Charges p.m. (Rs. 18,400/12) 1,533.33
Depreciation 833.34
Diesel and Oil 750.00
Total Cost before Commission 3,116.67
Commission 10% of Takings 479.48
Total Cost 3596.15
Profit (25% of Takings) 1198.72
Takings 4,794.87
Rs.4, 7947
Fare per passenger = 20 km Rs.0.80 (approx.)
1,20,000
Working note
50, 000
Depreciation Rs.833.34
5 years 12 months
25
Diesel and Oil = 20 km 3 2 trips 25 days Rs.750
100
Calculation of Commission and Profit:
Suppose Takings = x
10x 25x
Commission ; and Profit
100 100
10x 25x 10x 25x
x 3116.67 or x 3116.67
100 100 100 100
65 100
or x 3116.67 or x(Takings) 3116.67 4, 794.87
100 65
x(Takings) 4,794.87
Total kms travelled by passengers = 20 3 2 25 40 = 1,20,000 kms
Illustration 18. Global Transport Ltd. charges Rs. 90 per tonne for its 6 tonnes lorry load
from city 'A' to city 'B'. The charges for the return journey are Rs. 84 per tonne. No
concession or reduction in these rates is made for any delivery of goods at the
intermediate station 'C'. In January 2015, the truck made 12 outward journeys for city 'B'
with full load out of which 2 tonnes were unloaded twice in the way at city 'C. The truck
carried a load of 8 tonnes in its return journey for 5 times but was once caught by the
police and Rs. 1,200 was paid as fine. For the remaining trips the truck carried full load
out of which all the goods on load were unloaded once at city 'C'. The distance from city
'A' to city 'C' and city 'B' are 140 kms and 300 kms respectively. Annual fixed costs and
maintenance charges are Rs. 60,000 and Rs. 12,000 respectively. Running charges spent
during January 2015 are Rs. 2,944. You are required to find out the cost per absolute
tonne-km and the profit for January 2015.
222
Operating or
Service Costing
Illustration 19. [B.Com. (Delhi)]: A city municipality arranges for the removal of its
gabrage by means oif motor vehicle transport. The following vehicles are maintained:
No. of Vehicles Specification
30 5 tonne lorries
40 3 tonne lorries
50 2 tonne lorries
20 4 tonne lorries
On an average each lorry makes 5 trips a day and each trip covrs an average distance of 6
kms. Each lorry carries garbage of 50% of its capacity. On an annual average, 10% of the
lorries are laid up for repairs every day. he conservancy work is carried out daily.
Calculate tonne-ms per month
Solution: (a) Tonne Capacity
30 Vehicles 5 tonnes 150
40 Vehicles 3 tonnes 120
50 Vehicles 2 tonnes 100
223
Cost and Management
Accounting
20 Vehicles 4 tonnes 80
450
Calculate of tonnee kms
50 90
450 5 6 30 1,82, 250 tonne kms
100 100
10.6 QUESTIONS
3. The more kilometer you travel with your own veicle, the cheaper it becomes.
Comment.
4. A bus started from Delhi for Mussoorie with 50 passengers on board. 20 passengers
got off at Dehradun and the bus proceeded with the remaining passengers. In the
evening the same bus left Mussoorie with 50 passengers, 10 passengers got off at
Dehradun and the bus resumed its journey with remaining passengers for Delhi. The
distance between Delhi and Dehradun is 280 km. and between Dehradun to Mussoori
it is 20 km. Compute the cost per passenger km., if the total cost of running the bus
comes out to be Rs. 5,000.
[B. Com. (Hons.) Delhi] [Ans. Cost per passenger km. : 19 paise]
5. A transport operator runs 4 buses between 2 cities, the distance between the two
being 100 kms. Each bus has a seating capacity of 50 and actual passengers carried
were 75% of the capacity. Each bus makes 1 round trip every day and 30 days in the
month. Other details for the month :
Wages of drivers, conductors, cleaners Rs. 36,000
Salaries of office staff Rs. 14,000
Diesel, oil etc. Rs. 56,000
Repairs and maintenance Rs. 10,000
Road-tax and insurance Rs. 21,000
Cost of each bus Rs. 3,50,000
(charge depreciation @ 20% p.a.)
Interest and other charges Rs. 28,000
Calculate cost per passenger km. [B.Com. Delhi] [Ans. Rs. 0.21]
6. The Madras Transport Company which keeps a fleet of lorries, shows the following
information : Kilometres run for April 2008-30.000.
Wages for Apr! 12008 2,000
Petrol, oil, etc. for April 2008 4,000
Original cost of vehicles 1,00,000
224
Operating or
Service Costing
SUGGESTED REFERENCE
Mittal & Maheswari, Elements of Cost Accounting, Shree Mahavir Book Depot
(Publishers) 2015.
Varshney J.C., Principles and Practice of Cost Accounting, Wisdom Publication
House (2009).
Mittal & Maheswari, Management Accounting, Mahavir Publication, 2015.
Arora M.N. Cost Accounting, Vikas Publishing House Pvt. Ltd. 2013.
Jain & Narang, Cost Accounting, Kalyani Publishers 1998.
Arora M.N., Management Accounting, Himalaya Publishing House, 2006.
225
UNIT-11 RECONCILIATION OF COST AND FINANCIAL ACCOUNTS
LEARNING OBJECTIVES
STRUCTURE
11.1. Introduction
11.2. integrated system of accounts
11.3 need for reconciliation
11.4 Reason for the difference between cost and financial accounts
11.5 Method of reconciliation
11.6 Questions
11.1 INTRODUCTION
226
Reconciliation of
Cost and Financial
Accounts
Under this accounting system, both the cost accounts as well as financial accounts are
maintained in one and the same set of books. It means ‘the merger or integration of both
financial and cost accounts’, thus maintaining only one integrated ledger containing both
financial as well as costing records. Otherwise, in non-integrated accounting system, cost
accounts and financial accounts are kept separately. Under this system, however, certain
inter-locking accounts may be maintained so as to ensure integration. This is know as
‘interlocking’ of the two accounts.
Under integrated accounting method, there is no need to recouncile the results of cost
accounts with those of the financial accounts. It is so because in case of ‘integrated
accounting system; both accounts are treated as part of a single comprehensive
accounting method.
If financial and cost records have been integrated into a single set of there would not be
any need to reconcile them because there will be only profit loss figure. There are so
many transitions and data will be similar in both the sets but there will be differences due
to the differences in objectives and assumptions. Hence there is need of reconciliation of
profit under financial and cost accounting.
Management is enable to know the reasons for the difference in results of both
cost and financial accounts.
Reconciliation explains reasons for difference which facilitate internal control.
Reconciliation ensure the reliability of cost data.
Reconciliation ensurn management leeside meaning.
The various reasons which create difference between cost and financial profit or loss
shown by the two set of books may be listed under the following heads.
(i) Items included in financial accounts and not in cost accounts.
(ii) Items shown only in cost accounts.
(iii) Absorption of over head.
(iv) Method of stock valuation.
(v) Method of depreciation.
Differences in cost and financial accounts arise because of the following reasons:
There are a number of items which are included in financial accounts but find no place in
cost accounts. All such items of expenditure will reduce the financial profit for the year
while any item of income will increase the financial profit. Under this head the following
items are included:
228
Reconciliation of
Cost and Financial
Accounts
There are certain items which are included in cost accounts but not in financial accounts.
All expenditures incurred, whether for cash or credit pass through financial accounting
system, so the type of entry, which can appear only in cost accounts, is a normal charge.
The following two items are examples of this type:
(i) Interest on capital: only that part of the interest in charged which is not
actually paid but charged in cost accounts.
(ii) Charge in lieu of Rent: in case premises are owned by the firm, a nominee
charge in lieu of rent is made in cost accounts but not in financial accounts.
Overheads are recovered on the base of pre-determined rates i.e., estimates like
percentage on direct materials, percentage on direct wages etc so that the amount
recovered and the amount actually incurred will invariably disagree. If overheads are not
fully recovered which means that the amount of overheads absorbed (in cost accounts), is
less than the actual amount, the shortfall is called as under-recovery. On the other hand, if
overhead expenses recovered in cost accounts are more than actual, it is called as over
recovery. Thus, over or under-recovery of overheads leads to difference in cost accounts
and financial accounts.
Basis of stock valuation in cost accounts and financial accounts and financial accounts
may be different leading to a difference in the profits. In cost accounts, stock will be
valued at FIFO, LIFO or average stock etc. but in financial accounts the principle is cost
or market price, whichever is less. However, sometimes in both sets of books different
bases of depreciation may be followed.
Different methods of providing depreciation adopted in two sets of books may also lead
to some difference in the profit or loss figures. For example in cost accounts machine-
hour rate method, or replacement cost method may be used whereas in financial accounts
straight line method or diminishing balance method may be used. This will lead to a
difference in profits.
Different items of abnormal wastages, losses or gain which are included in financial
accounts but are not included in cost accounts. Thus, the figure of abnormal losses and
gains may affect the result of financial accounts.
229
Cost and Management
Accounting
Methods of reconciliation:
For reconciling the profit or loss as disclosed by the financial accounting with that shown
by the cost accounting, a reconciliation statement or memorandum of reconciliation
account is prepared. The following steps have to be taken for preparation of
reconciliation statement.
The following table will also help to prepared the reconciliation of cost and financial
accounts treatment of cause for differences
Serial Reason for difference Suitable Suitable
No adjustment adjustment
Base is base is
costing profit financial
or financial profit or
loss(+) or (-) costing loss
(+) or (-)
1 Over absorption of overhead in cost account Add (+) Less (-)
2 Over valuation of closing stock in financial Add (+) Less (-)
account
3 Over valuation of opening stock in cost Add (+) Less (-)
account
4 Excess provision for depreciation of building, Add (+) Less (-)
plants & machinery etc
Charged in cost account
5 Items of expenses charged in cost account but Add (+) Less (-)
not in financial accounts (Interest on capital,
Rent in premises etc)
6 Items of income recorded in financial Add (+) Less (-)
accounts but not in cost account
7 Under absorption of overhead in financial Less (-) Add (+)
account
8 Over valuation of openings stock in financial Less (-) Add (+)
account
9 Over valuation of closing stock in cost Less (-) Add (+)
account
10 Item of income tax, dividend paid, Less (-) Add (+)
preliminary expenses written off,
underwriting commission and debenture
discount written off and any appropriation of
profit included in financial account any
231
Cost and Management
Accounting
It is only a memorandum account and does not form part of the double entry books of
account.
Types of problems:
You are required to prepare a reconciliation of cost of cost and financial account from the
following structures.
(i) When profit or loss of financial account and cost account are given
232
Reconciliation of
Cost and Financial
Accounts
Illustration 2: MS Rana trader maintain separate cost bonus which discloser a profit Rs.
60,228 for the year ending March 31, 2014. The net profits disclosed by financial
accounts amounted to Rs. 39,520. Upon enquiry, it is found that:
(i) The company made a provision of Rs. 1,200 for bad debts.
(ii) Overheads charged to production in cost books were Rs. 15,000, whereas
actual overhead expenses amounted to Rs. 13,864.
(iii) Directors were paid fee amounting to Rs. 1,500.
(iv) Installation of a new plant involved an expenditure of Rs. 24,000 but in had
not gone into production as yet. Depreciation @ 5% was provided on the cost
of the plant.
233
Cost and Management
Accounting
(v) The company received interest on bank deposit amounting to Rs. 56.
(vi) It paid income tax Rs. 18,000.
Illustration 3: the following figures are available from financial accounts for the
year ended 31st March, 2015:
Rs
Direct material consumption 2,50,000
Direct wages 1,00,000
Factory overheads 3,80,000
Administration overheads 2,50,000
Selling and distribution overheads 4,80,000
Bad debts 20,000
Preliminary expenses (written off) 10,000
Legal charges 5,000
Dividend received 50,000
Interest on deposit received 10,000
Sales 1,20,000 units 7,00,000
Closing stock:
Finished stock- 40,000 units 1,20,000
Work-in-progress 80,000
The cost accounts reveal:
Direct material consumption: Rs, 4,80,000.
Factory overhead recovered at 20% on prime cost.
Administration overhead at Rs. 3 per unit of production.
Selling and distribution overhead at Rs. 4 per unit sold.
Required: prepare
(i) Cost sheet.
(ii) Financial profit and loss account,
(iii) Statement reconciling the profits disclosed by the costing
234
Reconciliation of
Cost and Financial
Accounts
Profit and loss account and financial profit and loss account.
Cost sheet
Particular Rs
Direct material consumption 2,80.000
Direct wages 1,00,000
8,56,000 2,14,000
40,000
1,60,000
Cost of goods sold 6,42,000
+selling & distribution-overhead(1,20,000 x Rs.4) 4,80,000
Working Notes:
*Units product = No. of unit sold + No. of units in closing Stock
= 1,20,000+40,000 = 1,60,000 units
Illustration 4: the following profit and loss account for the year ending 31st March
2015 has been extracted from the bonus of A Ltd
Additional information
Cost account manual states that the factory overheads are to be recovered at 50% of
direct wages, administration overheads of 10% of work cost and selling and
distribution overheads @ Re.l per unit sold
The units of product sold and stock in hand were 4,000 and 257 respectively.
Prepare:
(i) Statement of cost and profit as per cost accounts
(ii) Reconciliation statement.
Solution
Statement of cost and profit as per cost accounts
Particulars Rs
Direct material 10,000
Direct labor or direct wages 20,000
Prime cost 30,000
Add: factory overheads50%of direct wages 10,000
Work 40,000
Less: closing work in progress
Direct labor 600
Direct material 400
Factory expenses 300 1,300
Factory cost or work cost 38,700
Add: administration overheads 10% of work 3,870
cost 42,570
Total cost of production
236
Reconciliation of
Cost and Financial
Accounts
2,570
Less: cost off finished goods 42,570x257/4,257*
40,000
Total cost of production of goods sold 4,000
Add: selling and distribution overheads
@Re.1per unit sold(i.e4,000x1) 44,000
Cost of sales 6,000
Add :profit (balancing figure)
50,000
sales
Solution
Reconciliation statement
Particulars Rs +
Profit as per cost account 6,000
Add: over-absorption of factory overheads
(10,000 – 9,500) 500
Closing stock overvalued in financial accounts 130
(2,700 – 2,570)
Over-absorption of selling expenses
(4,000 – 3,800) 200
Less: under-absorption of administrative overheads. 1330
[5,200 – 3,870] 1000
Interest on capital 1500
Goodwill written off
6,830 3830
Profit as per financial account (6,830 – 3830) 3,000
Rs.
(1) Factory overheads under-absorbed 40,000
(2) Administration overheads over absorbed 60,000
(3) Depreciation charged in financial accounts 3,25,000
(4) Depreciation charged in cost accounts 2,75,000
(5) Interest on investment not included in cost accounts 96,000
(6) Income tax provided 54,000
(7) Interest on loan funds in financial accounts 2,45,000
(8) Transfer fees (credit in financial books) 24,000
(9) Stores adjustment (credit in financial books) 14,000
237
Cost and Management
Accounting
Prepare a statement showing reconciliation between the figure of net loss as per cost
accounts and the figure of net loss shown in the financial books.
238
Reconciliation of
Cost and Financial
Accounts
Illustration 7: from the following informations, reconcile the profit as per cost
accounts and find out the profit as per financial accounts:
Reconciliation statement
Particular Cost A/c Financial
Rs A/c Rs
Profit 86,250
Opening stock:
Material 10,500 10,300
Work in progress 8,500 8,000
Closing stock: material 14,200 15,000
Work in progress 6,000 5,600
Dividend received 600
Loss on sale of investment 1,000
Goodwill written off 2,500
Preliminary exp. Written off 3,000
Overhead incurred 40,000
Overhead absorbed in cost accounts 38,500 -
240
Reconciliation of
Cost and Financial
Accounts
Illustration 10:
(b)from the information given below ,prepare:
(i) a statement showing costing profit or loss and (ii)another statement
reconciling the costing profit with those shown by financial account:
Trading and profit & Loss account
241
Cost and Management
Accounting
Working notes:
1. Total factory overheads are Rs 45,000 out of which Rs 27,000 are fixed. Thus Rs
18,000 are available overheads. In cost accounts, fixed factory overheads must
have been determined on the basis of normal output of 2,25,000 units. Thus, the
fixed factory overheads recovered in cost accounts would be Rs 18,000
{27,000/2,25,000 * 1,50,000}. Variable overheads woulds would be Rs 18,000 in
cost account.
242
Reconciliation of
Cost and Financial
Accounts
Reconciliation statement
Particulars Amount Amount
Rs Rs
Profit as per cost accounts 22,000
Less: factory overheads under absorbed in
Cost accounts Rs (45,000 – 36,000) 9,000
Office overheads under absorbed in
Cost accounts Rs (13,500 – 9,000) 4,500
Selling and distribution overheads under-
Absorbed in cost accounts Rs (9,000 – 8,000)
Profit as per financial accounts 1,000 14,500
7,500
11.6 QUESTIONS
1. Describe the reasons which are responsible for the differences in profits as revealed
by cost accountings and financial accounting
2. Indicate the reasons why it is necessary for the cost and financial accounts of an
organization to be reconciles?
3. Write a note on the items excluded from cost accounts
4. Differentiate between reconciliation statement and reconciliation account
5. The net profit of a business house according to financial accounts was Rs 84,377
while profit shown by cost accounts was Rs 1,06,200 for the same year. Prepare
reconciliation statement to reconcile both the profits from the following information:
(i) Depreciation charged in financial accounts Rs 5,600, while recovered in
cost accounts Rs 6,250.
(ii) Works overheads under-absorbed in cost accounts Rs 1,560 while office
overheads over-recovered in cost accounts Rs 850.
(iii) Interest on loan not included in cost Rs4,000.
(iv) Loss due to obsolescence charged in financial accounts Rs 2,580.
(v) Bank interest and dividend deceived Rs 375.
(vi) Income-tax paid Rs 20,150.
(vii) Loss due to depreciation in inventories charged in financial accounts Rs
3,375.
(viii) Stores adjustment (credited in financial accounts) Rs 237.
[B.Com. (Pass) Delhi]
[Ans. Profit as per financial accounts Rs 76,377]
6. Prepare cost sheet from the following data provided by M/s. R.S. Ltd. For the year
ending 31st March, 2012:
243
Cost and Management
Accounting
Rs Rs
Raw materials 15,000 production 17,100
Direct labor 9,000 sales 16,000
Machine hours 900 selling price per unit 4
Machine hour rate 5 selling overhead per unit 50 paise
244
Reconciliation of
Cost and Financial
Accounts
SUGGESTED REFERENCE
Mittal & Maheswari, Elements of Cost Accounting, Shree Mahavir Book Depot
(Publishers) 2015.
Varshney J.C., Principles and Practice of Cost Accounting, Wisdom Publication
House (2009).
Mittal & Maheswari, Management Accounting, Mahavir Publication, 2015.
Arora M.N. Cost Accounting, Vikas Publishing House Pvt. Ltd. 2013.
Jain & Narang, Cost Accounting, Kalyani Publishers 1998.
Arora M.N., Management Accounting, Himalaya Publishing House, 2006.
245
UNIT-12 BUDGETING AND BUDGETARY CONTROL
LEARNING OBJECTIVES
STRUCTURE
12.1 Introduction
12.2 Advantages of Budgeting
12.3 Budgetary Control
12.4 Advantages and Limitation of Budgeting Control
12.5 Organisation and Administration
12.6 Sales Budget
12.7 Production Budgets
12.8 Material and Labour Budget
12.9 Factory overhead Budget
12.10 Selling overhead Budget
12.11 Cash Budgets
12.12 Performance Budgeting
12.13 Zero Base Budgeting
12.1 INTRODUCTION
The word budget is derived from a French word BOUGETTE, PURSE. It generally refers
to a list of all planned expenses and revenue. The main objective of the management
accounting is to provide information to management for proper planning and control.
Budgeting can act as an important tool of both planning and controlling. It is an
estimation of the revenue and expenses over a specified future period of time. A budget
can be made for a person, family, business, government, country, multinational
organization etc. 'No risk no gain' is the slogan of business. The higher the risk, the
higher is the profit. In order to maintain the profitability and solvency of any business, a
plan has to be formulated in relation to future financial requirements. This plan is known
as Budget. Thus, Budgeting is related to various methods of planning and preparation of
budget plans. It is an act of preparing the budgets. Hence budget is a statement of the
estimated revenue and expenditure of the government.
246
Budgeting and
Budgetary Control
The primary objectives of every business firms is to maximise the profits and to minimize
the cost. This can be possible if an organisation have good budgetary system. Budgetary
control system is quite helpful in reducing cost of a product in business. This is a
controlling technique in which the actual state of affairs is compared with the budget and
if there is any deviation, a necessary action will be taken by the management to remove
the deviation.
(i) The main objective of budgetary control is to control the production and other
costs with maximum output.
(ii) It establishes coordination amongst various department.
(iii) Various activities are controlled through budgetary control for the attainment of
budget estimates.
(iv) Budgetary control helps administrations in smooth running of the business. It can
be used in production, administration, sales and in estimating financial
requirements.
(v) Targets for various departments are ascertained through budgetary control.
Proper steps can be taken against those persons who are not in a position to attain
their targets.
(vi) Business policies are determined through budgetary control, which shows the
path for the future growth of the organisation.
247
Cost and Management
Accounting
248
Budgeting and
Budgetary Control
Budget period : Every budget is prepared for a definite time period, few budgets
are prepared for long term period and some short term budgets are also prepared.
Budget key factor : It is a factor which affects the level of activity. There can be
more than one key factor in any business organisation. Key factors are applicable
in raw material, management and working capital etc. 0\
Organisation chart: Budget chart should be prepared for budgetary control. In
this chart, rights of each officer are specified. The outline of budget chart
depends upon the nature and size of the business.
Budget Manual: It is a written document in which directions for the preparation
of budget are given and the method to be adopted is also described in it.
Level of activity : Level of activity can be fixed on the basis of past results or on
the basis of optimum capacity. The best level would be if the level is fixed on the
basis of general capacity keeping in mind the present conditions.
Budget controller: There should be a man controlling the budget, known as
budget controller. The main function of budget controller is to look into
preparation of budget and establishment of coordination. This officer may help in
removing obstacles in many departments.
Types of Budgets
The success of any business firms depends upon the quick turnover of its production.
Every company wants to increase its sales and also market share. Thus, every effors are
made to achieve the sales targets. Sales budgeting is a key function of sales management.
Sales budget is the most important budget based on which all other budgets are built up.
This budget is a forecast of quantities and values of sales to be achieved in a budget
period. Every company may have sales projection, which will be made as a periodic basis
and the sales budget will be prepared accordingly. In brief, a sales budget is an estimate a
future sales expressed in quantity and money. Every effort should be made to ensure that
its figures are as accurate as possible. Because this budget is starting budget and it will
have impact on other budget also. Here the sales manager should be made directly
responsible for the preparation and of the budget. However, before preparing this budget,
the sales manager should be taken in to account the following factors.
(i) Previous years sale figure and its trend, (ii) Estimates given by sales man, (iii)
Available plant capacity, (iv) Availability of raw materials and other necessary supplies,
249
Cost and Management
Accounting
(v) General economic trend of the country, (vi) Seasonal fluctuations in sales, (vii)
Available of finance, (viii) Nature and degree of competition in the market (ix) Planned
advertising and product promotion and (x) Political and legal environment.
250
Budgeting and
Budgetary Control
Production budget involves planning the level of production which in turn involves the
answer to the following four questions.
(i) What is to be produced? (ii) When to be produced? (iii) Where to be produced? and
(iv) How to be produced?
(i) Opening and closing, (ii) quantity required to meet projected sales, (iii) Availability of
storage facility, (iv) Amount of investment (finance) required (v) Policy of management
regarding procurement of goods and (v) Time taken in production process.
Formula
Total units to be produced =
Expected units of sales XXX
(+) Desired units in closing stock xxx
xxx
(-) Estimated units is opening stock xxx
Units to be produced xxx
Illustration 2. Aditya Ltd. plans to prepare a production budget for 3 products. The sales
forecasts of these products are 166400 units, 145600 units and 176800 units respectively.
The estimated requirements of inventory both at the beginning and at the end of the
budget period are shown in the following schedule:
A B C
Opening (Units) 32,000 24,000 40,000
Closing (Units) 41,600 22,320 55,200
251
Cost and Management
Accounting
This budget is an estimate of cost of output planned for a budgeted period. Further it
shows a summary of different items of cost to be incurred for the budgeted output. This
budget may be classified into material cost budget, labour cost budget and overhead cost
budget, because cost of production includes material, labour and factory overhead.
The following information has been made available from the records of incharge, Sunil
Tools limited for the last six months of 2014 and of only the sales of January 2015 in
respect of product X:
(i) The units to be sold in different months are:
July 2014 1,100 November 2014 2,500
August 2014 1,100 December 2014 2,300
September 2014 1,700 January, 2015 2,000
October 2014 1,900
(ii) There will be no work-in-progress at the end of any month.
(iii) Finished units equal to half the sales for the next month will be in stock at the
end of every month (including June, 2014)
(iv) Budgeted production and production cost for the year ending 31st December
2014 are as thus: Production (units) 22,000
Direct Materials per unit Rs. 10.00
Direct Wages per unit Rs. 4.00
Total Factory overheads apportioned to Product Rs. 88,000
It is required to prepare:
(a) a Production Budget for each of the last six months of 2014, and
(b) a Summarized Production Cost Budget for the same period.
252
Budgeting and
Budgetary Control
Solution (a) Production Budget for the six months ending 31-12-2014
July Aug. Sept. Oct. Nov. Dec. Total
Sales (in units) 1,100 1,100 1,700 1,900 2,500 2,300 10600
Add: Closing Stock 550 850 950 1,250 1,150 1,000 5750
1,650 1,950 2,650 3150 3,650 3,300 16350
Less: Opening Stock 550 550 850 950 1,250 1,150 5300
Production units 1,100 1,400 1,800 2,200 2,400 2,150 11050
Working Notes:
(1) Calculation of Closing Stock
1100 2500
July 550 unit October 1250 unit
2 2
1700 2300
August 850 unit November 1150 unit
2 2
1900 2300
September 900 unit December 1150 unit
2 2
Material budget is prepared with a view to ensure regular supply of raw materials
according to the requirements of production schedule. A schedule of materials
requirements is prepared. The quantity of material required for production and the
required inventory will indicate the quantity of each material, which should be made
available. The inventory of raw materials at the beginning of the budget period is
deducted and the balance quantity is produced during the budget period. Materials budget
checks wastage of raw material and at the same time helps in the determination of
economic order quantity. In brief this budget provides information about the material to
be a purchased from the market during the current period.
253
Cost and Management
Accounting
Material required :
X = 84,000 3 = 2,52,000 and Y = 84,000 5 = 4,20,000
Formula
Planned production in units x Direct Labour hour required per units = Budgeted labour
hours required.
Budgeted Direct Labour Cost =
Budgeted Labour hours required x cost per Direct Labour hours.
Illustration 5
Aradhya Ltd. manufactures two products using one grade of labour. Shown below is an
extract from the company's working paper for the next period's budget:
Particular Product Product
Budgeted Production (Units) 3500 4500
Standard hours allowed per product 5 4
Budgeted wage Rate Rs. 10 per hour.
Overtime premium is 50% and is payable, if a worker works for more than 40 hours a
week. There are 100 direct workers. The target productivity ratio for the productive hours
worked by the direct workers in actually manufacturing the production is 80%; is
addition the non-productive downtime is budgeted at 20% of the Productive hour worked.
There are twelve 5 days week in the budgeted period.
Calculate the wages/labour budget for direct workers showing hours required and wages
paid.
Solution
Particulars Product x Product x Total
Budgeted Production in units 3500 4500
Standard hours for budget 17500 18000 35,500
(3500 units (4500 units
x 5 hours x 5 hours)
Standard hours for budgeted production at
target efficiency ratio of product x = 35500 hours 100 44375
80
8350
Add Normal Production down time to (x) 44375x20%
Total
= labour hours required 53250
Less Normal labour hours 48000
= (100 workers 12 weeks 5 days 8 hours) =
Difference (i.e. overtime) 5250
Wages for normal hours 48000 Rs. 10 480000
Overtime wages (5250 Rs. 15) 78750
Total Wages 558750
255
Cost and Management
Accounting
The factory overhead budget shown all the planned/manufacturing expenses which are
needed to produce the budgeted production level of a period other than direct costs which
are already covered under direct material and direct labour .budget. In this budgets all
manufacturing expenses are classified as fixed, variable and sermvariable expenses. In
preparing the budget, fixed expenses can be estimated on the basis of past information
after taking into consideration the expected changes which may occur during the budget
period. Variable expenses are estimated on the basis of output.
Illustration 6: Prepare a factory overhead budget and calculate factory overhead rates at
50% and 70% capacity. The following particulars are given at 60% capacity.
Semivariable overheads:
Power (40% fixed) 30,000
Repairs and maintenance
(20% variable) 3,000
Fixed Overheads:
Depreciation 17,000
Insurance 4,000
Salaries 20,000
Estimated Direct Labour hours 186,000 hours
Solution
Manufacturing/Factory overhead budget
Particulars 50% 60% 70%
capacity capacity capacity
Rs. Rs. Rs.
Variable Overheads:
Indirect Materials 5,000 6,000 7,000
Indirect Labour . 15,000 18,000 21,000
Semi Variable Overheads
Semi variable overheads 12,000 12,000 12,000
Power Fixed
Variable 15,000 18,000 21,000
Repairs maintenance Fixed 2400 2400 2400
Variable 500 600 700
256
Budgeting and
Budgetary Control
Depreciation
Insurance 4000 4000 4000
Salaries 20,000 20,000 20,000
Total overheads
90900 80,000 105,100
Working Notes:
(1) Power
40
At 60% capacity power cost is Rs. 30,000 of which Rs. 12000 (ie. 30,000 ) is fixed
100
for all
capacity and Rs. 18000 is variable.
The variable portion of the power cost
18000
at 50% capacity = 50 Rs.15000
60
18000
and for 70% capacity = 70 Rs.2100
60
The selling and distribution overhead budget is the forecast of the cost of selling and
distribution for budget period and it is clearly related to sales Budget. This budget is
comprised of the budgets of all non-manufactures departments such as the sales,
marketing, advertising, ware housing etc. It is a component of master budget and is
prepared by all types of business.
257
Cost and Management
Accounting
Cash budget is a forecast related to cash for a certain period. Under it flow of cash is
controlled by estimating cash receipts and payments for a definite period. Cash should be
available in sufficient quantity in future. Cash is received from the sale, rent, interest,
dividend and sale of assets etc. Cash is paid for creditors, purchases, salaries, rent, taxes
and capital expenditures etc.
(i) Receipt and Payment Method: Under this method, a statement is prepared
under which all receipts from various sources and all payments to various items
are recorded. The difference of both the side is known as cash balance and cash is
forecasted.
(ii) Projected Balance Sheet Method: Under it, efforts are made to forecast cash
position at special points. At the end of the budget period, a projected Balance
sheet is prepared in which forecasts are made for all the assets and liabilities, but
it excludes cash and bank balance or overdraft. These items are projected in
Balance Sheet and the balance is known as cash in hand or balance at Bank.
.
(iii) Cash Flow Method: In this method, it is assumed that cash comes into the
organisation from Debtors, Stock, Bills Receivables and there is no increase or
decrease in fixed assets. In case of changes made, proper adjustments are made
and recorded. Few adjustments are to be made while preparing cash forecasts.
Cash balance is adjusted with changes in current assets and current liabilities etc
and then with the help of all adjustments cash balances are drawn, and
forecasted.
(iv) Adjusted Profit and Loss Method: The profit forecast for the budget period is
adjusted for non-cash items and for expected changes in assets and liabilities.
Thus net estimated profit is increased by the amount of non-cash transactions
which in turn is added by capital receipts, reduction in assets and increase in
liabilities to form total cash receipt. It is reduced by payments made and the
balance will be cash available.
259
Cost and Management
Accounting
interest and dividend (ix) Redemption of Preference share capital (x) Repayment of
loans, (xi) Other expenditures.
(c) Cash Balances : After recording all cash receipts and cash payments, cash balance is
ascertained. If total of cash receipts is greater than total of payments the balance is treated
as cash balance and if the total of payment is greater than the total of cash receipts, then
the balance is known as overdraft or credit balance of cash.
Illustration 8. Prepare a cash budget for 4 months from April 2015 to July 2015
from the following information:
Month Sales Credit Wages Manufactures Distribution &
Purchase Administration Selling expense
Overhead
April 30,000 30,000 6,000 1,000 600
May 33,000 21,000 7,000 1,100 700
June 36,000 20,000 8,000 1,100 700
July 39,000 18,000 9,000 1,200 800
August 42,000 17,000 10,000 1,300 800
Additional Information
(1) Cash balance on 1st April was Rs. 17,500.
(2) 50% of sales are on credit which are realised in subsequent month.
(3) Suppliers are paid in the month following the month of supply.
(4) Delay in payment of wages and overheads is 30 days.
(5) Dividends on investment Rs. 5,000 may be received in April, June and July.
(6) Machinery purchased for Rs. 30,000, payment made in there equal instalments in
April, June and July.
Solution: Cash Budget
Rs.
April May June July
(A) Balance b/d. 17,500 27,500 21,400 16,100
(B) Receipts:
Cash sales 50% 15,000 16,500 18,000 19,500
Debtors 15,000 16,500 18,000
Dividends 5,000 5,000
Total (A + B) 37,500 59,000 55,900 58,600
;
(C) Payments
Creditors 30,000 21,000 20,000
Wages 6,000 7,000 8,000
Administration overheads 1,000 1,100 1,100
Distribution expenses 600 700 700
Machine 10,000 10,000 10,000
Total C 10,000 37,600 39,800 39,800
Closing Balance (A + B-C) 27,500 21,400 16,100 18,800
260
Budgeting and
Budgetary Control
In business zero-base budgeting was introduced by Peter Payal of USA in 1969. But in
military it was in use since 1960. It helps the manager in implementation and formation
of various managerial activities. Under this system, every item is checked independently
before the preparation of the budget, so that its utility can be ascertained in real life.
Under conventional budget amendments are made to previous budget, whereas in zero
Budget every activity and item is tested and then budget is prepared accordingly for the
future.
(iv) Irrelevant items are not detected easily and it increases the cost unnecessary.
(v) Proper accountability is not defined and due to it the targets of the organisation
are affected.
263
Cost and Management
Accounting
Method for preparation of flexible budget: Fixed, variable and semi-variable costs are
kept into mind while preparing flexible budget. Fixed expenses do not change and
variable expenses change at every level of activity. Semi-variable expenses change at
every level of activity. There are some semi-variable expenses which remain fixed at
different level of activities and some of them change at different levels. The following
points are to be kept into mind:
(i) Fixed expenses are always remain fixed at each level of production.
(ii) Variable expenses per unit remains the same. The total variable expenses
increases with the increase in production and vice-versa.
(iii) Some part of semi-variable is variable and other part is of fixed nature.
(iv) There may be changes in the rates of sales for different level of production.
264
Budgeting and
Budgetary Control
Solution:-
Flexible Budget for Administration, Selling and Distribution Cost
Particulars 90% 100% 110%
Rs. Rs. Rs.
Sales Rs. 675,000 Rs. 750,000 Rs. 825,000
Administration Cost
Office salaries 90,000 90,000 90,000
General expenses 13,500 15,000 16,500
Depreciation 7,500 7,500 7,500
Rates and Taxes 8,750 8,750 8,750
Total Administration Cost (A) 119750 121250 122750
Distribution cost
Wages 15000 15000 15000
Rent 6750 7500 8250
Other expenses 27000 30,000 33,000
Total Distribution Cost (B) 48750 52500 56250
Selling cost
Salaries 54,000 60,000 66,000
Travelling exp 13,500 15,000 16,500
Sales office expenses 6750 7500 8250
General expenses 13500 15000 16500
Total Selling cost (C) 87,750 97,500 10,7250
Total Cost (A+B+C) 2,56,250 2,712,50 2,86,250
Profit (Sales-Total Cost) 4,18,750 4,78,750 5,38,750
Distinction between fixed Budget and Flexible Budget
Fixed Flexible
Budget Budget
1. Flexibility This budget is inflexible and This budget is flexible and can
does not change with the actual
be suitably recasted quickly
volume of output achieved. according to the level of
activity achieved.
2. Condition Fixed budget assumes that Flexible budget is designed to
conditions would remain change according to changed
static i.e. unchanged. conditions.
3. Classification In this Budget costs are not In this budget costs are class-
classified according to their ified according to the nature
variability i.e. fixed and of their variability.
variable.
4. Comparison Very difficult to compare Comparison can be made
performance if the volume of easily.
output is different.
5. Forecast It does not help in fore- It help in fore casting.
265
Cost and Management
Accounting
casting
6. Application It has limited application It has wider application
7. Tool for cost control It cannot be applied as a It can be applied as a tool for
tool for cost control. cost control.
Master Budget
The master budget is expressed in financial terms and set out plan for the operations and
resources of the firm. It is a summary of the budget schedules high lighting the budget
period. The master-budget called the comprehensive budget is the complex blueprint of
the planned operations of the firm. Master budget is an overall budget of the firm, which
includes all other small departmental budgets. It is a network consisting of many separate
budgets. It coordinates various activities of the business. It contains consolidated
summary of all the budgets prepared. Such budget is of no use for the departmental
executives. It draws the attention to those issues which provide immediate attention.
CIMA defines this budget as "the summary budget incorporating its component
functional budget and which is finally approved, adopted and employed." This master
budget is a summary of all functional budget is capsule form available in one report.
Preparation of master budget is a complex process. It includes all preparation of a
projected profit and loss account and balance sheet. Preparation of master budget
involves the following steps :
(i) Preparation of Sales Budget, (ii) Cost Budget, (iii) Projected Profit and loss Account,
(iv) Production cost Budget, (v) Cash budget, (vi) Projected Balance Sheet.
Difference between Budget and forecast
Budget Forecast
Meaning It means plan of action It means prediction
Preparation of budget only the authorised Any body can make a fore
management of a company cast about a company's
can prepare budget performance
Expressed It is expressed in terms of
rupees or quality It is not always expressed
in terms of rupees or
quantitative.
As a matter of policy, the company maintains the closing balance of finished goods and
raw materials as follows:
266
Budgeting and
Budgetary Control
(i) Finished goods — closing stock of a month will be 50% of the estimated sales
for the next months.
(ii) Raw materials — closing stock of a month will be equal to estimated
consumption for the next month.
Each unit of production consumes 2 kg of raw material costing Rs. 6 per kg.
Prepare the following budgets for the half year ending 30-9-2015:
(i) Production budget (monthwise is units)
(ii) Raw material purchase budget (monthwise in units and cost).
Solution
Production Budget for the half year ending 30-9-15
April May June July Aug. Sep. Total
Sales (units) 12,000 13,000 9,000 8,000 10,000 12,000 64000
Add: Closing stock 6,500 4,500 4,000 5,000 6,000 7,000 33000
Less: Opening stock (6,000) (6,500) (4,500) (4,000) (5,000) (6,000) 32000
Estimated Production 12,500 11,000 8,500 9,000 11,000 13,000 65000
267
Cost and Management
Accounting
268
Budgeting and
Budgetary Control
Solution
Flexible Budget
Add: Administration :
From the following information Regular a Cash Budget for the month of May and
June, 2015
(a) Cash balance a.4 on 1st May, 2015 was Rs. 8,000.
(b) 75% of the sales are realised in the same month and rest in the following month.
(c) Period of ere/flit from suppliers is one month.
(d) Time Lag in payment of wages is one month
269
Cost and Management
Accounting
Solution
Cash Budget
for the month of May and June, 2015
Particulars May Rs. June Rs.
Opening balance 8,000 25,000
Receipts :
Cash Sales 48,000 43,500
Collection from Debtors 15,500 16,000
A. Total Receipts
Cash Payments :
71,500 85,000
25,500 42,000
Working Notes:
(1) Calculation of cash sale
75 75
May 64000 Rs.15,500 June 58000 Rs.16, 000
100 100
(2) Calculation of Collection from Debtors
25 75
May 62000 Rs.15,500 June 64000 Rs.16, 000
100 100
270
Budgeting and
Budgetary Control
QUESTIONS
Theoretical Questions:
271
Cost and Management
Accounting
3. "For planning, the manager wants information about the future, for control about
the past." Comment upon this statement.
4. What is flexible budget? State its importance and limitations.
5. Mention the utility of sales budget. What factors are to be taken into
consideration before preparing sales budgets.
6. What is business budget ? State the procedure for preparation of a budget.
7. Explain Budgetary control and describe the advantages of a good budgetary
control.
8. Explain Flexible Budget and mention the advantages and limitation of a flexible
budget.
9. Describe production Budget and discuss its purposes. ;
10. Write short notes on:
(i) Master budget, (») Sales budget
(iii) Labour budget, (iv) Cash budget
11. What is cash budget ? How it is prepared ? Mention its utility.
12. Define cash budget and discuss its advantages and limitations.
13. Explains briefly zero base budgeting.
14. Ravi Ltd. plans to prepare production budget for its 3 products X, Y and Z, the
sales forecasts for their product is 80,000 units, 100,000 units and 1,40,000 units
respectively. The estimated requirements of inventory at the beginning and as the
end are as under:
A B C
1st January (units) 10,000 12,000 14,000
31st December (units) 8,000 6,000 4,000
Prepare a production budget. [Ans. A: 78,000 units, B: 94,000 units, C: 1,30,000 units]
(i) Finished units equal to half the sales for the next month will be in stock at the
end of each month including September 2014.
(ii) Product cost for 31st March 2015 are as under:
Production (units) 80,000
Material per unit Rs. 6
Labour per unit Rs. 3
Total Factory overheads Rs. 45,000
15. Prepare a sales overhead budget from the following information:
Rs.
Salaries of sales Deptt. 2,000
Advertisement 1,000
Expenses of sales deptt. 600
Salesman salaries 2,400
272
Budgeting and
Budgetary Control
273
Cost and Management
Accounting
SUGGESTED REFERENCE
Mittal & Maheswari, Elements of Cost Accounting, Shree Mahavir Book Depot
(Publishers) 2015.
Varshney J.C., Principles and Practice of Cost Accounting, Wisdom Publication
House (2009).
Mittal & Maheswari, Management Accounting, Mahavir Publication, 2015.
Arora M.N. Cost Accounting, Vikas Publishing House Pvt. Ltd. 2013.
Jain & Narang, Cost Accounting, Kalyani Publishers 1998.
Arora M.N., Management Accounting, Himalaya Publishing House, 2006.
274
BLOCK-5
The present block refers to the concepts of management accounting. In this block
the learners will learn about the financial statements analysis, accounting ration,
cash flow statement as well as the operational part of the budgeting that how the
financial activities of a business is managed. The present block refers the
following unit;
Unit 13: Financial Statement Analysis
275
UNIT-13 FINANCIAL STATEMENT ANALYSIS
CONTENTS
13.0 Objectives
13.1 Introduction
13.2 Techniques of financial analysis
13.3 Common size statement analysis
13.4 Comparative statement analysis
13.5 Objective of comparative statement financial statement
13.5.1 Performance analysis
13.5.2 Strength analysis
13.5.3 Planning and forecasting
13.6 Comparative income statement
13.7 Comparative balance sheet
13.8 Trend analysis
13.9 Statement of changes in fund
13.10 Ratio analysis
13.11 Check your progress
13.0 OBJECTIVES
13.1 INTRODUCTION
Every organization prepares financial statement at the end of every period. The users of
accounting information derive information from these financial statements in accordance
with their objectives. Financial Statements are divided in three parts, namely, Balance
Sheet, Profit & Loss Accounts and Cash Flow Statements. Balance Sheet shows the
financial strength of the business, profit & loss states the operation status of the company.
Cash flow statements are prepared to examine the change the changes in cash position of
the organization. Users of financial statement can get better idea about financial strengths
and weakness of the enterprises if they properly analyze information reported in these
statements. Financial analysis is the starting point for making plans before using any
sophisticated forecasting and planning procedures. Understanding the past is a
prerequisite for anticipation of futures.
Financial analysis is the process of identifying the financial strength and weakness of the
enterprises by properly establishing the relationship between Balance Sheet, Profit &
276
Financial Statement
Analysis
Loss accounts and cash flow statement. The analysis may be done by the management or
by the outsider of the business. The nature of analysis depends upon the objectives of the
analyst or user.
Now we discuss all above analysis techniques in details in the following sections.
Common size statements are those statements which are prepared by assuming a common
base. For instances, in case of profit & loss accounts, sales are consider 100% and
expenditures are calculated as percentage of sales. Similarly for Balance Sheet, items are
expressed as percentage of total assets or total funds.
Such comparisons can be done only where all the accounting policies of the firm are
same as in the previous year. If there is any change in the accounting policies, then the
analysis will not fulfill objective of the firm.
This technique is very useful when we compare our business with the other business of
the same nature. This technique may also be used when we compare our business with
the same nature of industry. Usually industry data are published in this form.
The above technique can be better understood from the following example:
If we apply the technique of common size statement, then the above Income statement
shall be presented as given below.
The above common size statement shows that in 2007-08, Cost of goods sold is 82.50%
of the sales whereas in 2006-07 is 82.20. Similarly Gross profit has been decreased in
2007-08 from 2006-07 and so on.
The common size statement is very simple to understand and it may be used for
comparison instead of to read typical financial statement.
The common size statement can be prepared for position statement also. It is illustrated
through the following example.
278
Financial Statement
Analysis
APPLICATION OF FUNDS
Gross Block 78.00 72.00
Less: Acc. Depreciation 27.00 24.00
Net Block 51.00 48.00
Capital Work in Progress 9.00 4.00
Investments 15.00 12.00
CURRENT ASSETS, LOANS & ADVANCES
Inventories 18.00 14.00
Sundry Debtors 14.00 16.00
Cash and Bank 8.00 7.00
Loans and advances 8.00 8.00
Less: Current Liabilities and Provisions
Sundry Creditors 24.00 22.00
Provisions 9.00 13.00
Net Current Assets 15.00 10.00
Miscellaneous Expenditures 2.00 2.00
Total Assets 92.00 76.00
SOURCES OF FUNDS
Share Capital 27.17 27.63
Reserve & Surplus 18.48 19.74
Total Shareholders fund 45.65 47.37
Secured Loans 38.04 35.53
Unsecured Loans 16.30 17.11
Total Debt 54.35 52.63
Total Liabilities 100.00 100.00
APPLICATION OF FUNDS
Gross Block 84.78 94.74
Less: Acc. Depreciation 29.35 31.58
Net Block 55.43 63.16
Capital Work in Progress 9.78 5.26
Investments 16.30 15.79
The above common size balance sheet shows all the assets in percentage form on the
basis of total asset and percentage composition of shareholders fund and long term debt
of the total liability.
When we prepare common size statement for one period by measuring the percentage,
known as vertical analysis whereas when compare these figures from other period, it
becomes horizontal analysis. For example, if we prepare common size statement only for
2007-08 of the above company. It is known as vertical analysis of common size
statement. However when we compare these figure from 2006-07, it becomes the
horizontal analysis of common size statement.
280
Financial Statement
Analysis
Comparative income statement shows the operating results of the enterprises for any
number of accounting periods. It shows the change in terms of values and percentage on
the basis of base year.
The aforesaid points may be better understand from the following given example.
Example-1
Prepare comparative Income Statement of Aslam Ltd with the help of following
information
Particulars 31.03.2006 31.03.2007
Rs. Rs.
Sales 5,00,000 8,00,000
Cost of goods sold 3,00,000 5,00,000
Direct expenses 40,000 20,000
Indirect Expenses 30,000 40,000
Income Tax 40% 50%
281
Cost and Management
Accounting
Solution
Comparative Income Statement of Aslam Ltd
Example-2
Income Statement of ABC Ltd
Particulars 2006 2005
Rs. Rs.
Sales (Net) 1265000.00 870000.00
Cost of goods sold 910800.00 643800.00
Gross Profit 354200.00 226200.00
Administrative Expenses 101200.00 78300.00
Selling and Distribution expenses 164450.00 95700.00
Operating Income 88550.00 52200.00
Other Income 125000.00 115000.00
Earning Before Interest and Tax 213550.00 167200.00
Interest paid 65000.00 74000.00
Profit before Tax 148550.00 93200.00
Taxes 42000.00 27000.00
Profit after tax 106550.00 66200.00
Solution
Comparative Income Statement of ABC Ltd
Particulars 2006 2005 Change Change
Rs. Rs. Rs. (%)
Sales (Net) 1265000.00 870000.00 395000.00 45.40
Cost of goods sold 910800.00 643800.00 267000.00 41.47
Gross Profit 354200.00 226200.00 128000.00 56.59
Administrative Expenses 101200.00 78300.00 22900.00 29.25
Selling and Distribution
Expenses 164450.00 95700.00 68750.00 71.84
Operating Income 88550.00 52200.00 36350.00 69.54
Other Income 125000.00 115000.00 10000.00 8.70
Earning Before Interest
and Tax 213550.00 167200.00 46350.00 27.72
282
Financial Statement
Analysis
Comparative balance sheets are used to compare assets, liabilities and capital at two or
more different dates. Such analyses are used to examine the progress of the enterprises
during particular period.
According to Foulka “Comparative balance sheet analysis is the study of trend of the
same items, group of items and computed items in two or more balance sheets of the
same business enterprises on different dates.”
The above definition explains that it is a analysis of trends for the group of items of the
balances sheet of any company. It may for the same year at two different dates or for two
different accounting periods.
Example-1
Prepare Comparative Balance Sheet of ABC Ltd from the following
283
Cost and Management
Accounting
2006 2005
Rs. Rs.
SOURCES OF FUNDS
Share Capital 230000.00 190000.00
Reserve & Surplus 165400.00 149700.00
Secured Loans 340000.00 380000.00
Unsecured Loans 135000.00 154500.00
Current Liabilities 244200.00 285600.00
TOTAL 1114600.00 1159800.00
APPLICATION OF FUNDS
Net Fixed Assets 435000.00 387000.00
Investments 340000.00 390000.00
Current Assets 339600.00 382800.00
TOTAL 1114600.00 1159800.00
Solution
COMPARATIVE BALANCE SHEET OF ABC LTD
APPLICATION OF FUNDS
Net Fixed Assets 435000.00 387000.00 48000.00 12.40
Investments 340000.00 390000.00 (50000.00) (12.82)
Current Assets 339600.00 382800.00 (43200.00) (11.29)
284
Financial Statement
Analysis
sources of funds. The company has issued share capital during the year for the
redemption of long term borrowings.
From the application of funds point of view, company has sold his investment for the
payment of debt and making the investments in his fixed assets. Company’s current
assets are decreasing more than decrease in current liabilities.
Overall analysis of balance sheet shows that, financial position of the company is not
favorable.
Example-2
From the following information, prepare a Comparative Balance Sheet of Z Ltd:
Solution
COMPARATIVE BALANCE SHEET OF Z LTD
APPLICATION OF FUNDS
Fixed assets 30,00,000 36,00,000 6,00,000 20
Investments 5,00,000 5,00,000 - -
Current Assets 15,00,000 10,50,000 (4,50,000) (30)
285
Cost and Management
Accounting
Trend analysis is a concept which is used by the financial analysts to examine the
increase or decrease in the value of assets, liabilities, expenses and revenues. This method
is used by the banking industry, usually, to examine and control over the fund disbursed
to the parties. This method is also used for analysis of financial position after a certain
period of time on the basis of past data of accounting.
Trend analysis is conducted for revenue and expenditure nature items of the business.
However, now a days, it is also used for critically evaluation of the net worth of
organization also.
Trend method may be used in the light of two factors, the rate of fixed growth and
general price level. To get a true growth of revenue, such as sales, it should be adjusted
with the true rate of general price level. If the general pricing level is not considered
while analyzing trend of growth, it can mislead management.
TREND ANALYSIS
ABC LTD
2008 2009 2010
Sales 100 120.00 155.90
Earning before interest & tax 100 147.70 156.75
Profit after tax 100 135.80 145.76
Current assets 100 160.00 175.00
Current liabilities 100 135.54 145.86
Gross fixed assets 100 134.40 152.80
Net assets 100 127.96 142.00
Total assets 100 185.65 190.34
Net worth 100 164.54 175.24
Dividend 100 115.00 145.00
For trend analysis, the procedures followed shall be 100 for the base year and on the basis
of this we measure the changes in the following relational year. In the above example, the
base year is 2008.
Following are some examples or indicator which has different meaning as per trend
analysis.
286
Financial Statement
Analysis
v. Rising Inventories
This may be because of slow moving items included in stock, inflated value of closing
stock, purchase invoice entered but goods not received etc.
This may be because of poor cost controls, poor management of business and fail to shift
cost to other business organization.
The balance sheet discloses the financial position of the firm on a specific date. It will be
different from the opening date balance sheet. However, closing balance sheet does not
reflect the changes occurred during that particular period. Therefore the analysts prepare
details of change in those figures of balance sheet between two different dates. This
statement is known as change in funds statement or fund flow statement. In other words,
funds flow statement is a statement that shows details of changes in funds of the
organization between two particular periods. It has been discussed in details of this paper
earlier.
287
Cost and Management
Accounting
288
Financial Statement
Analysis
APPLICATION OF FUNDS
Gross Block 96.00 82.00
Less: Acc. Depreciation 39.00 37.00
Net Block 57.00 45.00
Capital Work in Progress 12.00 14.00
Investments 16.00 12.00
289
Cost and Management
Accounting
Figures in lacs
Particular 2006 2007
Rs. Rs.
Equity Share Capital 120.00 120.00
Preference Share Capital 30.00 30.00
Reserve and Surplus 30.00 36.00
Fixed Assets 180.00 216.00
Investments 30.00 30.00
Current Assets 90.00 63.00
Secured Loans 60.00 54.00
Unsecured Loans 30.00 36.00
Current Liabilities 24.00 26.40
Provisions 6.00 6.60
290
UNIT-14 ACCOUNTING RATIOS
LEARNING OBJECTIVES
Contents
14.1 Introduction
14.2 Importance of Ratio Analysis
14.3 Types of Ratios
14.4 Liquidity Ratios
14.4.1 Current Ratio
14.4.2 Acid Test Ratio
14.4.3 Cash Ratio
14.5 Leverage Ratios
14.5.1 Debt Equity Ratio
14.5.2 Total Debt Ratio
14.5.3 Proprietary Ratio
14.5.4 Fixed Assets to Net worth Ratio
14.5.5 Debt Service Ratio
14.6 Activity Ratios
14.6.1 Inventory Turnover Ratio
14.6.2 Debtors Turnover Ratio
14.6.3 Creditors Turnover Ratio
14.6.4 Assets Turnover Ratio
14.6.3.1 Total Assets Turnover Ratio
14.6.3.2 Net Assets Turnover Ratio
14.6.3.3 Fixed Assets Turnover Ratio
14.6.3.4 Current Assets Turnover Ratio
14.6.3.5Net Working Capital Turnover Ratio
14.7 Profitability Ratios
14.7.1 Gross Profit Ratio
14.7.2 Net Profit Ratio
14.7.3 Operating Profit Ratio
14.7.4 Operating Ratio
14.7.5 Expenses Ratio
14.8 Investment Ratios
14.8.1 Return on Gross Capital Employed Ratio
14.8.2 Return on Net Capital Employed Ratio
14.8.3 Return on Share Capital Employed Ratio
14.8.4 Return on Equity Capital Employed Ratio
14.9 Check Your Progress
14.1 INTRODUCTION
291
Cost and Management
Accounting
various ratios between the related items of financial statements in the summarized form
instead of the heavy figures of amounts of the financial statement. Ratio Analysis is
summarized and comprehensive financial information derived from financial statement. It
helps in judging the financial operation and financial position of the enterprises.
As discussed in the previous modules that the information required for analysis of
financial statement depends upon the information required by finance manager for
decision making. Therefore, the finance manager will need various ratios for financial
planning and decision. However, some of the ratios have been classified on the basis of
their common features. Ratios for analysis of financial statement have been classified in
the following categories.
1. Liquidity Ratios
2. Leverage or Solvency Ratios
3. Activity Ratios
4. Profitability Ratios
Liquidity refers to the ability of a firm to meet its short-term financial obligations when
and as they fall due. The main concern of liquidity ratio is to measure the ability of the
firms to meet their short-term maturing obligations. The failure to meet obligations on
due time may result in bad credit image, loss of creditors confidence, and even in legal
proceedings against the firm. The greater the coverage of liquid assets to short-term
liabilities the better, as it is a clear signal that a company can pay its debts that are
coming due in the near future and still have fund for its ongoing operations. On the other
hand, a company with a low coverage rate should raise a red flag for investors as it may
be a sign that the company will have difficulty in running its operations, as well as
meeting its obligations. However very high degree of liquidity is also not desirable since
it would imply that funds are idle and earn nothing. Therefore it is necessary to strike a
proper balance between liquidity and lack of liquidity.
The liquidity ratios are also known as Balance Sheet ratios because the information used
for measurement of liquidity ratios are taken from the Balance Sheet only.
Ratios that explain about the liquidity of the firm are given below.
293
Cost and Management
Accounting
1. Current Ratio
2. Acid Test Ratio or Quick Ratio
3. Cash Ratio
Measurement
It is measured by dividing the current assets by current liabilities. This ratio is shown in
times. Symbolically,
Current Asset
Current Ratio =
Current Liabilities
Current asset are those which can be converted into cash within a short period of time,
normally not exceeding one year. Current assets include cash and bank balances,
marketable securities, inventory, and debtors excluding provisions for bad debts and
doubtful debtors, bills receivables and prepaid expenses.
Current liabilities are those liabilities which become payable within a period of one year.
It includes accounts payable, short term notes payable, short-term loans, current
maturities of long term debt, accrued income taxes and other accrued expenses.
Analysis
The analysis and comparison of Ratio depends upon the Industry ratio of the firm.
Generally, a Current Ratio of 2 times or 2:1 is considered to be satisfactory, though this is
not applicable in all types of business. Higher the current ratio, the better is the liquidity
of firm. However, too high ratio reflects an in-efficient use of resources and too low ratio
leads to insolvency.
294
Accounting
Ratios
This ratio is more conservative than the current ratio because it excludes inventory and
other current assets, which are more difficult to turn into cash or cash equivalents
immediately. It establishes the relationship between quick assets and current liabilities.
Measurement
It is measured by dividing the quick assets by the current liabilities.
Quick Assets
Quick Ratio =
Current liabilities
Quick assets are those parts of current assets, which may be converted into cash or cash
equivalents immediately or within reasonable short time without a loss in value of assets.
These include cash and bank balances, sundry debtors, bill’s receivables and short-term
marketable securities. Symbolically,
Analysis
The standard of quick ratio is 1:1. i.e. the liquid or quick assets should be equal to the
current liabilities of the firm. Another beneficial use of quick ratio is to compare the
quick ratio with the current ratio. If the current ratio is significantly higher than quick
ratio, then it shows that the firm have invested heavy amount in the Inventories.
Meaning
It shows the relationship between absolute liquid or super quick current assets and current
liabilities. This ratio only considers the absolute liquid assets of the firm.
Measurement
It is measured by dividing the absolute liquid assets by current liabilities. This ratio is
also shown in times. Symbolically,
Absolute liquid assets
Absolute Liquid Ratio =
Current liabilities
Absolute liquid assets include cash, bank balances and short term marketable securities.
Analysis
The cash Ratio of magnitude up to 1:2 may be satisfactory and a firm need not maintain
too much of highly/super liquid assets. If the super liquid assets are too much in relation
to the current liabilities then it may affect the profitability of the firm as these funds
would be idle.
295
Cost and Management
Accounting
These ratios indicate the degree to which the activities of a firm are supported by long
term creditors’ funds as opposed to owner’s funds. These ratios are measured to know the
ability of firm with regard to periodic payment of interest during the period and loan
repayment of principal on maturity or in predetermined installments at due dates of the
long term creditors.
The ratio is based on the relationship between borrowed funds and owner’s capital.
Therefore first part is computed from the balance sheet and the second part is calculated
from the profit and loss a/c.
Measurement
It is calculated by dividing the total long term funds (Debt) by shareholder funds (Equity)
or Net Worth. Symbolically,
Long term Debts (Debts)
Debt equity ratio =
Shareholder Funds or Equity
Long Term Debts may be defined as the debts which are payable after one year.
Generally, Debentures, Term Deposits and Public Deposits are long term debts for the
Company.
The shareholder funds include equity share capital, preference share capital and reserves
& surplus including accumulated profits. However fictitious assets like accumulated
296
Accounting
Ratios
deferred expenses, preliminary expenses, to the extent not written off, etc should be
deducted from the total of these items to achieve shareholder funds. The shareholder
funds so calculated are also known as net worth of the business.
Analysis
The Debt Equity Ratio of a firm should be compared with the industry average. The
reason is being that every industry has its own characteristics relating to capital
requirements. For example, in case of basic and heavy industries the Debt Equity Ratio
will be higher as compared to general manufacturing concerns.
Usually, a lower ratio is always safer, however too low ratio reflects an in-efficient use of
equity. Too high ratio reflects either there is a debt to a great extent or the equity base is
too small.
Measurement
The total debt ratio compares the total debts (long term debts as well as Short term debts)
with the total assets. Symbolically,
Total Debts
Total Debt ratio =
Total Assets
Analysis
The higher the Total Debt Ratio, more risky is the situation because all liabilities are to
be repaying sooner or later. In other words, higher liabilities imply greater financial risk
also. On the other hand, a low Debt Equity ratio implies a low risk to lenders and
creditors of the firm.
speculative situation because of the effect of high leverage and the greater possibility of
financial difficulty arising from excessive debt burden.
Measurement
It is computed by dividing proprietor’s (Shareholder’s) funds by total assets of the firm.
Proprietor’s Funds or Shareholder’s Funds
Proprietary Ratio =
Total Assets – Fictitious Assets
The shareholder’s funds include equity share capital, preference share capital, reserves
and surplus including accumulated profits. However fictitious assets like accumulated
deferred expenses, preliminary expenses, to the extent not written off, etc should be
deducted from the total of these items to derive shareholder’s funds. The shareholder
funds so calculated are also known as net worth of the business.
Analysis
A higher ratio indicates adequate safety for the creditors of business. However, a very
high ratio indicates improper mix of debt and equity of the business. It affects the
profitability of firm and consequently low return on investment. The mixing of debt and
equity shall be depends upon the Return on Investment of the firm.
Measurement
It is computed by dividing fixed assets by shareholder’s funds. This ratio is indicated in
percentage form.
Total Fixed Assets
Fixed Assets to Net Worth ratio = x 100
Net Worth
298
Accounting
Ratios
Measurement
It is computed by dividing the earnings before interest and tax (EBIT) by interest charges
on long term borrowings. This ratio is shown in times.
Earning Before Interest and Tax (EBIT)
Analysis
Higher the ratio, better is the performance of business in payment of interest to the
lenders and creditors. The lower the ratio, the more the company is burdened by debt
expense. When a company’s interest coverage ratio is only 1.5 or lower, its ability to
meet interest expenses may be questionable.
Turnover ratios are also known as activity ratios or efficiency ratios or performance ratios
with which a firm manages its current assets. These ratios look at how well a company
turns its assets into revenue as well as how efficiently a company converts its sales into
cash, i.e how efficiently and effectively a company is using its resources to generate sales
and enhancing Shareholder’s value. The better these ratios, the better it is for
shareholders of the business.
The following turnover ratios may be computed to judge the effectiveness of usage of
assets of the firm.
1. Inventory Turnover Ratio
2. Debtor Turnover Ratio
3. Creditor Turnover Ratio
4. Assets Turnover Ratio
The average inventory is the simple average of the opening and closing balance of
inventory during the year i.e. (Opening inventory + Closing inventory) / 2. In certain
circumstances opening balance of the inventory may not be known then closing balance
of inventory may be considered as average inventory of the firm.
Analysis
Inventory Turnover Ratio is a test of efficient inventory management. There is no ideal
standard for evaluating an Inventory Turnover Ratio of a firm so it should be compared
with the Inventory Turnover Ratio of the other firm or past Inventory Turnover Ratio of
the same firm. if the Inventory Turnover Ratio for a year below the range, it may signal
inactive stock. While the Inventory turnover ratio beyond the range may indicate
insufficient inventory signaling a risk of stock outs.
Higher the Days of Inventory Holding Period, Higher would be investments in the stocks
of the Company. This ratio would also affect the current ratio of the company.
Measurement
It is determined by dividing the net credit sales by average debtors. Symbolically,
Annual Net Credit Sales
Debtors Turnover Ratio =
Average Receivables
300
Accounting
Ratios
It is important to note that while calculating the Debtors turnover ratio, we do not deduct
provision for doubtful debtors and provision for discount on debtors from the book value
of debtors. Because the purpose of measurement is to know the conversion times of
debtors from sales instead of amount to be received from debtors.
Net credit sales consist of gross credit sales minus sales return. Trade debtor includes
sundry debtors and bill’s receivables. Average trade debtors shall be (Opening and
Closing balances of debtors and bills receivable / 2)
When the information about credit sales, opening and closing balances of trade debtors is
not available then the ratio can be calculated by dividing total sales by closing balances
of trade debtor.
Analysis
A higher ratio is better since it would indicate that debts are being collected more
promptly. If the organization collecting funds promptly, it may be invested else where for
other use. For measurement and comparison, the firm should obtain the DTR from
industry or other firm of the same industry. A ratio lower than the standard shows that
inefficiency in collection and higher investment in debtors.
The shorter the average collection period, the better the quality of debtors, as a short
collection period implies the prompt payment by debtors. An excessively long collection
period implies a very liberal and inefficient credit and collection performance and
policies. The delay in collection of cash impairs the firm’s liquidity. On the other hand,
too low collection period is not necessarily favorable, rather it may indicate a very
restrictive credit and collection policy which may curtail sales and hence adversely affect
profit.
301
Cost and Management
Accounting
Measurement
It is computed by dividing the net credit purchases by average creditors of the business.
As given below,
Net Credit Purchase
Creditor Turnover Ratio =
Average Trade Creditors
Net credit purchases consist of gross credit purchases minus purchase return during the
year.
When the information about credit purchases not give in question, then the ratio is
calculated by dividing total purchases by the closing balance of trade creditors.
Analysis
A high turnover ratio shows the less availability of credit period or early payments. It also
indicates that the firm is not availing full credit period. A very low or lower turnover ratio
shows the availability of more credit period or delayed payments. This ratio also affects
current ratio of the firm. A lower ratio shows higher liquidity toward the firm and vice
versa.
302
Accounting
Ratios
Net assets represent total assets minus current liabilities, Intangible and fictitious assets
like goodwill, patents, accumulated losses, deferred expenditure and so on.
Net fixed assets represent the cost of fixed assets minus depreciation.
Net Working capital is represented by the difference between current assets and current
liabilities of the firm during the period.
Profitability is the ability of a business to earn profit over a period of time. The
profitability ratios show the combined effects of liquidity, asset management (activity)
and debt management (gearing) on operating results. The overall measure of success of a
business is the profitability which results from the effective use of its resources.
The profitability ratio of the firm can be measured by calculating various profitability
ratios. General two groups of profitability ratios are calculated. Given below,
a. Profitability in relation to sales.
b. Profitability in relation to investments.
4. Operating Ratio
5. Expenses Ratio
It measures the relationship between gross profit and sales. This ratio is also called the
average mark up ratio. It is calculated by dividing gross profit by sales.
Gross Profit X 100
Gross profit ratio =
Net sales
Gross profit is the difference between sales and cost of goods sold. Higher the ratio, the
higher is the profit earned on sales. This ratio should be adequate to cover the
administrative, selling and distribution expenses and to provide for fixed charges of
finance, dividends and building up the reserves.
304
Accounting
Ratios
Operating profit is the difference between net sales and total operating expenses.
Operating profit = Net sales – cost of goods sold – administrative expenses –
selling and distribution expenses
Lower the Operating ratio, better is the performance of company because it would leave
higher margin to meet interest, dividend etc.
305
Cost and Management
Accounting
It is computed by dividing the Earnings After Tax (EAT) by gross capital employed.
Capital Employed = Equity Share Capital + Preference Share Capital + Reserve &
Surplus + Long term Debts – Fictitious Assets – Non Operating Assets such as
Investments
Returns on capital employed judges the overall performance of the firm. This ratio must
be compared with the past ratio of the firm or with the industry average.
been utilized by the firm. This ratio is also known as return on shareholders fund (equity
share holder as well as preference share holder).
It is calculated by dividing Earnings after tax (EAT) by shareholder capital employed.
This ratio indicates that how well the funds of owner have been used by the firm. This
ratio also shows that how much the firm is earning for its owner or shareholders.
Equity shareholders are entitled to all the profits remaining after the all outside claims
including dividends on preference share capital are paid in full. The earnings may be
distributed to them or retained in the business. Return on equity share capital
investments or capital employed establishes the relationship between earnings after tax
and preference dividend and equity shareholder investment or capital employed or net
worth.
Higher percentage indicates the management is utilizing its equity base and the better
return is to investors of the firm.
307
Cost and Management
Accounting
Or
308
Accounting
Ratios
Illustration -1
Calculate Liquidity, Activity and Profitability Ratios and Solvency Ratios from the
Following financial statement for the year 2007-2008.
Balance Sheet
2006-2007 2007-2008
Cash 4,00,000.00 3,20,000.00
Sundry Debtors 6,40,000.00 8,00,000.00
Marketable Securities 4,00,000.00 6,40,000.00
Stock 36,80,000.00 43,20,000.00
Prepaid Expenses 56,000.00 24,000.00
Total Current Assets 51,76,000.00 61,04,000.00
Total Assets 1,12,00,000.00 1,28,00,000.00
Current Liabilities 12,80,000.00 16,00,000.00
Loans 32,00,000.00 32,00,000.00
Capital 40,00,000.00 40,00,000.00
Accumulated Earnings 9,36,000.00 16,24,000.00
Solution
Liquidity Ratios
309
Cost and Management
Accounting
Profitability Ratios
Activity ratios
Cost of Goods Sold Rs. 56,00,000
Inventory Turnover Ratio = 1.4
Average Inventory Rs. 40,00,000
Solvency Ratios
Illustration – 2
With the help of following information, complete the Balance Sheet of ABC Limited.
Owner’s Fund Rs. 1,00,000
Current Debt to Total Debt 0.40
Total Debt to total Equity 0.60
Fixed Assets to Owner’s Equity 0.60
Total Assets Turnover Ratio 2.00
Inventory Turnover Ratio 8.00
310
Accounting
Ratios
Solution
Total Debt is 60% of Total Equity, therefore, Rs. 60,000.
Current Debt is 40% of Total Debt i.e. 40% of Rs. 60,000 or Rs. 24,000
Long Term Debt = Rs. 60,000 – Rs. 24,000 = Rs. 36,000
Fixed Assets is 60% of Owner’s Fund, therefore, Rs. 60,000
311
UNIT-15 BUDGETING – I
LEARNING OBJECTIVES
15.1 Introduction
15.2 Definition of Budget
15.3 Control
15.4 Budgetary Control
15.5 Objectives of Budget
15.6 Steps Involved in the preparation of budgets
15.7 Classification of Budgets
15.8 Classification on the Basis of Period
15.8.1 Long Term Budgets
15.8.2 Short Term Budgets
15.9 Classification on the Basis of Coverage
15.9.1 Master Budgets
15.9.2 Functional Budgets
15.9.2.1 Production Budgets
15.9.2.2 Sales Budget
15.9.2.3 Cash Budget
15.1 INTRODUCTION
All the business is full from uncertainty of return and high risk of return. This uncertainty
of risk and return may be controlled, almost minimized, by different managerial tools and
techniques. Budgeting is one of the most useful and widely used management technique
of planning and controlling in business. Therefore, the budgetary control becomes an
essential tool of management for planning and controlling of cost and profit.
In modern business, management plans its operation in advance for effective and efficient
utilization of its resources. Later on it is compared with the actual results from the
budgeted figures and reasons are investigated.
312
Budgeting - I
15.3 CONTROL
Control may be defined as comparing the results with the plans and take corrective
actions, whenever the results are in adverse. It is a technique according to which plan of
actions are guided for future. Control requires two things, specific plans according to
which the work is to be executed and compare the results with plan.
The reasons for variance must be investigate and accordingly the corrective and
preventive actions should be taken.
Budgetary control is the process of preparation of budgets and comparing the budgeted
figures for arriving at deviations and actions for future.
313
Cost and Management
Accounting
I. Budget Controller
Budget controller is responsible for the budget programme. The controller has
technical knowledge of the business and budgets.
II. Formulation of Budget Committee
The committee consists of all heads of various departments such as production, sales,
finance, personnel etc. it is the budget committee who discuss and approved the
budget figures.
Budgets are classifieds in the following categories on the basis of their nature.
I. On the basis of period
i. Long Term Budgets
ii. Short term Budgets
II. On the basis of coverage
i. Master Budgets
ii. Functional Budgets
III. On the basis of capacity
i. Flexible Budgets
ii. Fixed Budgets
IV. on the basis of conditions
i. Basic Budgets
ii.Current Budgets
Now the above all budgets are discussed in the following paragraphs
314
Budgeting - I
The above categories of budgets are only illustrative list of budgets prepared by the
organization. The above list may be varied in accordance with the nature of business and
their importance in organization. Such as Material Budgets shall be very important in the
manufacturing industry whereas purchase shall be more important in trading industry.
his sales is sufficient for these. Sales budget is prepared on the basis of age, geographical
location, weather and other factors in accordance with the nature of product of company.
Fixed Budget
Fixed budget is prepared for the fixed activity level of the business. In this budget, all the
expenditures and revenues are not classifieds as fixed, variable and semi-variable at the
particular level of activity. For example, the company has installed capacity of !0,000
units per day, used capacity 7,000 units per day and generating profit of Rs 50,000 per
day. If we prepare the fixed budget at utilization capacity up to 8,000 units per day. After
preparation of this budget, we would know that how much profit shall increase or
decrease along with change in other things.
Purchase Budget
Purchase budget is prepared to budget the expenditures on purchase of material and
services. In the purchase budget, we prepare, basically, two budgets. One is fort he
services to be acquired during the period and second is material to be purchase during the
period. During the preparation of purchase budget, various past data are used of the firm
such as material price, cost of various services.
Material Budgets
Material budget is prepared with the line of production budget. At the time of preparation
of material budget, one should keep in mind the requirement of material in production of
the company and the discount allowed on the material to be acquired by the vendor. The
discount allowed by the vendor should be compared with the Economic order quantity in
terms of cost of material.
316
Budgeting - I
Labour Budget
Human resource is an important factor for the organization. It must be utilized in the
optimum manner. In Labour budget, we budget the overall cost of various types of labour
required for the production such as skilled labour, unskilled labour and semi-skilled
labour. It is also prepared on the basis of gender, qualifications and age of the employee
and their experience. In labour budget, we also keep some provision for contingency for
labour such as staff welfare expenses, overtime payment, if any, and so on.
Overhead Budget
Overhead is also an important part of production budget. It is classified in the following
categories.
Administration Overhead
Administrations overhead are those overhead which is incurred by the company to
maintain the administration of the company. Administration includes the cost of
stationery used in office, salary to the directors and so on. Basically the administration
overhead is the cost for planning, organizing, directing and other function of the
management.
Flexible Budgets
Flexible budgets are those budgets which are prepared for different level of activities of
the business. However it may not prepared for the activity which is in excess of the
installed capacity of the business. This budget shall be in the line of fixed budget.
Basic Budgets
The budgets which are remains unchanged during a long period of time is known as basic
budgets. Such as budget for fixed assets and fixed costs.
Current Budgets
The budgets which are prepared for the short period of time and is related to the current
conditions of the business is known as current budget.
4. Expensive
The process of budgetary control is very expensive for the organization. It requires
various types of analysis and various meetings of the staff and so on.
318
UNIT-16 BUDGETING – II
LEARNING OBJECTIVES
16.1 Introduction
16.2 Sales Budget
16.3 Production Budget
16.4 Cash Budget
16.5 Importance of Cash Budget
16.6 Methods of preparing Cash Budget
16.6.1 Receipts and Payments Method
16.6.2 Adjusted Profit and Loss Accounts
16.6.3 Balance Sheet Method
16.7 Check Your Progress
16.1 INTRODUCTION
In the previous module, we studied the basic concepts of budget and their preparation in
the organization. We also discussed the steps required for budget preparation in the
organization. After preparation of budget, the organization can keep close control over
the various departments of organization. In this unit we shall discussed the various types
of budgets in details.
Sales are the backbone of the firm. An organization is known to be growing by the
amount of his sales during the period. The organization may earn and pay his expenses
only where his sales is sufficient for these.
At the single time, the organization may use two or more factors for the preparation of
sales budgets of the organization. For example, number of sales man and the
geographical area of the particular region may be used for the preparation of one sales
budget in the organization.
Illustration – 1
Life Care Exim Private Limited manufactures two types of products, Product A and
Product B. the company provides you the following data relating to sales for the period
ended 31st March 2008.
Sales in Kilogram
Quarter Product A Product B
First 2500 3200
Second 5900 1600
Third 5400 2000
Fourth 6200 1200
Selling Price per kg Rs. 24 Rs. 50
The targets fixed by the sales manager for the year 2008-2009 are given below.
Sales quantity increase (decrease) (20%) 25%
Selling price increase (decrease) (25%) (20%)
The company sales his product - A in area X, area Y and area Z, amounting 10%, 20%
and 70% respectively. However product - B in area X, area Y and area Z, amounting
70%, 20% and 10% respectively.
On the basis of the above information, prepare sales budget for the year 2008-2009.
Solution
Sales Budget (product-wise) for the year 2008-2009
Period Product – A Product – B
Units Rate Amount Units Rate Amount
Kg Rs. Rs. Kg Rs. Rs.
First 2000 30 60000 4000 40 160000
Second 4720 30 141600 2000 40 80000
Third 4320 30 129600 2500 40 100000
Fourth 4960 30 148800 1500 40 60000
8000 480000 5000 400000
320
Budgeting - II
Period X Y Z Total
Quarter Rs. Rs. Rs. Rs.
First 6000 12000 42000 60000
Second 14160 29320 99120 141600
Third 12960 25920 90720 129600
Fourth 14880 29760 104160 148800
Total 48000 96000 336000 480000
Period X Y Z Total
Quarter Rs. Rs. Rs. Rs.
First 112000 32000 16000 160000
Second 56000 16000 8000 80000
Third 70000 20000 10000 100000
Fourth 42000 12000 6000 60000
Total 280000 80000 40000 400000
Working Notes
Budgeted Sales for 2008 -2009 shall be 16000 Kg (20000 Kg – 4000 Kg)
Further the sales shall be divided in the ratio of 1:2:7 among the Area X, Area Y and
Area Z.
Sales Price of product – A for the year 2008-2009 shall be Rs. 30 (Rs. 24 + 25% of
Rs. 24)
Budgeted Sales for 2008 -2009 shall be 10000 Kg (8000 Kg + 2000 Kg)
Further the sales shall be divided in the ratio of 7:2:1 among the Area X, Area Y and
Area Z.
321
Cost and Management
Accounting
Sales Price of product – B for the year 2008-2009 shall be Rs. 40 (Rs. 50 - 20% of Rs.
50)
Production (in units) shall be computed with the help of following formula.
Budgeted Sales + Closing stock of finished goods – Opening Stock of finished goods
Illustration – 2
ABC Limited provides the following figures for the period of three months.
Particulars Product – A Product – B Product – C
Sales (Units)
January 100000 120000 40000
February 80000 100000 40000
March 120000 140000 40000
Sales Price Per Unit (Rs.) 10 20 40
Targets for subsequent quarter
Increase in sales quantity (%) 20 10 10
Increase in sales price (%) NIL 10 25
Opening Stock as on 01st January 50% of the sales shall be maintained as opening
stock for every month
Closing stock as on 31st March 40000 50000 10000
The company shall maintain closing stock for January and February at the percentage of
50% of the subsequent month sales.
322
Budgeting - II
Solution
Calculation of Sales (in units)
January February March
Product –A
Budgeted 100000 80000 120000
Add: Increase @ 20% 20000 16000 24000
TOTAL 120000 96000 144000
Product –B
Budgeted 120000 100000 140000
Add: Increase @ 10% 12000 10000 14000
TOTAL 132000 110000 154000
Product –C
Budgeted 40000 40000 40000
Add: Increase @ 10% 4000 4000 4000
TOTAL 44000 44000 44000
Cash budget shows the summary of cash inflows and cash outflows during a particular
period. Basically, it is prepared to know the surplus or deficit in the cash in future of the
organization. Usually, the cash budget is prepared on monthly basis. However it may be
prepared for quarter, half year and annually, if the company has sufficient data for
323
Cost and Management
Accounting
projections in the future. Cash budget is prepared with the help of all budgets such as
sales budget, production budget, overhead budget, material budget and capital
expenditure budget in the organization.
The receipts shall include the amount to be received from the borrowing and capital
raised during the period. It covers the amount received in advance from the customers or
amount held as security from the parties.
The payment shall include all the payments are required to pay during the period by the
company. The payment shall also include the payment of long term debt and their interest
during the period.
324
Budgeting - II
Illustration – 3
ABC Limited provides you the following information for half year ended on June 2008.
Prepare the cash budget from the following information.
i. The company is doing any sales activity during the first month of its operation. In
subsequent month, the company is doing the sales activity in cash 25% only.
ii. The payments to the creditors for material purchase shall be done on monthly basis
from the date of invoice received.
iii. The wages and salaries shall be paid on fortnightly basis, on 22 nd and 07th of the
month.
iv. Other expenses shall be paid on one month basis except the selling and distribution
expenses which are required to paid 5% immediately of the amount of selling and
distribution expenses.
v. All the expenses and receipts are accruing evenly during the period.
Particulars Rs.
Sales 720000
Manufacturing Expenses 96000
Material consumed 300000
Wages and salaries 120000
Administrative Expenses 108000
Selling and distribution expenses 84000
Depreciation on fixed assets 100000
The company has the registration under Central Excise Act in India. The product is
classified for 10% duty on the selling price. The duty is payable at the end of every quarter
of the calendar for the sales up to February, May, August and November respectively.
1. State the factors which shall be consider during the preparation of sales budget of
the company.
………………………………………………………………………………………………
………………………………………………………………………………………………
………………………………………………………………………………………………
……………………
326
Budgeting - II
4. Prepare the sales budget from the following information for the previous year.
5. Prepare the cash budget for the second quarter of the calendar year from the
following information.
Particulars February March April May June
Sales 37500 42000 45000 60000 67500
Material Consumed 22500 24000 26250 30000 30000
Wages 4500 4875 5250 6750 7250
Factory Overhead 3750 4125 4500 5625 7000
Administration Overhead 3000 3000 3000 3000 3500
Selling and Distribution 2250 2250 2625 3285 3500
overhead
Adjustments
1. Materials are purchased at the credit period of two months from the date of
purchase.
2. 20% of the sales are made for cash and balance is recovered after one month
immediately succeeding the month of sales.
3. All the payments of expenses including the amount of overhead are required to pay
after one month.
4. Advance tax for Income Tax is due on 15th June of the month amounting Rs 28750.
327